80
S 03 - 2019

lovetoan.files.wordpress.com€¦ · 2 NGUYŽN TĂNG VŨ - NGUYŽN NG¯C DUY - VƯƠNG TRUNG DŨNG LÊ PHÚC LÚ - TRƒN BÁ Đ—T T−P SAN TOÁN H¯C STAR EDUCATION SŁ thø 03

  • Upload
    others

  • View
    3

  • Download
    0

Embed Size (px)

Citation preview

Page 1: lovetoan.files.wordpress.com€¦ · 2 NGUYŽN TĂNG VŨ - NGUYŽN NG¯C DUY - VƯƠNG TRUNG DŨNG LÊ PHÚC LÚ - TRƒN BÁ Đ—T T−P SAN TOÁN H¯C STAR EDUCATION SŁ thø 03

TẬP SAN TOÁNSTAR EDUCATION

Số 03 - 2019

T R U N G T Â M S T A R E D UCA T I O N

Page 2: lovetoan.files.wordpress.com€¦ · 2 NGUYŽN TĂNG VŨ - NGUYŽN NG¯C DUY - VƯƠNG TRUNG DŨNG LÊ PHÚC LÚ - TRƒN BÁ Đ—T T−P SAN TOÁN H¯C STAR EDUCATION SŁ thø 03

Mục lục

1 Công thức nội suy Lagrange 3

2 Một số kĩ thuật giải phương trình hàm trên tập R+ 16

3 Định lý Wolstenholme và ứng dụng 33

4 Bổ đề ERIQ và ứng dụng 43

5 Đếm bằng hai cách trong tổ hợp 54

6 Đề thi tham khảo hướng tới VMO 2019-2020 63

1

Page 3: lovetoan.files.wordpress.com€¦ · 2 NGUYŽN TĂNG VŨ - NGUYŽN NG¯C DUY - VƯƠNG TRUNG DŨNG LÊ PHÚC LÚ - TRƒN BÁ Đ—T T−P SAN TOÁN H¯C STAR EDUCATION SŁ thø 03

2

NGUYỄN TĂNG VŨ - NGUYỄN NGỌC DUY - VƯƠNG TRUNG DŨNG

LÊ PHÚC LỮ - TRẦN BÁ ĐẠT

TẬP SAN TOÁN HỌC

STAR EDUCATIONSố thứ 03 - 2019

Tập san Toán học STAR EDUCATION vừa qua đã ra mắt được hai số và được đôngđảo bạn đọc đón nhận. Và như kế hoạch lúc đầu, mỗi năm STAR sẽ giới thiệu:

• Quyển 1: ra mắt vào cuối học kỳ 1, hướng tới kỳ thi HSG quốc gia.

• Quyển 2: ra mắt vào cuối học kỳ 2, hướng tới kỳ thi tuyển sinh THPT, Đại học.

Trong tập san lần này, đối tượng chủ yếu được hướng đến là các em học sinh đangchuẩn bị cho kỳ thi HSG quốc gia vào 3 ngày cuối tuần tới. Bên cạnh các chuyên đề,tập san cũng có ba đề ôn tập dành cho các thí sinh.

Về nội dung, chúng tôi có đủ các bài viết cho 5 phân môn, đặc biệt ngoài lực lượnggiáo viên trẻ tại STAR EDUCATION, Ban biên tập còn nhận được bài viết của thầyKiều Đình Minh, GV Chuyên Hùng Vương, Phú Thọ và em Trương Tuấn Nghĩa, học sinhTHPT Chuyên KHTN Hà Nội. Rất mong nhận được thêm sự đóng góp của bạn đọc gầnxa cho tập san để ngày một phong phú hơn, phục vụ tốt hơn cho cộng đồng dạy vàhọc Toán.

Dự kiến số tiếp theo, Tập san 04, sẽ được xuất bản vào tháng 05 tới với nội dung chủyếu dành cho các bạn chuẩn bị thi tuyển sinh 10 và tuyển sinh ĐH, xin đón nhận cácbài viết gửi về cho Ban biên tập. Mọi đóng góp xin gửi về các địa chỉ [email protected] hoặc [email protected].

Bản quyền thuộc trung tâm STAR EDUCATION, được đăng tải miễn phí trên mạng.

Mong rằng tài liệu này sẽ được đón nhận và được chia sẻ rộng rãi. Xin chân thànhcảm ơn.

Tập san Toán học STAR EDUCATION

Page 4: lovetoan.files.wordpress.com€¦ · 2 NGUYŽN TĂNG VŨ - NGUYŽN NG¯C DUY - VƯƠNG TRUNG DŨNG LÊ PHÚC LÚ - TRƒN BÁ Đ—T T−P SAN TOÁN H¯C STAR EDUCATION SŁ thø 03

Công thức nội suy Lagrange

Vương Trung Dũng(GV PTNK TP Hồ Chí Minh)

1. Giới thiệu

Cho đa thức P(x) có bậc nhỏ hơn n+ 1 và n+ 1 số thực phân biệt x i; i = 1, n+ 1. Khi

đó P(x) được xác định duy nhất như sau: P(x) =n+1∑

i=1P(x i) ·

n+1∏

j=1j 6=i

x−x j

x i−x j(*)

Chứng minh. Xét đa thức Q(x) = P(x)−n+1∑

i=1P(x i) ·

n+1∏

j=1j 6=i

x−x j

x i−x j. Khi đó degQ(x) ≤ n và

Q(x i) = 0 nên Q(x) có (n+ 1) nghiệm x i. Do đó Q(x)≡ 0.Tính duy nhất của P(x)được suy ra ngay từ nhận xét rằng hai đa thức bậc nhỏ hơnhoặc bằng n nhận giá trị bằng nhau tại (n+ 1) điểm thì chúng trùng nhau.

Từ công thức trên, ta suy ra ngay kết quả quan trọng sau:Cho đa thức P(x) có bậc không quá n và nhận giá trị hữu tỉ tại n+ 1 số hữu tỉ khácnhau. Chứng minh P(x) ∈Q[x].

Công thức nội suy Lagrange có thể dùng để:

1. Tính giá trị của một đa thức tại một điểm (cho biết giá trị tại n+ 1 điểm phânbiệt, tính giá trị tại điểm mới.

2. Tính tổng liên quan đến các đẳng thức tổ hợp. Chứng minh các BĐT.

3. Nội suy liên quan đến số hữu tỷ, đa thức nguyên.

2. Các bài toán chọn lọc

Bài toán 1. Cho n số phân biệt a1, a2, ..., an và đa thức P(x) có bậc deg P(x)≤ n−2.Chứng minh rằng

T =P(a1)

(a1 − a2)(a1 − a3)...(a1 − an)+ ...+

P(an)(an − a1)(an − a2)...(an − an−1)

= 0.

3

Page 5: lovetoan.files.wordpress.com€¦ · 2 NGUYŽN TĂNG VŨ - NGUYŽN NG¯C DUY - VƯƠNG TRUNG DŨNG LÊ PHÚC LÚ - TRƒN BÁ Đ—T T−P SAN TOÁN H¯C STAR EDUCATION SŁ thø 03

4 VƯƠNG TRUNG DŨNG

Lời giải. Áp dụng công thức nội suy Lagrange ta được:

P(x) = P(a1)(x − a2)(x − a3)...(x − an)(a1 − a2)(a1 − a3)...(a1 − an)

+ ...+ P(an)(x − a1)(x − a2)...(x − an−1)(an − a1)(an − a2)...(an − an−1)

Đồng nhất hệ số của xn−1 ở hai vế ta thu được T = 0 (đpcm).

Nhận xét. Từ đó, ta có cơ sở phương pháp hệ số bất định như sau:Cho a1, a2, ..., an là n số thực đôi một phân biệt và deg f ≤ n− 1. Chứng minh rằng tồntại các số thực A1, A2, ...,An sao cho

f (x)(x − a1)...(x − an)

=A1

x − a1+

A2

x − a2+ ...+

An

x − an.

Bài toán 2. India 2001 Cho a ≥ 3 và p(x) ∈ R[x], degp = n. Chứng minh rằng

max{|a j − p( j)| : 0≤ j ≤ n+ 1} ≥ 1.

Lời giải:Đặt y j = p( j); j = 1, n+ 1. Khi đó đa thức p(x) là đa thức duy nhất có bậc không quán+ 1 và nhận giá trị bằng yk khi x = j nên theo công thức nội suy Lagrange

p(x) =n+1∑

j=0

y j(x − 0)...(x − ( j − 1))(x − ( j + 1))...(x − (n+ 1))( j − 0)...( j − ( j − 1))( j − ( j + 1))...( j − (n+ 1))

.

Vì degp = n nên hệ số của xn+1 là

n+1∑

j=0

y j

j!(n+ 1− j)!(−1)n+1− j= 0.

Nhân hai vế cho (n+ 1)! ta được

n+1∑

j=0

Cn+1j (−1)n+1− j y j = 0. (∗)

Giả sử rằng a j − 1< y j < a j + 1,∀ j. Khi đó

(−1)n+1− ja j − 1< (−1)n+1− j y j < (−1)n+1− ja j + 1,∀ j.

Do đón+1∑

j=0Cn+1

j (−1)n+1− j y j >n+1∑

j=0Cn+1

j ((−1)n+1− ja j − 1) = (a − 1)n+1 − 2n+1 ≥ (3 −

1)n+1 − 2n+1 = 0, mâu thuẫn với (*). Từ đó suy ra điều phải chứng minh.

Bài toán 3. Với mỗi cặp số nguyên m, n mà 1≤ m≤ n, đặt Rmn =

m∑

k=0(m−k)n(−1)kC k

n+1.

Với các số nguyên như thế chứng minh rằng Rn−m+1n = Rm

n .

Tập san Toán học STAR EDUCATION

Page 6: lovetoan.files.wordpress.com€¦ · 2 NGUYŽN TĂNG VŨ - NGUYŽN NG¯C DUY - VƯƠNG TRUNG DŨNG LÊ PHÚC LÚ - TRƒN BÁ Đ—T T−P SAN TOÁN H¯C STAR EDUCATION SŁ thø 03

VƯƠNG TRUNG DŨNG

Lời giải. Trước tiên để ý rằng 1≤ m≤ n⇒ 1≤ n−m+ 1≤ n. Từ đó theo định nghĩaĐặt

Rn−m+1n =

n−m+1∑

k=0(n−m+ 1− k)n(−1)kC k

n+1 =n−m+1∑

k=0

(n+ 1− k)−m�n(−1)kCn+1−k

n+1

=n+1∑

i=m(i −m)n(−1)n+1−iC i

n+1 (thay i = n+ 1− k⇒ k = n+ 1− i)

=n+1∑

i=m(m− i)n(−1)n(−1)n+1−iC i

n+1 = −n+1∑

k=m(m− k)n(−1)kC k

n+1(thay k = n+ i).

(1.1)So sánh với công thức xác định của Rm

n và chú ý rằng (m− k)n(−1)kC kn+1 khi k = m ta

thấy Rn−m+1n = Rm

n khi và chỉ khi

n+1∑

k=0

(m− k)n(−1)kC kn+1 = 0. (2)

Để chứng minh (2) ta đặt P(x) = (m− x)n và các nút nội suy x j = j, 1≤ j ≤ n+1, ápdụng công thức nội suy Lagrange ta có

P(0)−n+1∑

k=1

P(k)ωk(0) = 0, (3)

trong đó ωk(x) =

i 6=k(x − x i)

i 6=k(xk − x i)

. Nhưng ta cũng có

ωk(0) =Π(−i).k(k− i).k

= (−1)n(n+ 1)!

i>k(k− i).

i>k(k− i)k

= −(−1)kC kn+1,

nên từ (3) ta suy ra được (2).

Bài toán 4. Cho p là số nguyên tố và P(x) ∈ Z[x] là một đa thức bậc d thoả mãn

1. P(0) = 0, P(1) = 1;

2. Với mọi số nguyên dương n thì số dư trong phép chia P(n) cho p là 0 hoặc 1.

Chứng minh rằng d ≥ p− 1.

Lời giải. Giả sử d ≤ p− 2. Áp dụng công thức nội suy Lagrange với p− 1 nút nội suyx i; i = 1, 2, ..., p− 2 ta được

P(x) =p−2∑

i=0

P(i)p−2∏

j=0, j 6=i

x − ji − j

.

Tập san Toán học STAR EDUCATION

Page 7: lovetoan.files.wordpress.com€¦ · 2 NGUYŽN TĂNG VŨ - NGUYŽN NG¯C DUY - VƯƠNG TRUNG DŨNG LÊ PHÚC LÚ - TRƒN BÁ Đ—T T−P SAN TOÁN H¯C STAR EDUCATION SŁ thø 03

6 VƯƠNG TRUNG DŨNG

Suy ra

P(p− 1) =p−2∑

i=0

P(i).(−1)p−iC ip−1.

Do p nguyên tố nên C ip−1 ≡ (−1)i (mod p), với mọi i = 1,2, ..., p− 2. Do đó

P(p− 1)≡ −p−2∑

i=0

P(i) (mod p).

Suy rap−1∑

i=0

P(i)≡ 0(mod p).

Nhưng từ hai điều điện của giả thiết ta lại cóp−1∑

i=0P(i)≡ k(mod p) với k = 1,2, ..., p−1,

vô lí. Vậy d ≥ p− 1.

Bài toán 5. Chứng minh rằng không tồn tại đa thức P ∈ R[x] có bậc dương sao choP(m) là số nguyên tố với mọi số nguyên dương m.

Lời giải. Giả sử tồn tại đa thức thoả yêu cầu đề bài khi đó ta chứng minh n!P(x) ∈Z[x].

Thật vậy, theo công thức nội suy Lagrange ta có P(x) =n∑

i=0P(i).

n∏

j=0, j 6=i

x − ji − j

nên

n!P(x) = x(x − 1)...(x − n)n∑

i=0

P(i)C in(−1)n−i

x − i∈ Z[x].

Chọn hai số nguyên tố p, q sao cho p, q > n và P(a) = p, P(b) = q với a, b là hai sốnguyên dương nào đó. Khi đó theo định đí thặng dư Trung Hoa, tồn tại số nguyêndương c sao cho

¨

c ≡ a(mod p)c ≡ b(mod q)

¨

n!P(c)≡ n!P(a)≡ 0(mod p)n!P(c)≡ n!P(b)≡ 0(mod q).

Do đó n!P(c)≡ 0(mod pq) Điều này là vô lí, vì P(c) là số nguyên tố và (n!, pq) = 1.

Bài toán 6. Cho đa thức P(x) có bậc n và thỏa mãn điều kiện P(k) =k

k+ 1; k =

0, 1,2...n. Tính P(n+ 1)

Lời giải. Áp dụng công thức nội suy Lagrange

P(x) =n∑

k=0

kk+ 1

x(x − 1)...(x − k+ 1)(x − k− 1)...(x − n)k(k− 1)...1(−1)...(k− n)

.

Tập san Toán học STAR EDUCATION

Page 8: lovetoan.files.wordpress.com€¦ · 2 NGUYŽN TĂNG VŨ - NGUYŽN NG¯C DUY - VƯƠNG TRUNG DŨNG LÊ PHÚC LÚ - TRƒN BÁ Đ—T T−P SAN TOÁN H¯C STAR EDUCATION SŁ thø 03

VƯƠNG TRUNG DŨNG

Từ đó

P(x) =n∑

k=0

kk+ 1

(n+ 1)...(n− k+ 2)(n− k)...1k(k− 1)...1(−1)...(k− n)(n− k+ 1)

=n∑

k=1(−1)n−kk

(n+ 1)!(k+ 1)!(n− k+ 1)!

=1

n+ 2

n∑

k=1(−1)n−kkC k+1

n+2

Bài toán 7. Cho đa thức P(x) có bậc không quá 2n và |P(x)| ≤ 1,∀k ∈ {−n,−n+1, ..., n− 1, n}. Chứng minh rằng

|P(x)| ≤ 4n,∀x ∈ [−n, n].

Lời giải. Ta chứng minh

n∏

i=−n,i 6=k

|x − i| ≤ (2n)! vàn∏

i=−n,i 6=k

|x − i|= (n+ k)!(n− k)!.

Suy ra

|P(x)| ≤n∑

k=−n

(2n)!(n+ k)!(n− k)!

=n∑

j=0

C j2n = 22n = 4n.

Bài toán 8. (Rumani 1981) Cho đa thức P(x) có bậc n và thỏa mãn P(x) =1

C kn+1

, n=

0, 1,2, ..., n. Tính P(n+ 1).

Lời giải. Áp dụng công thức nội suy Lagrange cho P(x) tại n+1 nút nội suy là 0,1, ..., nta được

P(x) =n∑

k=0

P(k)�

n∏

i 6=k,i=0

x − ik− i

Màn∏

k=0(k− i) = k(k− 1)...(k− k+ 1)(k− k− 1)...(k− n) = k!(n− k)!(−1)n−k. Do đó

P(x) =n∑

k=0

(−1)n−k(n− k+ 1)!(n+ 1)!(n− k)!

(x − i)],

suy raP(n+ 1) =

(−1)n−k =∑

(−1) j

(vì∏

(n+ 1− i) =(n+ 1)!n+ 1− k

). Dẫn đến

P(n+ 1) =

¨

0, n= 2m+ 1

1, n= 2m.

Tập san Toán học STAR EDUCATION

Page 9: lovetoan.files.wordpress.com€¦ · 2 NGUYŽN TĂNG VŨ - NGUYŽN NG¯C DUY - VƯƠNG TRUNG DŨNG LÊ PHÚC LÚ - TRƒN BÁ Đ—T T−P SAN TOÁN H¯C STAR EDUCATION SŁ thø 03

8 VƯƠNG TRUNG DŨNG

Nói cách khác:

0=n+1∑

i=0

(−1)iC in+1P(i) = (−1)n+1P(n+ 1) +

¨

1, n= 2k0, n= 2k+ 1.

Từ đó P(n+ 1) =

¨

1, n= 2k0, n= 2k+ 1.

Bài toán 9. Cho đa thức P(x) = xn+ a1 xn−1+ ...+ an ∈ R[x]. Giả sử x0, x1, ..., xn làcác số nguyên thỏa x0 > x1 > ... > xn. Chứng minh rằng tồn tại k ∈ {0, 1, ..., n} sao

cho |P(xk)| ≥n!2n

.

Lời giải. Theo công thức nội suy Lagrange, ta có

P(x) =n∑

i=0

P(x i)n∏

j 6=i, j=0

x − x j

x i − x j,

so sánh hệ số bậc cao nhất ở hai vế ta được

1=n∑

i=0

n∏

j 6=i, j=0

P(x i)x i − x j

.

Vì x0, x1, ..., xn là một dãy giảm ngặt các số nguyên nên ta có

n∏

j 6=i, j=0|x j − x i| =

i−1∏

j=0|x j − x i|

n∏

j=i+1|x i − x j|

≥ i!(n− i)!=1n!

C in.

Đặt |P(xk)|= max¦

|P(x0)|, |P(x1)|, ..., |P(xn)|©

. Theo bất đẳng thức tam giác ta có

1≤n∑

i=0

|P(x i)|n∏

j 6=i, j=0|x j − x i|

≤|P(xk)|

n!

n∑

i=0

C in =

2n|P(xk)|n!

.

Suy ra |P(xk)| ≥n!2n

.

Bài toán 10. Giả sử a1, a2, ..., an là những số nguyên dương thoả mãn ai 6= a j khi

i 6= j. Chứng minh rằng với bất kì số nguyên dương k ta luôn cón∑

i=1

aki

j 6=i(ai − a j)

cũng

là một số nguyên.

Tập san Toán học STAR EDUCATION

Page 10: lovetoan.files.wordpress.com€¦ · 2 NGUYŽN TĂNG VŨ - NGUYŽN NG¯C DUY - VƯƠNG TRUNG DŨNG LÊ PHÚC LÚ - TRƒN BÁ Đ—T T−P SAN TOÁN H¯C STAR EDUCATION SŁ thø 03

VƯƠNG TRUNG DŨNG

Lời giải. Xét hai đa thức f (x) = x k và g(x) =n∏

i=1(x − ai). Ta thực hiện phép chia

f (x) = q(x)g(x) + r(x), trong đó degr(x) ≤ n − 1, suy ra f (ai) = r(ai) với mọii = 1, 2, ..., n. Theo công thức nội suy Lagrange, ta có

r(x) =n∑

i=1

r(ai)∏

j 6=i

x − a j

ai − a j.

Dễ thấy r(x) là đa thức hệ số nguyên và r(ai) = f (ai) = aki với mọi i = 1, 2, ..., n.

Hệ số cao nhất của r(x) là hệ số của xn−1 và bằngn∑

i=1

aki

i 6= j(ai − a j)

. Từ đây suy ra

n∑

i=1

aki

i 6= j(ai − a j)

là một số nguyên.

Bài toán 11. Trong mặt phẳng toạ độ, một điểm được gọi là hỗn tạp nếu một tronghai toạ độ của nó là số hữu tỉ còn toạ độ còn là là số vô tỉ. Tìm tất cả các đa thức màđồ thị của nó không chứa bất cứ một điểm hỗn tạp nào.

Lời giải. Ta gọi một đa thức là thuần khiết nếu nó không chứa bất cứ một điểm hỗntạp nào. Ta sẽ chứng minh chỉ có những chỉ có những đa thức bậc nhất hệ số hữu tỉlà thoả yêu cầu đề bài.Giả sử tồn tại P(x) = an xn+an−1 xn−1+ ...+a1 x+a0, n≥ 2 là một đa thức thuần khiết,từ đó suy ra P(x) nhận giá trị hữu tỉ tại mọi giá trị hữu tỉ của biến x nên theo côngthức nội suy Lagrange P(x) ∈Q[x].Gọi m là mẫu chung lớn nhất của các ak thế thì P(mx) và −P(mx) là các đa thứcthuần khiết với hệ số nguyên và trong hai đa thức này có một đa thức có hệ số bậccao nhất là số nguyên dương. Do đó không mất tổng quát có thể giả sử an > 0 và ak

là các số nguyên.Gọi p là số nguyên tố sao cho p không là ước của an. Gọi r là số nguyên đủ lớn sao

cho tồn tại số dương x0 thoả mãn P(x0) =pr + 1

p(Vì hệ số bậc cao nhất của P là số

dương nên số r như thế là tồn tại). Vì P(x0) là số hữu tỉ nên x0 cũng là số hữu tỉ, ta

viết x0 =st, với (s, t) = 1. Khi đó

pr + 1p

= P(x0) =ansn + an−1sn−1 t + ...+ a0 tn

tn

hay(pr + 1)tn = p(ansn + an−1sn−1 t + ...+ a0 tn).

Vì p chia hết vế phải nên p cũng chia hết vế trái, dẫn đến p|t. Hơn nữa, vì n ≥ 2 vếtrái chia hết cho p2, dẫn đến

p|(ansn + an−1sn−1 t + ...+ a0 tn).

Vì p|t nên p chia hết an−1sn−1 t + ...+ a0 tn suy ra p|ansn.Vì p|t và (s, t) = 1 nên p không là ước của s và do đó theo sự tồn tại của p ta có p

Tập san Toán học STAR EDUCATION

Page 11: lovetoan.files.wordpress.com€¦ · 2 NGUYŽN TĂNG VŨ - NGUYŽN NG¯C DUY - VƯƠNG TRUNG DŨNG LÊ PHÚC LÚ - TRƒN BÁ Đ—T T−P SAN TOÁN H¯C STAR EDUCATION SŁ thø 03

10 VƯƠNG TRUNG DŨNG

không là ước của an. Điều này cho ta p không là ước của ansn, mâu thuẫn. Từ đó suyra không có một đa thức thuần khiết nào với bậc lớn hơn 1.Như vậy mọi đa thức thuần khiết có bậc là 0 hoặc 1. Mọi đa thức hằng đều có điểmhỗn tạp. Dễ dàng kiểm tra mọi đa thức bậc nhất với hê số hữu tỉ đều thoả yêu cầu đềbài. Ta có điều phải chứng minh.

Bài toán 12. (USAMO 2002) Cho F(x) ∈ R[x] là một đa thức monic có bậc n. Chứngminh rằng F(x) là trung bình cộng của hai đa thức bậc n có đủ n nghiệm thực.

Lời giải. Gọi y1, y2, ..., yn là các số thực thoả

¨

yi < min{0, 2F(i)}, nếu i lẻ

yi > max{0,2F(i)}, nếu i chẵn.

Theo công thức nội suy Lagrange tồn tại một đa thức bậc nhỏ hơn n sao cho P(i) = yi,với i = 1, 2, ..., n. Đặt

G(x) = P(x) + (x − 1)(x − 2)...(x − n) và H(x) = 2F(x)− G(x).

Khi đó G(x), H(x) là hai đa thức monic, hệ số thực có bậc n và có trung bình cộng làF(x). Vì y1, y3, y5, ... < 0 và y2, y4, y6... > 0, G(i) = yi, G(i + 1) = yi+1 trái dấu nhaunên G(x) có nghiệm thuộc đoạn [i, i + 1], với mọi i = 0,1, ..., n− 1. Do đó G(x) cóít nhất n− 1 nghiệm, nghiệm còn lại cũng là nghiệm thực vì nếu ngược lại nó sẽ cóthêm một nghiệm phức liên hợp.Tương tự, nếu i là số lẻ thì G(i) = yi < 2F(i) dẫn đến H(i) = 2F(i)− G(i) > 0, cònnếu i là số chẵn thì G(i) = yi > 2F(i) dẫn đến H(i) = 2F(i)−G(i)< 0. Điều này cũngcho ta H(x) có đủ n nghiệm thực như trong trường hợp của G(x).

Bài toán 13. Cho a1, a2, a3, a4, b1, b2, b3, b4 là các số thực thoả bi − a j 6= 0, với mọii, j = 1, 2,3, 4. Giả sử hệ phương trình

X1

b1 − a1+

X2

b1 − a2+

X3

b1 − a3+

X4

b1 − a4= 1

X1

b2 − a1+

X2

b2 − a2+

X3

b2 − a3+

X4

b2 − a4= 1

X1

b3 − a1+

X2

b3 − a2+

X3

b3 − a3+

X4

b3 − a4= 1

X4

b4 − a1+

X2

b4 − a2+

X3

b4 − a3+

X4

b4 − a4= 1

có nghiệm duy nhất. Hãy tính X1 + X2 + X3 + X4 theo ai, bi.

Tập san Toán học STAR EDUCATION

Page 12: lovetoan.files.wordpress.com€¦ · 2 NGUYŽN TĂNG VŨ - NGUYŽN NG¯C DUY - VƯƠNG TRUNG DŨNG LÊ PHÚC LÚ - TRƒN BÁ Đ—T T−P SAN TOÁN H¯C STAR EDUCATION SŁ thø 03

VƯƠNG TRUNG DŨNG

Lời giải. Đặt P(x) =4∏

i=1(x − ai)−

4∏

i=1(x − bi). Khi đó hệ số của x3 trong P(x) là

4∑

i=1

bi −4∑

i=1

ai.

Theo công thức nội suy Lagrange

P(x) =4∑

i=1

P(ai)4∏

j=1, j 6=i

x − a j

ai − a j.

Vì hệ số của x3 trong tích4∏

j=1, j 6=i(x − a j) là 1 nên hệ số của x3 trong P(x) là

4∑

i=1

P(ai)4∏

j=1, j 6=i(a j − ai)

.

Để ý rằngP(b j)

n∏

j=1, j 6=i(b j − ai)

= b j−ai which are the denominator of the 4 given equations!

Với j = 1,2, 3,4 cho x = b j vào trong công thức nội suy của P(x) và chia cả hai vếcho P(b j) ta được

1=4∑

i=1

P(ai)P(b j)

4∏

j=1, j 6=i

b j − ai

a j − ai=

4∑

i=1

P(ai)

(b j − ai)4∏

j=1, j 6=i(ai − a j)

.

So sánh với hệ phương trình đã cho, vì tính duy nhất nghiệm nên ta có X i =P(ai)

n∏

j 6=i, j=1(ai − a j)

,

với i = 1, 2,3, 4. Do đó4∑

i=1

X i =4∑

i=1

P(ai)4∏

j=1, j 6=i(ai − a j)

=4∑

i=1

bi −4∑

i=1

ai.

Bài toán 14. Cho các số thực a1, a2, ..., an và đa thức P(x) = (a−a1)(x−a2)...(x−an).Chứng minh rằng tồn tại số nguyên dương c sao cho

max{|P(x)| : 0≤ x ≤ 2} ≤ cn.max{|P(x)| : 0≤ x ≤ 1}.

Lời giải. Đặt S = max{|P(x)| : 0 ≤ x ≤ 1}. Với mỗi i = 0,1, ..., n đặt bi =in

fi(x) = (x − b0)...(x − bi−1)(x − bi+1)...(x − bn). Theo công thức nội suy Lagrange tacó

P(x) =n∑

i=0

P(bi)fi(x)fi(bi)

.

Tập san Toán học STAR EDUCATION

Page 13: lovetoan.files.wordpress.com€¦ · 2 NGUYŽN TĂNG VŨ - NGUYŽN NG¯C DUY - VƯƠNG TRUNG DŨNG LÊ PHÚC LÚ - TRƒN BÁ Đ—T T−P SAN TOÁN H¯C STAR EDUCATION SŁ thø 03

12 VƯƠNG TRUNG DŨNG

Với mỗi w ∈ [0, 2], |w− bk| ≤ |2− bk|, với k = 0,1, ..., n nên

| fi(w)| ≤ | fi(2)|=n∏

i=0(2−

in)

=2n(2n− 1)(2n− 2)...(n+ 1)

nn=(2n)!n!.nn

.

Do đó |P(bi)| ≤ S và

| fi(bi)|=i!(n− i)!

nn.

Theo bất đẳng thức tam giác

|P(w)| ≤n∑

i=0

|P(bi)|.�

fi(w)fi(bi)

�≤ Sn∑

i=0

C i2n.Cn

2n−i.

Từ đó ta cón∑

i=0

C i2nCn

2n−i ≤n∑

i=0

C i2nCn

2n = 22nCn2n ≤ 24n.

Suy ra

max{|P(w)| : 0≤ w≤ 2} ≤ 24nS = 16nmax{|P(x)| : 0≤ x ≤ 1}.

Bài toán 15. Tìm tất cả các đa thức P(x) ∈ R[x] sao cho với mọi số hữu tỉ r phươngtrình P(x) = r có nghiệm hữu tỉ.

Lời giải. Dễ thấy những đa thức bậc nhất P(x) = ax + b ∈ Q[x] thỏa yêu cầu đề bài.Ta sẽ chỉ ra rằng chỉ có những đa thức như vậy. Thật vậy, gọi P(x) là đa thức thỏa yêucầu đề bài, degP(x) = n. Với mỗi r = 0,1, 2, ..., n gọi x r ∈ Q sao cho P(x r) = r. Theocông thức nội suy Lagrange

P(x) =n∑

r=0

rn∏

i 6=r,i=0

x − x i

x r − x i.

Khai triển vế phải dễ thấy P(x) có hệ số hữu tỉ. Gọi M là mẫu thức chung của các hệsố và đặt Q(x) = M P(x) là đa thức có hệ số nguyên. Gọi k là hệ số bậc cao nhất củaQ(x) và c là hệ số tự do của P(x). Gọi p1, p2, p3... là một dãy các số nguyên tố. Khi đó

phương trình P(x) = c +pi

Mcó nghiệm t i ∈Q. Khi đó đa thức Q(x)− (cM + pi) có hệ

số bậc cao nhất là k còn hệ số tự do là −pi.

Vì Q(t i) = 0 nên t i =1di

hoặc t i =pi

di(với di là một ước của k). Vì các giá trị P(t i) là

phân biệt nên các giá trị của t i cũng phân biệt. Suy ra t i nhận giá trị1di

nhiều nhất là

bằng số ước của k, điều này dẫn đến tồn tại d|k sao cho có vô hạn lần t i =pi

d. Điều

này cho ta P(x)− (c +d xM) có vô hạn nghiệm.

Vậy P(x) = ax + b, với a =dM6= 0 và b, c ∈Q.

Tập san Toán học STAR EDUCATION

Page 14: lovetoan.files.wordpress.com€¦ · 2 NGUYŽN TĂNG VŨ - NGUYŽN NG¯C DUY - VƯƠNG TRUNG DŨNG LÊ PHÚC LÚ - TRƒN BÁ Đ—T T−P SAN TOÁN H¯C STAR EDUCATION SŁ thø 03

VƯƠNG TRUNG DŨNG

3. Bài tập tự luyện

Bài 1. Với mọi 1≤ j ≤ 2019 đặt

b j = j20192019∏

i=1,,i 6= j

(i2019 − j2019).

Tính1b1+

1b2+ ...+

1b2019

.

Bài 2. Cho P(x) là đa thức monic bậc n > 1. Biết rằng P(x) có n nghiệm thựcx1, x2, ..., xn phân biệt và khác 0. Chứng minh rằng

n∑

i=1

1x i P ′(x i)

=(−1)n+1

x1 x2...xn.

Bài 3. Bết f (x) và g(x) là hai đa thức bậc 2014 sao cho f (n)+ (−1)n g(n) = 2n, vớimọi n= 1, 2, ..., 4030. Tìm hệ số của x2014 trong đa thức g(x).

Bài 4. Cho P(x) là đa thức bậc không quá 2017 và thỏa mãn |P(k)| ≤ k+1, với mọik ∈ {0,1, ..., 2017}. Chứng minh rằng |P(2018)|+ |P(−1)| ≤ 2019(22018 − 1).

Bài 5. Chứng minh rằng không tồn tại đa thức P(x) ∈ R[x] với bậc n ≥ 1 sao choP(m) là số nguyên tố với mọi số nguyên dương m.

Bài 6. Chứng minh rằng với mỗi số nguyên dương n tồn tại đa thức P(x) ∈ Z[x] bậcn sao cho P(0), P(1), ..., P(n) phân biệt và các số đó đều có dạng 2.2019k + 3, k ∈ N.

Bài 7. Cho đa thức P(x) = a0 + a1 x + a2 x2 + ...+ an xn. Đa thức P được gọi là đathức nhỏ nếu thỏa mãn đồng thời

(i) Với mọi i = 1, 2, .., n ta đều có 0≤ ai < 1.

(ii) P(x) là số nguyên với mọi x .

Có bao nhiêu đa thức nhỏ có bậc không quá n?

Tập san Toán học STAR EDUCATION

Page 15: lovetoan.files.wordpress.com€¦ · 2 NGUYŽN TĂNG VŨ - NGUYŽN NG¯C DUY - VƯƠNG TRUNG DŨNG LÊ PHÚC LÚ - TRƒN BÁ Đ—T T−P SAN TOÁN H¯C STAR EDUCATION SŁ thø 03

14 VƯƠNG TRUNG DŨNG

Bài 8. Cho p là một số nguyên tố. Tìm số nguyên dương n nhỏ nhất sao cho tồntại đa thức P(x) bậc n thỏa mãn P(0) = 0, P(1) = 1 và với mọi λ ∈ N thì p |(P(λ)− 2)(P(λ)− 1)P(λ).

Bài 9. Cho p > 2 là một số nguyên tố và f là đa thức hệ số nguyên có bậc p− 1. Giảsử với mọi số nguyên a, b nếu a− b không chia hết cho p thì f (a)− f (b) cũng khôngchia hết cho p. Chứng minh rằng hệ số cao nhất của f chia hết cho p.

Bài 10. Cho dãy (εn)n∈N, trong đó εi = ±1. Chứng minh rằng dãy số này tuần hoànvới chu kì là lũy thừa của 2 khi và chỉ khi εn = (−1)P(n), trong đó P(x) ∈Q[x] là mộtđa thức giá trị nguyên (đa thức giá trịnguyên là đa thức thỏa P(n) ∈ Z,∀n ∈ Z).

Bài 11. Cho hai số nguyên t, n không nhỏ hơn 2. Tìm số nguyên m lớn nhất sao chotồn tại đa thức P(x) ∈ Q[x] có bậc n thỏa mãn : Với mỗi số nguyên k ∈ {1,2, ..., m}

thìP(k)

tklà số nguyên còn

P(k)tk+1

không là số nguyên.

Bài 12. Cho đa thức f (x) ∈ R[x] mà không là hằng số. Chứng minh rằng vớimỗi số c > 0, tồn tại số nguyên dương n0 thỏa mãn điều kiện: Nếu đa thức monicP(x) ∈ R[x] có bậc k ≤ n0 thì các số nguyên x thỏa mãn | f (P(x))| ≤ c không vượtquá k.

Tập san Toán học STAR EDUCATION

Page 16: lovetoan.files.wordpress.com€¦ · 2 NGUYŽN TĂNG VŨ - NGUYŽN NG¯C DUY - VƯƠNG TRUNG DŨNG LÊ PHÚC LÚ - TRƒN BÁ Đ—T T−P SAN TOÁN H¯C STAR EDUCATION SŁ thø 03

Tài liệu tham khảo

[1] Nguyễn Duy Thái Sơn, 2017 Một số bài toán liên quan đến chủ đề đa thức - Bàigiảng tập huấn giáo viên chuyên toán.

[2] Phạm Viết Huy, Tính chất số học trong một số bài toán đa thức.

[3] Kim Y.Li, Lagrange Interpolation Formula.

[4] Huỳnh Kim Linh, Nguyễn Minh Tuấn, Doãn Quang Tiến, 2019, Chuyên đề đathức và số học - Page "Tạp chí Tư liệu và toán học".

15

Page 17: lovetoan.files.wordpress.com€¦ · 2 NGUYŽN TĂNG VŨ - NGUYŽN NG¯C DUY - VƯƠNG TRUNG DŨNG LÊ PHÚC LÚ - TRƒN BÁ Đ—T T−P SAN TOÁN H¯C STAR EDUCATION SŁ thø 03

Một số kĩ thuật giải phương trình hàm trêntập R+

Trần Bá Đạt(GV THPT Gia Định, TP Hồ Chí Minh)

1. Bài toán mở đầu

Phần này trình bày hai bài toán hàm số, một bài yêu cầu tìm hàm xác định trên Rvà một bài yêu cầu tìm hàm xác định trên R+. Qua đó, ta sẽ nhận thấy được nhữngthuận lợi và khó khăn của việc giải phương trình hàm trên R+.

Bài toán 1. Tìm tất cả các hàm f : R −→ R thỏa mãn

(i) f (x + f (y)) = 2y + f (x), với mọi x , y ∈ R.

(ii) Với mọi x > 0, tồn tại y < 0 thỏa mãn f (y) = x .

Lời giải. Đặt a = −f (0)

2. Trong điều kiện thứ nhất, cho x = 0 và y = a, ta có

f ( f (a)) = 0.

Thay x = 0, y = f (a) vào đề bài, ta lại có

2 f (a) = 0.

Do đó f (a) = 0, kéo theo f (0) = f ( f (a)) = 0. Thay x = 0 vào phương trình đề cho,ta lại có

f ( f (y)) = 2y, với mọi y ∈ R.

Lấy f hai vế, ta suy ra

2 f (y) = f (2y), với mọi y ∈ R.

Thay y bởi f� y

2

vào phương trình đề cho, ta suy ra

f (x + y) = f�

x + f�

f� y

2

���

= f (x) + 2 f� y

2

= f (x) + f (y), với mọi x , y ∈ R.

16

Page 18: lovetoan.files.wordpress.com€¦ · 2 NGUYŽN TĂNG VŨ - NGUYŽN NG¯C DUY - VƯƠNG TRUNG DŨNG LÊ PHÚC LÚ - TRƒN BÁ Đ—T T−P SAN TOÁN H¯C STAR EDUCATION SŁ thø 03

TRẦN BÁ ĐẠT

Mặt khác, từ điều kiện thứ hai, với mọi x > 0, tồn tại y < 0 thỏa f (y) = x nên

f (x) = f ( f (y)) = 2y < 0.

Nói cách khác, f < 0 trên (0,+∞). Kết hợp với tính cộng tính, ta suy ra f đơn điệugiảm, kéo theo f (x) = ax . Thử lại, ta nhận f (x) = −

p2x làm nghiệm duy nhất của

bài toán.

Bài toán 2 (Gặp gỡ Toán học 2019). Tìm tất cả các hàm f : R+ −→ R+ thỏa mãn

f (x + f (y)) = 2y + f (x), với mọi x , y ∈ R+.

Lời giải.

Nhận xét. f đơn ánh.

Lấy a > b > 0 thỏa f (a) = f (b). Lần lượt thay y = a và y = b vào phương trình, tacó

f (x + f (a)) = 2a+ f (x),

vàf (x + f (b)) = 2b+ f (x), với mọi x > 0.

Do đó a = b, tức f đơn ánh.

Nhận xét. f (2x) = 2 f (x) với mọi x > 0.

Thay x bởi x + f (y) vào phương trình đề cho, ta có

f (x + 2 f (y)) = 2y + f (x + f (y))= 2y + 2y + f (x)= 4y + f (x), với mọi x , y > 0.

Mặt khác, thay y bởi 2y vào phương trình đề cho, ta cũng có

f (x + f (2y)) = 4y + f (x), với mọi x , y > 0.

Do đó

f (x + 2 f (y)) = f (x + f (2y)), với mọi x , y > 0.

Áp dụng tính đơn ánh của f , ta thu được

2 f (y) = f (2y), với mọi y > 0.

Nhận xét. f ( f (y)) = 2y với mọi y > 0.

Thay x = f (y) và y bởi 2y vào phương trình đề cho, ta có

f ( f (y) + f (2y)) = 4y + f ( f (y)) ,

hayf (3 f (y)) = 4y + f ( f (y)) , với mọi y > 0. (1)

Tập san Toán học STAR EDUCATION

Page 19: lovetoan.files.wordpress.com€¦ · 2 NGUYŽN TĂNG VŨ - NGUYŽN NG¯C DUY - VƯƠNG TRUNG DŨNG LÊ PHÚC LÚ - TRƒN BÁ Đ—T T−P SAN TOÁN H¯C STAR EDUCATION SŁ thø 03

18 TRẦN BÁ ĐẠT

Mặt khác, thay x = 2 f (y) vào phương trình đề cho, ta cũng có

f (3 f (y)) = 2y + 2 f ( f (y)), với mọi y > 0. (2)

Từ (1) và (2), ta thu được

f ( f (y)) = 2y, với mọi y > 0.

Thay y bởi f� y

2

vào phương trình đề cho, ta được

f�

x + f�

f� y

2

���

= 2 f� y

2

+ f (x),

hayf (x + y) = f (x) + f (y), với mọi x , y > 0.

Mà f > 0 trên (0,+∞) nên f tăng ngặt. Do đó f (x) = ax với mọi x > 0. Thay vàophương trình đề cho, ta tìm được a =

p2.

Vậy phương trình có nghiệm duy nhất f (x) =p

2x .

Cả hai bài toán trên đều được khai thác từ đẳng thức f (x + f (y)) = 2y + f (x). Tuynhiên, bài toán đầu tiên yêu cầu tìm hàm xác định trên R còn bài toán thứ hai yêucầu tìm hàm trên R+. Việc xử lí bài toán thứ hai (trên R+) khó khăn hơn bài toán thứnhất (trên R) vì các phép thế yêu cầu chặt chẽ hơn. Đặc biệt, với phương trình hàmtrên R+, ta không được phép thế giá trị 0, một phép thế xuất hiện ở hầu hết các bàiphương trình hàm trên R.Ngược lại, việc tìm hàm số xác định và nhận giá trị trên R+ có một điểm lợi là nếu tachứng minh được hàm số đã cho cộng tính thì ta sẽ thu được tính tuyến tính của hàmsố, thay vì phải chứng minh thêm các yếu tố phụ (đơn điệu, liên tục, bị chặn) như cácbài toán hàm số trên R.Vì lí do đó, bài viết này trình bày một số bài toán hàm số trên R+, đồng thời đưa ralời giải và một số kĩ thuật thường dùng đối với những bài toán như vậy.

2. Kiến thức chuẩn bị

Đầu tiên, ta trình bày một tính chất hàm thường được khai thác trong các bài toánhàm số trên R+ và liên hệ giữa nó với hàm tuần hoàn.

Định nghĩa 1 (Hàm trên tuyến tính). Hàm số f : R+ −→ R+ được gọi là trên tuyếntính nếu tồn tại a > 0 thỏa

f (x)≥ ax , với mọi x > 0.

Định nghĩa 2. Cho X , Y ⊂ R và f : X −→ Y và số thực dương T. Hàm f được gọi làtuần hoàn với chu kì T (gọi tắt là tuần hoàn) nếu với mọi x ∈ X thì x + T ∈ X và

f (x + T ) = f (x).

Tập san Toán học STAR EDUCATION

Page 20: lovetoan.files.wordpress.com€¦ · 2 NGUYŽN TĂNG VŨ - NGUYŽN NG¯C DUY - VƯƠNG TRUNG DŨNG LÊ PHÚC LÚ - TRƒN BÁ Đ—T T−P SAN TOÁN H¯C STAR EDUCATION SŁ thø 03

TRẦN BÁ ĐẠT

Định nghĩa 3. Cho hàm số f : R+ −→ R+ là hàm trên tuyến tính. Khi đó f khôngtuần hoàn.

Lời giải. Vì f trên tuyến tính nên tồn tại hệ số a > 0 thỏa

f (x)≥ ax , với mọi x > 0.

Giả sử f tuần hoàn với chu kì T > 0. Lấy x > 0 bất kì, ta có

f (x) = f (x + nT )≥ ax + anT, với mọi n ∈ N.

Cho n tiến tới +∞, ta có điều vô lí. Do đó T = 0.

Bên cạnh đó, nếu hàm f : R+ −→ R+ là cộng tính, ta cũng suy ra f (x) = ax nhờ kếtquả f cộng tính và f > 0 trên (0,+∞) thì f (x) = ax .

Tính chất 1. Cho hàm f : R+ −→ R+ là cộng tính. Khi đó f (x) = ax với mọi x > 0.

3. Một số kĩ thuật thường dùng

3.1. Chứng minh tính trên tuyến tính

Đầu tiên, ta đến với một bài toán xuất hiện tính trên tuyến tính và cách áp dụng đặctrưng này để giải phương trình hàm.

Bài toán 3 (Iran 2018). Cho hàm số f : R+ −→ R+ thỏa mãn

x + f (y)x f (y)

= f�

1y+ f

1x

��

, với mọi x , y > 0.

a) Chứng minh rằng f (x)≥ x với mọi x > 0.

b) Tìm f .

Lời giải. a) Phương trình được viết lại thành

1f (y)

+1x= f

1y+ f

1x

��

, với mọi x , y > 0.

Thay x bởi1x

, ta được

1f (y)

+ x = f�

1y+ f (x)

, với mọi x , y > 0.

Giả sử tồn tại α > 0 sao cho f (α)< α. Thay x = α và y =1

α− f (α), ta được

1f (α− f (α))

+α= f (α),

Tập san Toán học STAR EDUCATION

Page 21: lovetoan.files.wordpress.com€¦ · 2 NGUYŽN TĂNG VŨ - NGUYŽN NG¯C DUY - VƯƠNG TRUNG DŨNG LÊ PHÚC LÚ - TRƒN BÁ Đ—T T−P SAN TOÁN H¯C STAR EDUCATION SŁ thø 03

20 TRẦN BÁ ĐẠT

hay

f (α)> α.

Điều này mâu chuẫn với f (α)< α. Do đó f (x)≥ x với mọi x > 0.

b) Khi đó ta có

x +1

f (y)= f

1y+ f (x)

≥1y+ f (x)≥

1y+ x , với mọi x , y > 0.

Do đó

1f (y)

≥1y

, với mọi y > 0,

hay f (x) ≤ x với mọi x > 0.Kết hợp với f (x) ≥ x với mọi x > 0, ta suy raf (x) = x với mọi x > 0. Thử lại, ta nhận f (x) = x là nghiệm duy nhất củaphương trình.

Trong bài toán này, ta đã chứng minh rằng f (x) ≥ x với mọi x > 0 và sử dụng nó đểchứng minh f (x) ≤ x với mọi x > 0, từ đó suy ra f (x) = x với mọi x > 0. Ta có thểáp dụng kĩ thuật này để giải bài toán sau.

Bài toán 4 (Mohammad Jafari). Tìm hàm số f : R+ −→ R+ thỏa mãn

f ( f (x) + 2y) = f (2x + y) + 2y, với mọi x , y > 0.

Lời giải. Đầu tiên, ta chứng minh f (x) ≥ 2x với mọi x > 0 bằng phản chứng. Thậtvậy, nếu tồn tại α > 0 thỏa f (α)< 2α thì thay x = α, y = 2α− f (α) , ta được

f (4α− f (α)) = f (4α− f (α)) + 2(2α− f (α)).

Điều này chứng tỏ f (α) = 2α (vô lí).Như vậy f (x)≥ 2x với mọi x > 0. Áp dụng vào đề bài, ta có

f (2x + y) + 2y = f ( f (x) + 2y)≥ 2 f (x) + 4y,

hay

f (2x + y)≥ 2 f (x) + 2y, với mọi x , y > 0.

Thay y bởi f (y) trong bất đẳng thức trên, ta có

f (2x + f (y))≥ 2 f (x) + 2 f (y)≥ f (x) + 2 f (y) + 2x , với mọi x , y > 0.

Kết hợp với đẳng thức đề cho, ta lại có

f (2y + x) + 2x = f ( f (y) + 2x)≥ f (x) + 2 f (y) + 2x ,

Tập san Toán học STAR EDUCATION

Page 22: lovetoan.files.wordpress.com€¦ · 2 NGUYŽN TĂNG VŨ - NGUYŽN NG¯C DUY - VƯƠNG TRUNG DŨNG LÊ PHÚC LÚ - TRƒN BÁ Đ—T T−P SAN TOÁN H¯C STAR EDUCATION SŁ thø 03

TRẦN BÁ ĐẠT

hay tương đương

f (2y + x)≥ f (x) + 2 f (y), với mọi x , y > 0. (2.1)

Thay x bởi f (x) trong bất đẳng thức trên và áp dụng phương trình đề cho, ta lại có

f (2x + y) + 2y = f ( f (x) + 2y)≥ f ( f (x)) + 2 f (y)≥ f ( f (x)) + f (y) + 2y,

hay

f (2x + y) = f ( f (x)) + f (y), với mọi x , y > 0.

Do đó f là hàm tăng. Theo (2.1), ta có

f (2x + y) + 2y = f ( f (x) + 2y)≥ 2 f (y) + f ( f (x))≥ f (y) + 2y + f (2x),

hay

f (2x + y)≥ f (2x) + f (y), với mọi x , y > 0.

Từ đây ta suy ra f trên cộng tính. Áp dụng vào phương trình đề bài, ta lại có

f (2x + y) + 2y = f ( f (x) + 2y)≥ f ( f (x) + y) + f (y)≥ f ( f (x) + y) + 2y,

hay

f (2x + y)≥ f ( f (x) + y), tương đương f (x)≤ 2x , với mọi x > 0.

Vậy f (x) = 2x với mọi x > 0.

Bài toán sau đây không sử dụng tính trên tuyến tính của hàm số nhưng vẫn mang tư

tưởng tương tự, tức chứng minh lần lượt f (x)≥1x

và f (x)≤1x

.

Bài toán 5 (Tây Ban Nha 2018). Tìm tất cả các hàm số f : R+ −→ R+ thỏa mãn

f (x + f (y)) = y f (x y + 1), với mọi x , y > 0.

Lời giải. a) Giả sử tồn tại α > 1 sao cho f (α) > 1 hoặc tồn tại α < 1 sao cho

f (α) < 1. Thay y = α, x =1− f (α)

1−αvào phương trình của đề bài, ta suy ra

α= 1 (Vô lí).

Vậy f (x)≥ 1 với mọi x < 1 và f (x)≤ 1 với mọi x > 1.

Tập san Toán học STAR EDUCATION

Page 23: lovetoan.files.wordpress.com€¦ · 2 NGUYŽN TĂNG VŨ - NGUYŽN NG¯C DUY - VƯƠNG TRUNG DŨNG LÊ PHÚC LÚ - TRƒN BÁ Đ—T T−P SAN TOÁN H¯C STAR EDUCATION SŁ thø 03

22 TRẦN BÁ ĐẠT

b) Với y > 1, thay x =y − 1

yvào phương trình đề cho, ta có

f�

y − 1y+ f (y)

= y f (y), với mọi y > 1.

Ta chứng minh rằng y f (y) = 1 với mọi y > 0.

– Nếu tồn tại y > 1 thỏa y f (y)> 1, ta suy ra

f�

y − 1y+ f (y)

> 1,

kéo theo

y − 1y+ f (y)< 1.

Điều này tương đương với y f (y)< 1 (vô lí).

– Nếu tồn tại y > 1 thỏa y f (y)< 1, ta suy ra

f�

y − 1y+ f (y)

< 1,

kéo theo

y − 1y+ f (y)> 1.

Điều này tương đương với y f (y)> 1 (vô lí).

Do vậy f (y) =1y

với mọi y > 1. Với y ≤ 1, thay x = 1 vào phương trình đề

cho, ta có

f (1+ f (y)) = y f (y + 1).

Do f (y) + 1 và y + 1 đều lớn hơn 0 nên ta suy ra

11+ f (y)

=y

y + 1.

Điều trên tương đương với f (y) =1y

. Vậy f (y) =1y

với mọi y > 0. Thử lại, ta

nhận hàm này là nghiệm duy nhất của phương trình.

3.2. Thêm biến

Phương pháp thêm biến đặc biệt hữu hiệu trong các bài toán hàm số trên R+. Tấtnhiên, để hoàn thiện một bài toán, ta thường dùng phương pháp thêm biến kết hợpvới một số phương pháp đi kèm như đối xứng, hoặc tính bằng hai cách. Đầu tiên, tatrình bày lại lời giải cho bài toán mở đầu bằng phương pháp thêm biến và tính bằnghai cách.

Tập san Toán học STAR EDUCATION

Page 24: lovetoan.files.wordpress.com€¦ · 2 NGUYŽN TĂNG VŨ - NGUYŽN NG¯C DUY - VƯƠNG TRUNG DŨNG LÊ PHÚC LÚ - TRƒN BÁ Đ—T T−P SAN TOÁN H¯C STAR EDUCATION SŁ thø 03

TRẦN BÁ ĐẠT

Bài toán 6 (Gặp gỡ Toán học 2019). Tìm tất cả các hàm f : R+ −→ R+ thỏa mãn

f (x + f (y)) = 2y + f (x), với mọi x , y ∈ R+.

Lời giải. Ta dễ dàng chứng minh được f đơn ánh dựa vào định nghĩa. Trong phươngtrình đề cho, thay y bởi y + z, ta có

f (x + f (y + z)) = 2(y + z) + f (x)= f (x + f (y)) + 2z= f (x + f (y) + f (z)), với mọi x , y, z > 0.

Theo tính đơn ánh, ta suy ra

f (x + y) = f (x) + f (y), với mọi x , y > 0.

Do đó f (x) = ax với mọi x > 0. Thử lại, ta nhận f (x) =p

2x làm nghiệm duy nhấtcủa phương trình.

Bài toán sau đây cũng có thể xử lí thêm biến bằng cách thay y bởi y + f (z).

Bài toán 7 (IMO Shortlist 2007). Tìm hàm số f : R+ −→ R+ thỏa mãn

f (x + f (y)) = f (x + y) + f (y), với mọi x , y > 0.

Lời giải. Trong phương trình đề bài, thay y bởi y + f (z), vế trái trở thành

f (x + f (y + f (z)) = f (x + f (y + z) + f (z))= f (x + f (y + z) + z) + f (z)= f (x + y + 2z) + f (y + z) + f (z), với mọi x , y, z,> 0.

Vế phải của biểu thức ban đầu cũng trở thành

f (x + y + f (z)) + f (y + f (z)) = f (x + y + z) + f (z) + f (y + z) + f (z), với mọi x , y, z > 0.

Vì vế trái bằng vế phải, so sánh hai đẳng thức vừa có được, ta suy ra

f (x + y + 2z) = f (x + y + z) + f (z), với mọi x , y, z > 0

hay

f (x + y) = f (x) + f (y), với mọi x > y > 0.

Với x , y > 0 bất kì. Lấy z > x + y, ta có

f (x + y + z) = f (x + y) + f (z), với mọi z > x , y > 0.

Mặt khác

f (x + y + z) = f (x) + f (y + z) = f (x) + f (y) + f (z), với mọi z > x , y > 0.

Do đó

f (x + y) = f (x) + f (y), với mọi x , y > 0

Điều này chứng tỏ f cộng tính trên R+ nên f (x) = ax với mọi x > 0. Thay lại vào đềbài, ta nhận f (x) = 2x làm nghiệm duy nhất của bài toán.

Tập san Toán học STAR EDUCATION

Page 25: lovetoan.files.wordpress.com€¦ · 2 NGUYŽN TĂNG VŨ - NGUYŽN NG¯C DUY - VƯƠNG TRUNG DŨNG LÊ PHÚC LÚ - TRƒN BÁ Đ—T T−P SAN TOÁN H¯C STAR EDUCATION SŁ thø 03

24 TRẦN BÁ ĐẠT

Ngoài cách xử lí trên, ta còn có một hướng xử lí khác cho bài toán này bằng việc quansát thấy sự độc lập giữa các biểu thức f (y), y đối với x .

Lời giải. Hiển nhiên f (x) ≥ x với mọi x > 0. Thay x bởi x − y vào phương trình đềbài, ta có

f (x + f (y)− y)) = f (x) + f (y), với mọi x > y > 0.

Thay y bởi y + f (z)− z vào đề bài, ta sẽ có

f (x + f (y + f (z)− z)) = f (x + y + f (z)− z) + f (y + f (z)− z), với mọi x > 0, y > z > 0.

Áp dụng đề bài vào cả hai vế, ta sẽ thu được

f (x + f (y) + f (z)) = f (x + y) + f (z) + f (y) + f (z)= f (x + y) + f (y) + 2 f (z), với mọi x > 0, y > z > 0.

Mặt khác, thay x bởi x + f (z) vào đề bài, ta cũng có

f (x + f (z) + f (y)) = f (x + f (z) + y) + f (y), với mọi x > 0, y > z > 0.

So sánh hai đẳng thức vừa có, ta suy ra

f (x + y + f (z)) = f (x + y) + 2 f (z), với mọi x > 0, y > z > 0.

Mà f (x+ y)+2 f (z) = f (x+ y+ f (z)−z)+ f (z) = f (x+ y+2( f (z)−z)), với mọi x >0, y > z > 0 nên ta suy ra

f (x + y + 2( f (z)− z)) = f (x + y + f (z)), với mọi x > 0, y > z > 0.

Mà f trên tuyến tính nên không tuần hoàn, kéo theo

2( f (z)− z) = f (z),

hay f (z) = 2z với mọi z > 0. Thử lại, ta nhận đây là nghiệm hàm duy nhất của bàitoán.

Bài toán sau đây cũng giải được bằng cách thêm biến.

Bài toán 8 (Hà Nam 2019). Tìm tất cả các hàm số f : R+ −→ R+ thỏa mãn

f (y) f�

1y+ f

1x

��

= 1+f (y)

x, với mọi x , y > 0.

Lời giải. Thay x bởi1x

, bài toán được viết lại thành

f�

1y+ f (x)

= x +1

f (y), với mọi x , y > 0.

Tập san Toán học STAR EDUCATION

Page 26: lovetoan.files.wordpress.com€¦ · 2 NGUYŽN TĂNG VŨ - NGUYŽN NG¯C DUY - VƯƠNG TRUNG DŨNG LÊ PHÚC LÚ - TRƒN BÁ Đ—T T−P SAN TOÁN H¯C STAR EDUCATION SŁ thø 03

TRẦN BÁ ĐẠT

Từ đây ta suy ra f đơn ánh, f (x)≥ x với mọi x > 0. Thay x bởi1x+ f

1z

, ta có

f�

1y+

1x+ f

1z

��

=1x+ f

1z

+1

f (y),

hay

f�

1y+

1z+

1f (x)

=1x+ f

1z

+1

f (y), với mọi x , y, z > 0.

Đổi chỗ y và z, ta suy ra

f�

1z

+1

f (y)= f

1y

+1

f (z),

tức

f�

1y

=1

f (y)+ c, với mọi y > 0.

Ta có

f (x) =1

f�

1x

� + c =1

1f (x)

+ c+ c,

hay

c([ f (x)]2 − c f (x)− 1) = 0, với mọi x > 0.

Vì f không bị chặn trên nên tồn tại x0 sao cho [ f (x0)]2− c f (x0)−1> 0. Từ đó ta suyra c = 0. Do vậy

f�

1x

=1

f (x), với mọi x , y > 0.

Khi đó bài toán trở thành

f (y + f (x)) = x + f (y), với mọi x , y > 0.

Tiếp tục thay y bởi y + f (z), ta suy ra

f (y + f (z) + f (x)) = x + f (y + f (z)) = x + z + f (y) = f (y + f (x + z)), với mọi x , y, z > 0.

Sử dụng tính đơn ánh, ta suy ra

f (x) + f (z) = f (x + z), với mọi x , z > 0.

Từ đây suy ra f cộng tính trên R+. Kéo theo f (x) = ax với mọi x > 0. Thay vào đềbài, ta suy ra f (x) = x là nghiệm duy nhất của phương trình.

Ta nhận thấy rằng bài toán này chính là Bài toán 3. Tuy nhiên, việc nhìn bài toán ấytừ các góc độ khác nhau đã đưa ra cho ta lời giải mang thêm những kĩ thuật mà có thểsử dụng được cho những bài toán khác. Bài toán dưới đây là một biến thể khác củahai bài toán được đề cập ở trên. Tất nhiên, cách giải dưới đây cũng hoàn toàn khác vàyêu cầu sử dụng tính chất hàm cộng tính, bị chặn dưới trên một khoảng mở thì tuyếntính.

Tập san Toán học STAR EDUCATION

Page 27: lovetoan.files.wordpress.com€¦ · 2 NGUYŽN TĂNG VŨ - NGUYŽN NG¯C DUY - VƯƠNG TRUNG DŨNG LÊ PHÚC LÚ - TRƒN BÁ Đ—T T−P SAN TOÁN H¯C STAR EDUCATION SŁ thø 03

26 TRẦN BÁ ĐẠT

Bài toán 9 (Áo 2018). Cho α là số thực khác 0. Tìm tất cả các hàm f : R+ −→ R+thỏa mãn

f ( f (x) + y) = αx +1

f�

1y

� , với mọi x , y > 0.

Lời giải. Ta dễ dàng chứng minh được f đơn ánh. Thay y bởi f (y), ta có

f ( f (x) + f (y)) = αx +1

f�

1f (y)

� , với mọi x , y > 0. (2.2)

Thay đổi vai trò của x và y , ta suy ra

αx +1

f�

1f (y)

� = αy +1

f�

1f (x)

� ,

hay

1

f�

1f (x)

� −αx =1

f�

1f (y)

� −αy = c, với mọi x , y > 0.

Thay1

f�

1f (x)

� = αx + c vào (2.2), ta có

f ( f (x) + f (y)) = αx +αy + 2c.

Khi đó với mọi x , y, z, t thỏa x + y = z + t thì ta có

f ( f (x) + f (y)) = f ( f (z) + f (t)).

Theo tính đơn ánh ta suy ra f (x)+ f (y) = f (z)+ f (t) với mọi x + y = z+ t. Đặc biệt

f (x + 1) + f (y + 1) = f (x + y + 1) + f (1), với mọi x > 0.

Đặt g(x) = f (x+1)− f (1) thì g bị chặn dưới, cộng tính trên (0,+∞) nên g(x) = ax ,suy ra f (x +1) = ax + b. Do đó f (x) = cx + d với mọi x > 1. Thay vào đề bài, ta suyra d = 0, c = α = 1. Điều này chứng tỏ f (x) = x với mọi x > 1. Mặt khác, với x ≤ 1,ta có

f (x) + f (3) = f (x + 1) + f (2),

suy ra

f (x) = x , với mọi x > 0.

Thử lại, ta nhận f (x) = x là nghiệm hàm duy nhất của phương trình.

Trong [2], tác giả cũng trình bày một số bài toán có cách giải tương tự.

Tập san Toán học STAR EDUCATION

Page 28: lovetoan.files.wordpress.com€¦ · 2 NGUYŽN TĂNG VŨ - NGUYŽN NG¯C DUY - VƯƠNG TRUNG DŨNG LÊ PHÚC LÚ - TRƒN BÁ Đ—T T−P SAN TOÁN H¯C STAR EDUCATION SŁ thø 03

TRẦN BÁ ĐẠT

Bài toán 10. Tìm tất cả các hàm f : R+ −→ R+ thỏa mãn

f (x + 3 f (y)) = f (x) + f (y) + 2y, với mọi x , y > 0.

Lời giải. Thay y bởi y + 3 f (z), ta được

V T = f (x + 3 f (y + 3 f (z))) = f (x + 3( f (y) + f (z) + 2z))= f (x + 6z + 3 f (y) + 3 f (z))= f (x + 6z + 3 f (y)) + f (z) + 2z= f (x + 6z) + f (y) + 2y + f (z) + 2z, với mọi x , y, z > 0.

Mặt khác,

V P = f (x) + f (y + 3 f (z)) + 2y + 6 f (z)= f (x) + f (y) + f (z) + 2z + 2y + 6 f (z), với mọi x , y, z > 0.

Do đó

f (x + 6z) = f (x) + 6 f (z), với mọi x , z > 0.

Thay x bởi 6x rồi hoán đổi vai trò của x , z, ta suy ra

6 f (z) = f (6z) + c, với mọi z > 0.

Từ đó suy ra

f (x + 6z) = f (x) + f (6z) + c, với mọi x , z > 0.

Điều này chứng tỏ g(x) = f (x)+ c là hàm cộng tính trên R+ nên tuyến tính. Thay lạivào đề bài, ta suy ra f (x) = x với mọi x > 0.

Bài toán 11. Tìm tất cả các hàm f : R+ −→ R+ thỏa mãn

f (2x + 2 f (y)) = x + f (x) + 2y, với mọi x , y > 0.

Lời giải. Thay y bởi 2y + 2 f (z), ta có

V T = f (2x + 2 f (2y + 2 f (z))) = f (2x + 2(y + f (y) + 2z))= f (2x + 2y + 4z + 2 f (y))= x + y + 2z + f (x + y + 2z) + 2y, với mọi x , y, z > 0.

Mặt khác,

V P = x + f (x) + 2(2y + 2 f (z)) = x + 4y + f (x) + 4 f (z), với mọi x , y, z > 0.

Từ đó suy ra

f (x + y + 2z) + 2z = y + f (x) + 4 f (z), với mọi x , y, z > 0.

Đổi vài trò của x , y , ta suy ra

f (x) + y = f (y) + x , với mọi x , y > 0.

Do đó f (x) = x + c, với mọi x > 0. Thay lại vào đề bài, ta suy ra f (x) = x là nghiệmduy nhất của bài toán.

Tập san Toán học STAR EDUCATION

Page 29: lovetoan.files.wordpress.com€¦ · 2 NGUYŽN TĂNG VŨ - NGUYŽN NG¯C DUY - VƯƠNG TRUNG DŨNG LÊ PHÚC LÚ - TRƒN BÁ Đ—T T−P SAN TOÁN H¯C STAR EDUCATION SŁ thø 03

28 TRẦN BÁ ĐẠT

Tóm lại, khi đề bài có dạng f (x + g(y)) thì ta có thể thêm biến bằng cách thay y bởiy + g(z) và biến đổi hai vế rồi so sánh.Bên cạnh đó, hai bài toán sau trong [2] và [3] sử dụng tính đối xứng của các biến.Đặc biệt ở bài toán thứ hai, ta cần dùng thêm tính chất hàm trên tuyến tính thì khôngtuần hoàn để chứng minh đơn ánh.

Bài toán 12. Tìm tất cả các hàm f : R+ −→ R+ thỏa mãn

f (x f (y))− x = f (x y), với mọi x ,> 0.

Lời giải. Thay x bởi f (x), ta có

f ( f (x) f (y)) = f (x) + f ( f (x)y)= f (x) + y + f (x y), với mọi x ,> 0.

Hoán đổi vai trò của x , y ta suy ra

f (x) + y + f (x y) = f (y) + x + f (x y), với mọi x , y > 0.

Do đó f (x)− x = f (y)− y = c, hay f (x) = x + c với mọi x > 0. Thay vào đề bài, tasuy ra f (x) = x + 1 là nghiệm duy nhất của bài toán.

Bài toán 13 (Mohammad Jafari). Tìm tất cả các hàm số f : R+ −→ R+ thỏa mãn

f (x + f (x) + y) = f (2x) + f (y), với mọi x , y > 0.

Lời giải. Đầu tiên, ta chứng minh f đơn ánh. Lấy a, b thỏa mãn f (a) = f (b). Thaylần lượt x = a, y = b và x = b, y = a vào phương trình đề cho, ta có

f (a+ f (a) + b) = f (2a) + f (b), f (b+ f (b) + a) = f (2b) + f (a).

Do đó f (2a) = f (2b). Lần lượt thay x = a, x = b vào phương trình đề cho, ta lại có

f (2a) + f (y) = f (a+ f (a) + y) = f (b+ f (b) + y), với mọi y > 0.

Nếu a 6= b thì f tuần hoàn với chu kì T = |a − b|, khi x đủ lớn. Điều này mâu thuẫnvới f (x)≥ x . Vậy f đơn ánh.Thay y bởi 2y vào phương trình đề cho và sử dụng tính đối xứng của hai vế, ta suy ra

f (x + f (x) + 2y) = f (y + f (y) + 2x),

hay

f (x)− x = f (y)− y = c, với mọi x , y > 0,

Thử lại f (x) = x+c vào đề bài, thấy thỏa. Vậy phương trình có nghiệm là f (x) = x+c,với c > 0.

Cuối cùng là bài toán vừa được sử dụng làm đề thi Trường Đông Miền Nam 2019, yêucầu dùng tính đối xứng và toàn ánh trên một khoảng mở.

Tập san Toán học STAR EDUCATION

Page 30: lovetoan.files.wordpress.com€¦ · 2 NGUYŽN TĂNG VŨ - NGUYŽN NG¯C DUY - VƯƠNG TRUNG DŨNG LÊ PHÚC LÚ - TRƒN BÁ Đ—T T−P SAN TOÁN H¯C STAR EDUCATION SŁ thø 03

TRẦN BÁ ĐẠT

Bài toán 14 (Trường Đông Miền Nam 2019). Tìm tất cả các hàm f : R+ −→ R+thỏa mãn

f (x f (y) + f (x)) = 2 f (x) + x y, với mọi x , y > 0.

Lời giải. Đặt a = f (1). Trong phương trình đề cho, thay x = 1, ta được

f ( f (y) + a) = 2a+ y, với mọi y > 0.

Điều này chứng tỏ f toàn ánh trên (2a,+∞). Tiếp tục thay y bởi f (y)+a vào phươngtrình trên, ta suy ra

f ( f ( f (y) + a) + a) = 2a+ f (y) + a,

hay

f (y + 3a) = y + 3a, với mọi y > 0.

Bằng quy nạp, ta chứng minh được

f (y + 3na) = y + 3na, với mọi y > 0, n ∈ N.

Trong phương trình đề cho, thay y bởi y + 3a và kết hợp với phương trình đó, ta có

f (x f (y) + f (x) + 3ax) = 2 f (x) + x y + 3ax , với mọi x , y > 0.

Với mỗi x , z > 0, tồn tại n ∈ N đủ lớn và y > 0 thỏa x f (y)+ f (x) = z+3na, ta suy ra

f (z + x) = f (z) + x = f (x) + z, với mọi x , z > 0.

Do đó f (x) = x + c. Thử lại ta suy ra f (x) = x + 1 là nghiệm duy nhất.

Ngoài ra, Bài toán 11 cũng có thể được xử lí tương tự như bài toán trên. Phần xử línày xin dành cho bạn đọc.

4. Một số bài toán áp dụng

Cuối cùng là một số bài tập áp dụng các kĩ thuật vừa nêu ra trong phần trước.

Bài toán 15. Tìm tất cả các hàm f : R+ −→ R+ thỏa mãn

f (x + y)2 = f (x)2 + 2 f (x y) + f (y)2, với mọi x , y > 0.

Bài toán 16 (Mohammad Jafari). Tìm hàm số f : R+ −→ R+ thỏa mãn

f ( f (x) + 2y) = f (2x + y) + 2y, với mọi x , y > 0.

Tập san Toán học STAR EDUCATION

Page 31: lovetoan.files.wordpress.com€¦ · 2 NGUYŽN TĂNG VŨ - NGUYŽN NG¯C DUY - VƯƠNG TRUNG DŨNG LÊ PHÚC LÚ - TRƒN BÁ Đ—T T−P SAN TOÁN H¯C STAR EDUCATION SŁ thø 03

30 TRẦN BÁ ĐẠT

Bài toán 17 (Mohammad Jafari). Cho hai hàm số f , g : R+ −→ R+ thỏa mãn

f (g(x) + y) = f (x) + g(y), với mọi x , y ∈ R+,

g( f (x) + y) = g(x) + f (y), với mọi x , y ∈ R+,

f (1) = g(1).

Chứng minh rằng f (x) = g(x) với mọi x > 0.

Bài toán 18. Tìm tất cả các hàm f : R+ −→ R+ thỏa mãn

f (x + f (y)) = 3 f (x)− 2x + f (y), với mọi x , y > 0.

Bài toán 19. Tìm tất cả các hàm f : R+ −→ R+ thỏa mãn

f (x + f (x) + y) = f (y) + 2x , với mọi x , y > 0.

Bài toán 20. Tìm tất cả các hàm f : R+ −→ R+ thỏa mãn

f (x + f (x) + 2y) = f (2x) + y + f (y), với mọi x , y > 0.

Bài toán 21. Tìm tất cả các hàm f : R+ −→ R+ thỏa mãn

f (x + f (y)) + x = f (x + f (x)) + y, với mọi x , y > 0.

Bài toán 22 (IMO Shortlist 2005). Tìm tất cả các hàm f : R+ −→ R+ thỏa mãn

f (x) f (y) = 2 f (x + y f (x)), với mọi x , y > 0.

Bài toán 23 (Thái Lan 2019). Tìm tất cả các hàm f : R+ −→ R+ thỏa mãn

f (x + y f (x) + y2) = f (x) + 2y, với mọi x , y > 0.

Bài toán 24 (Nhật Bản 2019). Tìm tất cả các hàm liên tục f : R+ −→ R+ thỏa

f�

f (y)f (x)

+ 1�

= f�

x +yx+ 1

− f (x), với mọi x , y > 0.

Tập san Toán học STAR EDUCATION

Page 32: lovetoan.files.wordpress.com€¦ · 2 NGUYŽN TĂNG VŨ - NGUYŽN NG¯C DUY - VƯƠNG TRUNG DŨNG LÊ PHÚC LÚ - TRƒN BÁ Đ—T T−P SAN TOÁN H¯C STAR EDUCATION SŁ thø 03

TRẦN BÁ ĐẠT

Bài toán 25. Tìm tất cả các hàm số f : R+ −→ R+ thỏa mãn

f (x + f (x + y)) = f (2x) + f (y), với mọi x , y > 0.

Bài toán 26. Tìm tất cả các hàm f : R+ −→ R+ thỏa mãn

f (x + f (x) + 2y) = f (2x) + 2 f (y), với mọi x , y > 0.

Tập san Toán học STAR EDUCATION

Page 33: lovetoan.files.wordpress.com€¦ · 2 NGUYŽN TĂNG VŨ - NGUYŽN NG¯C DUY - VƯƠNG TRUNG DŨNG LÊ PHÚC LÚ - TRƒN BÁ Đ—T T−P SAN TOÁN H¯C STAR EDUCATION SŁ thø 03

Tài liệu tham khảo

[1] T. Andreescu, I. Boreico: Functional Equations. Electronic Edition (2007)

[2] P-C. Wu, : Functional Equation, Lecture Notes (2019)

[3] M. Jafari: 220 Problems Proposed by Mohammad Jafari.

[4] M. Jafari: 116 Algebra Problems Proposed by Mohammad Jafari.

32

Page 34: lovetoan.files.wordpress.com€¦ · 2 NGUYŽN TĂNG VŨ - NGUYŽN NG¯C DUY - VƯƠNG TRUNG DŨNG LÊ PHÚC LÚ - TRƒN BÁ Đ—T T−P SAN TOÁN H¯C STAR EDUCATION SŁ thø 03

Định lý Wolstenholme và ứng dụng

Kiều Đình Minh(GV Trường THPT CHuyên Hùng Vương, Phú Thọ)

Lê Phúc Lữ(Cao học ĐH KHTN TP Hồ Chí Minh)

1. Kiến thức cần nhớ

Định lý 1. Cho p là số nguyên tố, khi đó, ta có 2 định lý quan trọng:(1) Wilson: (p− 1)!+ 1≡ 0 (mod p).(2) Fermat nhỏ: ap−1 ≡ 1 (mod p) với (a, p) = 1.

Các định lý này có thể chứng minh dựa vào tính chất của hệ thặng dư đầy đủ (TDĐĐ),thu gọn và nghịch đảo của một số nguyên theo modulo p, cụ thể là:

Định lý 2. Với mọi số nguyên i ∈ {1,2, 3, ..., p− 1} thì tồn tại duy nhất số j ∈{1,2, 3, ..., p− 1} sao cho i j ≡ 1 (mod p).

(nếu (a, m) = 1 thì ax + y sẽ chạy qua 1 hệ TDĐĐ của m khi x chạy qua 1 hệ TDĐĐcủa m). Đặc biệt: i = 1⇒ j = 1 và i = p− 1⇒ j = p− 1.

Định lý 3. Định lý Viète: Cho đa thức P(x) = an xn+ an−1 xn−1+ ...+ a2 x2+ a1 x + a0

và giả sử đa thức này có n nghiệm là r1, r2, ..., rn khác 0. Khi đó, ta có:

r1 + r2 + ...+ rn = −an−1

an, r1r2...rn =

(−1)na0

an,

(−1)n−1a1

an=

n∑

i=1

j 6=i

r j = r1r2...rn

1r1+

1r2+ ...+

1rn

Ta đã biết rằng một đa thức P(x) bậc n thì có tối đa n nghiệm. Điều này vẫn đúngtrong phương trình đồng dư, tức là P(x) ≡ 0 (mod m) với P(x) có bậc m cũng có tốiđa m nghiệm (không có cặp nghiệm nào đồng dư với nhau theo mod m). Chẳng hạnnhư: P(x) = x3 + x + 1≡ 0 (mod 3) có nghiệm là x ≡ 1 (mod 3).

Định nghĩa 1. Đồng dư thức của phân số: Ta quy ước ký hiệu ab ≡

cd (mod m), trong

đó a, b, c, d ∈ Z và b, d 6= 0, (b, m) = (d, m) = 1 khi ad − bc ≡ 0 (mod m). Chẳnghạn, thay vì viết i j ≡ 1 (mod p) như ở trên, ta có thể viết 1

i ≡ j (mod p).

33

Page 35: lovetoan.files.wordpress.com€¦ · 2 NGUYŽN TĂNG VŨ - NGUYŽN NG¯C DUY - VƯƠNG TRUNG DŨNG LÊ PHÚC LÚ - TRƒN BÁ Đ—T T−P SAN TOÁN H¯C STAR EDUCATION SŁ thø 03

34 KIỀU ĐÌNH MINH - LÊ PHÚC LỮ

Định lý 4. Đa thức Lagrange. Xét đa thức f (x) = (x +1)(x +2)...(x + p−1) với p làsố nguyên tố. Khai triển ra, ta có: f (x) = x p−1+ap−2 x p−2+ ...+a2 x2+a1 x +(p−1)!Định lý Lagrange về đa thức này nói rằng các hệ số ai với i = 1, 2, ..., p − 2 đều chiahết cho p.

Cách 1. Thật vậy, Xét đa thức P(x) = f (x) −�

x p−1 − 1�

, có bậc là p − 2. Hơn nữaP(1)≡ P(2)≡ ...≡ P(p− 1)≡ 0 (mod p) nên đa thức này có p− 1 nghiệm, tức là tấtcả các hệ số của đa thức này phải chia hết cho p. Suy ra ap−2, ap−3, ..., a2, a1, (p−1)!+1đều phải chia hết cho p. Định lý được chứng minh.

Cách 2. Nếu p là một số nguyên tố và 0 < i < p thì C ip chia hết cho p. Thật vậy,

ta có C ip =

pi C i−1

p−1. Nhưng (p, i) = 1,∀i, 0 < i < p và C ip là một số nguyên nên

suy ra C ip chia hết cho p. Ta có f (x + 1) = (x + 2) ... (x + p), suy ra (x + p) f (x) =

(x + 1) f (x + 1)⇒ p f (x) = (x + 1) f (x + 1)− x f (x) . Do đó

px p−1 + pa1 x p−2 + ...+ pap−2 x + p!=

= (x + 1)p − x p + a1

(x + 1)p−1 − x p−1�

+ ...+ ap−2

(x + 1)2 − x2�

+ (x + 1− x) (p− 1)!

Đồng nhất các hệ số của x p−2, x p−3, ..., x , ta được

pa1 = C2p + C1

p−1a1

pa2 = C3p + C2

p−1a1 + C1p−2a2

....

pap−2 = C p−1p + C p−2

p−1 a1 + ...+ C12 ap−2

Áp dụng tính chất đã nêu trên, từ đẳng thức thứ nhất có p|a1. Tương tự p|a1, a2, ..., ap−2.

2. Định lý Wolstenholme

Định lý 5. Dạng 1. Cho p là số nguyên tố lớn hơn 3, khi đó ta có:

112+

122+

132+ ...+

1

(p− 1)2≡ 0(modp).

Chứng minh. Sử dụng nghịch đảo của một số theo modulo p thì tổng trên cũng làtổng của một hoán vị của các số

12, 22, ..., (p− 1)2

và ta có

112+

122+

132+ ...+

1

(p− 1)2≡ 12+22+ ...+(p− 1)2 =

p(p− 1)(2p− 1)6

≡ 0 (mod p).

Tập san Toán học STAR EDUCATION

Page 36: lovetoan.files.wordpress.com€¦ · 2 NGUYŽN TĂNG VŨ - NGUYŽN NG¯C DUY - VƯƠNG TRUNG DŨNG LÊ PHÚC LÚ - TRƒN BÁ Đ—T T−P SAN TOÁN H¯C STAR EDUCATION SŁ thø 03

KIỀU ĐÌNH MINH - LÊ PHÚC LỮ

Định lý 6. Dạng 2. Cho p là số nguyên tố lớn hơn 3, khi đó ta có:

11+

12+ ...+

1p− 1

≡ 0 (mod p2))

Chứng minh. Dễ thấy rằng tổng trên đồng dư với 0 theo modulo p, định lý này đòihỏi một kết quả mạnh hơn. Chú ý rằng

11+

12+ ...+

1p− 1

≡ 1 · 2 · 3...(p− 1)�

11+

12+ ...+

1p− 1

(modp2)

nên sử dụng lại đa thức Lagrange và định lý Viète, ta cần chứng minh a1 ≡ 0(mod p2).Thật vậy, từ đồng nhất thức:

(x + 1)(x + 2)...(x + p− 1) = f (x) = x p−1 + ap−2 x p−2 + ...+ a2 x2 + a1 x + (p− 1)!,

ta cho x = −p thì vế trái chính là (−1)(−2)...(−p + 1) = (−1)p−1(p − 1)! = (p − 1)!.Do đó: (−p)p−1 + ap−2(−p)p−2 + ...+ a2(−p)2 = pa1. Dễ thấy vế trái chia hết cho p3

nên vế phải cũng phải chia hết cho p3 hay p2�

� a1.

Định lý 7. Dạng 3. Cho p là số nguyên tố lớn hơn 3, chứng minh rằng

C p2p ≡ 2 (mod p3) hay C p−1

2p−1 ≡ 1 (mod p3).

Chứng minh. Hai kết quả trên là tương đương vì C p2p =

(2p)!p!p! = 2 (2p−1)!

p!(p−1)! = 2C p−12p−1.

Nếu áp dụng đồng nhất thức Vandermonde, ta chỉ chứng minh được rằng C p2p ≡

2(modp2). Thật vậy, ta đã biết rằng:

C p2p = (C

0p )

2 + (C1p )

2 + ...+ (C pp )

2.

Hai số hạng đầu và cuối bằng 1, còn các số hạng ở giữa đều chia hết cho p2 vìC i

p =p!

i!(p−i)! chia hết cho p với mọi i = 1, 2,3, ..., p− 1. Để đưa từ modulo p2 sang p3,ta phải có một cách hiệu quả hơn.

C p2p−1 − 1=

(2p− 1)!p!(p− 1)!

− 1=(p+ 1)(p+ 2)...(p+ p− 1)

(p− 1)!− 1

=(p+ 1)(p+ 2)...(p+ p− 1)− (p− 1)!

(p− 1)!

Do�

p3, (p− 1)!�

= 1 nên ta chỉ cần chứng minh (p + 1)(p + 2)...(p + p − 1) ≡ (p −1)!(modp3) hay f (p) ≡ (p − 1)!(modp3). Tuy nhiên, f (p) − (p − 1)! = ap−2pp−2 +ap−3pp−3 + ...+ a2p2 + a1p chia hết cho p3 nên định lý được chứng minh.

3. Các ví dụ áp dụng

Tập san Toán học STAR EDUCATION

Page 37: lovetoan.files.wordpress.com€¦ · 2 NGUYŽN TĂNG VŨ - NGUYŽN NG¯C DUY - VƯƠNG TRUNG DŨNG LÊ PHÚC LÚ - TRƒN BÁ Đ—T T−P SAN TOÁN H¯C STAR EDUCATION SŁ thø 03

36 KIỀU ĐÌNH MINH - LÊ PHÚC LỮ

Bài toán 1. Cho p > 3 là số nguyên tố lẻ.

1. Chứng minh rằng: 1− 122 + 1

32 − ...− 1(p−1)2

≡ 0 (mod p).

2. 2) Đặt 11+

12+ ...+ 1

p =mn ; m, n ∈ Z và (m, n) = 1. Chứng minh rằng p4|mp−n.

Lời giải. 1) Ta chỉ cần ghép các số hạng thích hợp: 1i2 − 1

(p−i)2= p2−2pi

i2(p−i)2chia hết cho p

với 1≤ i ≤ p− 1.2) Ta có

11+

12+ ...+

1p− 1

=mn−

1p=

mp− nnp

.

Áp dụng định lý Wolstenholme 2 cho vế trái thì mp−nnp ≡ 0(modp2). Vì n chia hết cho

p nên np chia hết cho p2, suy ra mp− n chia hết cho p4.

Bài toán 2. (Mở rộng định lý Wolstenholme 3) Cho p > 3 là số nguyên tố và m, k ∈Z+. Chứng minh C p−1

mpk−1≡ 1 (modpk+2).

Lời giải. Biến đổi trực tiếp, ta có đẳng thức f (−x) = f (x − p). Ta có:

C p−1mpk−1

− 1=(mpk − 1)!

(p− 1)!(mpk − p)!− 1=

(mpk − p+ 1)(mpk − p+ 2)...(mpk − p+ p− 1)(p− 1)!

− 1

=f (mpk − p)(p− 1)!

− 1=f (−mpk)(p− 1)!

− 1=f (−mpk)− (p− 1)!

(p− 1)!

Biểu thức trên tử khi khai triển ra có dạng (−mpk)p−1+ap−2(−mpk)p−2+...+a2(−mpk)2+a1(−mpk). Vì p2

� a2 nên dễ thấy rằng biểu thức trên chia hết cho pk+2.Ta thấy rằng định lý đã nêu rất mạnh nhưng cách chứng minh, biến đổi cũng tươngtự như định lý ban đầu.

Bài toán 3. (APMO 2006) Cho p > 3 là số nguyên tố và r là số cách đặt p quân cờlên bàn cờ hình vuông có kích thước p× p sao cho không có 2 quân cờ nào cùng hàng(nhưng có thể cùng cột). Chứng minh rằng p5|r.

Lời giải. Số cách chọn p quân cờ tùy ý là C pp2 . Số cách chọn thỏa mãn đề bài là (trừ đi

p cách không thỏa mãn) là: C pp2 − p. Ta cần chứng minh C p

p2 − p ≡ 0(modp5), mà

C pp2 =

(p2)!p!(p2 − p)!

=p2 · (p2 − 1)!

p · (p− 1)!(p2 − p)!= pC p−1

p2−1

nên ta đưa về C p−1p2−1 ≡ 1(modp4). Áp dụng định lý Wolstenholme mở rộng, ta có

đpcm.

Tập san Toán học STAR EDUCATION

Page 38: lovetoan.files.wordpress.com€¦ · 2 NGUYŽN TĂNG VŨ - NGUYŽN NG¯C DUY - VƯƠNG TRUNG DŨNG LÊ PHÚC LÚ - TRƒN BÁ Đ—T T−P SAN TOÁN H¯C STAR EDUCATION SŁ thø 03

KIỀU ĐÌNH MINH - LÊ PHÚC LỮ

Bài toán 4. (Putnam 1980) Cho p là số nguyên tố, chứng minh rằng

p∑

i=0

C ipC i

p+i ≡ 2p + 1 (modp2).

Lời giải. Với i = 1,2, 3, ..., p− 1 thì C ip chia hết cho p và

C ip+i − 1=

(p+ i)!i!p!

− 1=(p+ 1)(p+ 2)...(p+ i)

i!− 1=

(p+ 1)(p+ 2)...(p+ i)− i!i!

Dễ thấy tử số của biểu thức cuối chia hết cho p nên C ip+i ≡ 1(modp2). Với i = 0 thì

C0p C0

p = 1 và với i = p thì C pp C p

2p ≡ 2(modp2) theo định lý Wolstenholme 3.

Nhận xét. Nếu x ≡ 0, y ≡ 1 (mod p) thì x y − x = x(y − 1) chia hết cho p2 nênx y ≡ x(modp2). Từ đó suy ra

p∑

i=0

C ipC i

p+i ≡ 1+p−1∑

i=1

C ipC i

p+i + 2≡ 1+p−1∑

i=1

C ip + 2=

p∑

i=0

C ip + 1= 2p + 1(modp2).

Bài toán 5. (KHTN 2010) Cho dãy số (un) xác định bởi công thức:

u1 = 0,

un+1 =(4n+ 2)n3

(n+ 1)4un +

3n+ 1

(n+ 1)4, n≥ 1

Chứng minh rằng dãy số này chứa vô số số hạng nguyên dương.

Lời giải. Ta có: (n+ 1)4un+1 = (4n+ 2)n3un + 3n+ 1, đặt vn = n3un thì

(n+ 1)vn+1 = (4n+ 2)vn + 3n+ 1⇔ (n+ 1)(vn+1 + 1) = (4n+ 2)(vn + 1).

Lại đặt wn = vn + 1 thì wn+1 =4n+2n+1 vn. Ta cũng tính được v1 = 0, w1 = 1 nên

wn =4n− 2

nwn−1 =

4n− 2n·

4n− 6n− 1

wn−2 = ...=2n−1(2n− 1)!!

n!w1 =

2n−1(2n− 1)!!n!

,

trong đó (2n+ 1)!! là tích các số lẻ từ 1 đến 2n+ 1, (2n)! là tích các số chẵn từ 2 đến2n. Ta biến đổi tiếp:

2n−1(2n− 1)!!n!

=2n−1(n− 1)!(2n− 1)!!

n!(n− 1)!=(2n− 2)!!(2n− 1)!!

n!(n− 1)!=(2n− 1)!n!(n− 1)!

= Cn−12n−1.

Suy ra wn = Cn−12n−1 ⇒ vn = Cn−1

2n−1 − 1⇒ un =Cn−1

2n−1−1n3 . Theo định lý Wolstenholme 3 thì

up ∈ Z, mà có vô số số nguyên tố nên cũng có vô số số hạng của dãy là số nguyên. Tacó đpcm.

Tập san Toán học STAR EDUCATION

Page 39: lovetoan.files.wordpress.com€¦ · 2 NGUYŽN TĂNG VŨ - NGUYŽN NG¯C DUY - VƯƠNG TRUNG DŨNG LÊ PHÚC LÚ - TRƒN BÁ Đ—T T−P SAN TOÁN H¯C STAR EDUCATION SŁ thø 03

38 KIỀU ĐÌNH MINH - LÊ PHÚC LỮ

Bài toán 6. 1) (IMO 1979) Chứng minh rằng 11 −

12 +

13 − ...+ 1

1319 ≡ 0 (mod1979).2) Tổng quát hóa bài toán.

Lời giải. 1) Chú ý rằng 1979 nguyên tố, ta chỉ cần tìm cách tách ghép thích hợp.

11−

12+

13− ...+

11319

=11+

12+

13+ ...+

11319

− 2�

12+

14+ ...+

11318

=11+

12+

13+ ...+

11319

−�

11+

12+

13+ ...+

1659

≡11+

12+

13+ ...+

11319

+�

11979− 1

+1

1979− 2+

11979− 3

+ ...+1

1979− 659

=11+

12+

13+ ...+

11319

+1

1320+ ...+

11978

≡989∑

i=1

1i+

11979− i

≡ 0 (mod1979)

2) Với ý tưởng tách ghép như trên, ta thấy rằng: - Nếu p = 3k+ 2 là số nguyên tố thì11 −

12 +

13 − ...+ 1

2k+1 ≡ 0 (mod p).- Nếu p = 3k+ 1 là số nguyên tố thì 1

1 −12 +

13 − ...− 1

2k ≡ 0 (mod p).

Bài toán 7. (Putnam 1996): Cho p ≥ 5 là số nguyên tố và đặt k =�2p

3

thìk∑

i=1C i

p ≡

1 (modp2).

Lời giải. Với p là số nguyên tố, ta có bổ đề quan trọng sau: 1p C k

p ≡(−1)k−1

k (mod p) với

1 ≤ k ≤ p − 1.Chỉ cần biến đổi trực tiếp là có đpcm. Chú ý rằng đây là một ý tưởngquan trọng, sử dụng cho nhiều bài khác.Trở lại bài toán, ta thấy rằng C k

p ≡ (−1)k−1 pk (modp2). Đến đây, áp dụng bài toán 5 ở

trên với kết quả: tổng đan dấu của đồng dư với 0 theo modulo p, ta có đpcm.

Bài toán 8. (IMO Shortlist 2013) Cho p là số nguyên tố lẻ. Đặt Sa =a1 +

a2

2 + ...+ap−1

p−1 , a ∈ Z+ Đặt S3 + S4 − 3S2 =mn với m, n ∈ Z+. Chứng minh rằng p|m.

Lời giải. Áp dụng bổ đề ở bài trên, ta có: ak

k ≡ (−1)k−1 ak

p C kp (mod p). Suy ra

Sa =≡p−1∑

k=1

(−1)k−1 ak

pC k

p =1p

ap − 1−p∑

k=0

(−1)kC kp ak

=(a− 1)p − ap + 1

p(mod p).

Do đó:

S3 + S4 − 3S2 ≡2p − 3p + 1+ 3p − 4p + 1− 3(2− 2p)

p

= −4p − 4 · 2p + 4

p= −(2p − 2)2

p(mod p)

Theo định lý Fermat nhỏ thì 2p ≡ 2 (mod p) nên p2�

� (2p − 2)2 hay (2p−2)2

p ≡ 0 (mod p)Từ đó suy ra đpcm. Đây là bài toán N7 trong đề IMO Shortlist 2011. Ý tưởng khó chínhlà phát hiện và sử dụng được bổ đề trên.

Tập san Toán học STAR EDUCATION

Page 40: lovetoan.files.wordpress.com€¦ · 2 NGUYŽN TĂNG VŨ - NGUYŽN NG¯C DUY - VƯƠNG TRUNG DŨNG LÊ PHÚC LÚ - TRƒN BÁ Đ—T T−P SAN TOÁN H¯C STAR EDUCATION SŁ thø 03

KIỀU ĐÌNH MINH - LÊ PHÚC LỮ

Bài toán 9. (Chọn đội tuyển Nghệ An 2012) Cho p > 3 là số nguyên tố và đặtM = {1,2, 3, ..., p}. Với mỗi số nguyên dương k thỏa 1 ≤ k ≤ p, xét tập hợp Ek =

{A⊂ M : |A|= k} và đại lượngxk =∑

A∈Ek

(min A+max A). Chứng minh rằngp−1∑

k=1xkC k

p ≡

0 (modp3).

Lời giải. Trước hết, với mỗi phần tử i, ta đếm có bao nhiêu tập hợp có k phần tử nhậni là min. Nếu i = 1 thì có C k−1

p−1 tập hợp; nếu i = 2 thì có C k−1p−2 tập hợp. Từ đó suy ra,

có tất cả C k−1p−i tập hợp nhận i là phần tử nhỏ nhất, trong đó p − i ≥ k − 1. Tương tự,

có tất cả C k−1i−1 tập hợp nhận i là phần tử lớn nhất, trong đó i ≥ k. Do đó:

xk =∑

A∈Ek

(min A+max A) =∑

p−i+1≥k

iC k−1p−i +

i≥k

iC k−1i−1

=∑

i≥k

(p+ 1− i)C k−1i−1 +

i≥k

iC k−1i−1

= (p+ 1)∑

i≥k

C k−1i−1 = (p+ 1)C k

p

Từ đây suy ra:p−1∑

k=1xkC k

p ≡ (p + 1)p−1∑

k=1C k

p C kp = (p + 1)

C p2p − 2

≡ 0(modp3) theo định

lý Wolstenholme 3.

Bài toán 10. (J28.Mathematical Reflections) Cho p là số nguyên tố sao cho p ≡1 (mod3) và đặt q =

�2p3

. Nếu 11.2 +

13.4 + ...+ 1

(q−1)q =mn với m, n ∈ Z, thì p|m.

Lời giải. Đặt p = 3k + 1 và q =�2p

3

= 2k. Khi xét phương trình modulo p taphải chứng minh nó đồng dư với 0 mod p. Ta có S = 1

1.2 +1

3.4 + ... + 1(q−1)q =

11 −

12 +

13 −

14 + ... + 1

q−1 −1q . Bây giờ ta nhóm các phân số lại và thay q = 2k , được

S =q∑

i=1

1i − 2

q/2∑

i=1

12i =

2k∑

i=1

1i −

k∑

i=1

1i . Từ định lý Wolstenholme, ta có 1

1 +12 + ... + 1

p−1 ≡

0�

modp2�

. Bởi vì −i ≡ p− i (modp) , nên suy ra

S =p−1∑

i=1

1i−

p−1∑

i=2k+1

1i+

k∑

i=1

1p− i

≡ 0−3k∑

i=2k+1

1i+

k∑

i=1

13k+ 1− i

≡ 0 (modp) .

Bài toán 11. (O.264. Mathematical Reflections) Cho p > 3 là một số nguyên tố.Chứng minh rằng

2p−1 ≡ 1�

modp2�

⇔(p−1)/2∑

k=2

1k

1+12+ ...+

1k− 1

≡ 0 (mod p).

Tập san Toán học STAR EDUCATION

Page 41: lovetoan.files.wordpress.com€¦ · 2 NGUYŽN TĂNG VŨ - NGUYŽN NG¯C DUY - VƯƠNG TRUNG DŨNG LÊ PHÚC LÚ - TRƒN BÁ Đ—T T−P SAN TOÁN H¯C STAR EDUCATION SŁ thø 03

40 KIỀU ĐÌNH MINH - LÊ PHÚC LỮ

Lời giải. Ta cần chỉ ra rằng(p−1)/2∑

k=2

Hk−1 (1)k

=12

H(p−1)/2 (1)�2−H(p−1)/2 (2)

≡ 2q2 (modp) .

Với Hk−1 (d) = 1 + 12d + ... + 1

(k−1)d. Từ đó, theo định lý Wolstenholme Hp−1 (1) ≡

Hp−1 (2)≡ 0 (modp) với số nguyên tố p > 3, suy ra

2q =2p − 2

p=

1p

p−1∑

k=1

pk

≡p−1∑

k=1

(−1)k−1

k(modp) =

=p−1∑

k=1

1k− 2

(p−1)/2∑

k=1

12k= Hp−1 (1)−H(p−1)/2 (1)≡ −H(p−1)/2 (1) (modp)

H(p−1)/2 (2) =12

(p−1)/2∑

k=1

1k2+

12

p−1∑

k= p+12

1

(p− k)2≡

12

Hp−1 (2)≡ 0 (modp)

Vì vậy12

H(p−1)/2 (1)�2−H(p−1)/2 (2)

≡12

(−2q)2 + 0�

≡ 2q2 (modp) .

Bài toán 12. (O.354. Mathematical Reflections) Tìm tất cả các số nguyên tố p sao chobiểu thức

p2

1+ 12 + ...+ 1

p−1

là một số nguyên.

Lời giải. Dễ kiểm tra rằng biểu thức là một số nguyên với p = 2, 3,5, 7. Ta chỉ ra rằngchỉ có những số nguyên tố như vậy thỏa mãn bài toán. Xét số nguyên tố p ≥ 11. ĐặtHp−1 = 1+ 1

2 + ...+ 1p−1 =

ab , (a, b) = 1. Theo định lý Wolstenholme có p2|a. Vì vậy p2

Hp−1

là một số nguyên khi a = p2. Ta xét các trường hợp sau

1. Nếu p = 11 thì 23.32.7 chia hết b. Vậy 1< H10 ≤112

8.9.7 < 1, mâu thuẫn.

2. Nếu 13 ≤ p ≤ 53 thì có hai số nguyên tố q1, q2 sao cho p+12 ≤ q1 < q2 < p − 1.

Do 23q1q2|b, ta có 1< Hp−1 ≤p2

8q1q2<

p2

2(p+1)2< 1, mâu thuẫn.

3. Cuối cùng, nếu p ≥ 59 thì tồn tại m ≥ 5 sao cho 2m ≤ p − 1 < 2m+1. Theo địnhlý Chebyshev (bổ đề Bertrand), tồn tại số nguyên tố q sao cho p+1

2 ≤ q < p− 1.Do 2mq|b, ta có

4,07< log2 59< Hp−1 ≤p2

2mq<

2m+1 + 1�

p

2m−1 (p+ 1)< 4+

116< 4, 07.

Điều mâu thuẫn này hoàn tất chứng minh.

Nhận xét. Đây là bài toán khó, có sự kết hợp nhiều kết quả quan trọng. Về bổ đềBertrand bạn đọc có thể xem ở số 432 tháng 6 năm 2013 trên tạp chí TH và TT.

Tập san Toán học STAR EDUCATION

Page 42: lovetoan.files.wordpress.com€¦ · 2 NGUYŽN TĂNG VŨ - NGUYŽN NG¯C DUY - VƯƠNG TRUNG DŨNG LÊ PHÚC LÚ - TRƒN BÁ Đ—T T−P SAN TOÁN H¯C STAR EDUCATION SŁ thø 03

KIỀU ĐÌNH MINH - LÊ PHÚC LỮ

4. Bài tập tự luyện

1. Đặt 112016 + 1

22016 + ...+ 120162016 = a

b với a, b ∈ Z+ và (a, b) = 1. Chứng minh rằngtổng a+ b chia hết cho 2017.

2. Giả sử p là số nguyên tố lớn hơn 3; q =�2p

3

; m, n ∈ Z+ thỏa mãn mn = 1− 1

2 +13 −

14 + ...+ (−1)q−1 1

q . Chứng minh rằng p|m.

3. Cho p là số nguyên tố n là số nguyên dương sao cho n ≥ p. Chứng minh rằng�

Cnn+p

�2− Cn

n+2p − C2pn+p chia hết cho p2.

4. (O.292. Mathematical Reflections) Với mỗi số nguyên dương k, đặt Tk =k∑

j=1

1j2 j .

Tìm tất cả các số nguyên tố p sao chop−2∑

k=1

Tkk+1 ≡ 0 (modp).

5. (O.54. Mathematical Reflections) Cho p = 2q + 1 là một số nguyên tố lớn hơn3. Chứng minh rằng p chia hết tử số của

1≤i, j≤q,i+ j>q

1i j , ở đây tổng được lấy trên

tất cả các cặp có thứ tự (i, j) .

6. (Morley, 1895) Với mọi số nguyên tố p ≥ 5, p = 2k + 1 thì (−1)p−1

2 C k2k ≡

4p−1�

modp3�

.

Tập san Toán học STAR EDUCATION

Page 43: lovetoan.files.wordpress.com€¦ · 2 NGUYŽN TĂNG VŨ - NGUYŽN NG¯C DUY - VƯƠNG TRUNG DŨNG LÊ PHÚC LÚ - TRƒN BÁ Đ—T T−P SAN TOÁN H¯C STAR EDUCATION SŁ thø 03

Tài liệu tham khảo

[1] Tạp chí Mathematical Reflections.

[2] Diễn đàn Toán học mathscope.org.

[3] Tạp chí Toán học và tuổi trẻ.

[4] Diễn đàn artofproblemsolving.com.

42

Page 44: lovetoan.files.wordpress.com€¦ · 2 NGUYŽN TĂNG VŨ - NGUYŽN NG¯C DUY - VƯƠNG TRUNG DŨNG LÊ PHÚC LÚ - TRƒN BÁ Đ—T T−P SAN TOÁN H¯C STAR EDUCATION SŁ thø 03

Bổ đề ERIQ và ứng dụng

Trương Tuấn Nghĩa(Lớp 11A1, Trường THPT Chuyên KHTN Hà Nội)

1. Giới thiệu

Bổ đề ERIQ được đặt tên bởi tác giả Kostas Vittas trên diễn đàn AoPS với nick namevittasko. (là các chữ viết tắt của cụm từ Equal Ratios In Quadrilateral). Nội dungbổ đề:Cho tứ giác ABC D, lấy các điểm M , N nằm trên cạnh AD, BC sao cho MA

M D =NBNC . Khi

đó, trung điểm của AB, MN , C D thẳng hàng.

Chứng minh. Gọi X , Y, Z là trung điểm của AB, MN , C D. Lấy P,Q nằm trên X M , X Nsao cho DP, CQ ‖ AB.

Khi đó, theo định lý Thales, ta có MAM D =

AXDP =

MXM P ; NB

NC =AYCQ =

NXNQ . Suy ra DP = CQ;

MXM P =

NXNQ hay MN ‖ PQ. Do DP = CQ; DP ‖ CQ nên PCQD là hình bình hành hay Z

là trung điểm PQ.

Kết hợp với Y là trung điểm của MN , ta có X , Y, Z thẳng hàng.

Nhận xét. Ta có thể chứng minh X , Y, Z là các điểm chia cùng tỉ lệ trên AB, MN , C Dthẳng hàng bằng cách tương tự. Tiếp theo, ta sẽ đến với một số các mở rộng và ứng dụngcủa bổ đề trên.

2. Ứng dụng

43

Page 45: lovetoan.files.wordpress.com€¦ · 2 NGUYŽN TĂNG VŨ - NGUYŽN NG¯C DUY - VƯƠNG TRUNG DŨNG LÊ PHÚC LÚ - TRƒN BÁ Đ—T T−P SAN TOÁN H¯C STAR EDUCATION SŁ thø 03

44 TRƯƠNG TUẤN NGHĨA

Bài toán 1. Cho tứ giác ABC D, lấy M , N nằm trên cạnh AD, BC sao cho MAM D =

NBNC .

Lấy các điểm X , Y, Z sao cho các tam giác XAB, Y MN , ZC D đồng dạng và X , Y, Z lầnlượt nằm trên các nửa mặt phẳng bờ AB không chứa C , MN không chứa D và C Dchứa A. Chứng minh rằng X , Y, Z thẳng hàng.

Lời giải. Lấy P,Q ∈ X M , X N sao cho DP ‖ XA, CQ ‖ X B.

Theo định lý Thales, DP = XA. M DMA , CQ = X B. NB

NC mà MAM D =

NBNC nên DP = CQ Mặt

khác vì ∠AX B = ∠C Z D nên ∠Z DP = ∠ZCQ. Do đó, Í Z DP =Í ZCQ(c.g.c) dẫn tới∠PZ D = ∠QZC hay ∠C Z D = ∠PZQ. Vì DP ‖ XA, CQ ‖ X B nên X M

M P =X NNQ (=

MAM D ) nên

MN ‖ PQ. Lấy Y ′ ∈ X Z sao cho X Y ′

Y ′Z =X MM P =

X NNQ . Theo định lý Thales, X Y ′

Y ′Z =X MM P =

X NNQ

nên

Y ′M ‖ Z P, Y ′N ‖ ZQ

Y ′M = Y ′N(= Z P.X Y ′

X Z= ZQ.

X Y ′

X Z)

Hay ∠MY ′N = ∠MY N , Y ′M = Y ′N . Do đó, Y ′ ≡ Y hay X , Y, Z thẳng hàng.

Bài toán 2. Cho tứ giác ABC D có phân giác trong của các góc ∠A,∠B,∠C ,∠D đồngquy tại I . AD cắt BC tại E, AB cắt C D tại F . Gọi M , N là trung điểm AC , EF. Chứngminh rằng M , N , I thẳng hàng.

Lời giải. Gọi P,Q là giao điểm của đường thẳng qua I , vuông góc với IB với BA, AC .Đầu tiên, dễ thấy I là giao 3 phân giác Í ABE. Do BI là phân giác ∠ABC nên Í BPQcân tại B hay I là trung điểm PQ.

Tập san Toán học STAR EDUCATION

Page 46: lovetoan.files.wordpress.com€¦ · 2 NGUYŽN TĂNG VŨ - NGUYŽN NG¯C DUY - VƯƠNG TRUNG DŨNG LÊ PHÚC LÚ - TRƒN BÁ Đ—T T−P SAN TOÁN H¯C STAR EDUCATION SŁ thø 03

TRƯƠNG TUẤN NGHĨA

Ta có ∠BPQ = 90◦ − ∠ABE2 = ∠AEB

2 + ∠BAE2 ,∠IAB = ∠BAE

2 nên ∠PIA = ∠AEB2 . Tương tự

thì ∠EIQ = ∠BAE2 . Do đó, Í PIA∼Í QEA(g.g) nên PA.QE = PI .QI . Hoàn toàn tương

tự, PF.QC = PI .QI . Vậy ta có PAFA =

QCQE nên theo bổ đề ERIQ, M , I , N lần lượt là trung

điểm của PQ, AC , EF thẳng hàng.

Bài toán 3. Cho tứ giác ABC D nội tiếp, không là hình thang. Gọi E, F là giao điểm củacác cặp đường thẳng (AB, C D); (AD, BC). Giả sử phân giác của góc ∠AEC ,∠AFB cắtnhau tại I . Gọi M , N lần lượt là trung điểm của AC , BD. Chứng minh rằng I ∈ MN .

Lời giải. Giả sử các điểm có vị trí như hình vẽ, các trường hợp khác tương tự.

Tập san Toán học STAR EDUCATION

Page 47: lovetoan.files.wordpress.com€¦ · 2 NGUYŽN TĂNG VŨ - NGUYŽN NG¯C DUY - VƯƠNG TRUNG DŨNG LÊ PHÚC LÚ - TRƒN BÁ Đ—T T−P SAN TOÁN H¯C STAR EDUCATION SŁ thø 03

46 TRƯƠNG TUẤN NGHĨA

Gọi P,Q lần lượt là giao điểm của F I với AB, C D. Do ∠ABC + ∠C DA = 180◦ nên∠FAB = ∠FC D nên Í FAB ∼Í FC D(g.g)(∗) và

∠EPQ = ∠FAB +∠AF I = ∠FC D+∠BF I = ∠EQP

hay Í EPQ cân tại E. Mà EI là phân giác ∠AED nên I là trung điểm PQ. Mặt kháctheo (*), FA

FB =FCF D nên theo tính chất đường phân giác, AP

PB =CQQD . Do đó theo bổ đề

ERIQ, trung điểm AC , BD, PQ thằng hàng hay I ∈ MN . (đpcm)

Bài toán 4. (AOPS). Cho Í ABC , trực tâm H,P bất kỳ trên BC , X bất kỳ trên HP.Gọi E, F 6= A là giao điểm của đường tròn đường kính AX với CA, AB. Tiếp tuyến tạiE, F của (AEF) cắt nhau tại T . Đường thẳng qua P vuông góc BC cắt CA, AB tại Z , Y .Gọi L là trung điểm ZY . Chứng minh rằng LT chia đôi BC .

Lời giải. Trước hết, ta phát biểu và chứng minh hai bổ đề sau:Bổ đề 1. ChoÍ ABC , đường cao BE, C F . Gọi M là trung điểm của BC . Khi đó, M E, M Flà tiếp tuyến của (AEF).Bổ đề trên có thể chứng minh dễ dàng qua các phép cộng góc.Bổ đề 2. Cho tứ giác ABC D, AB cắt C D tại E. Gọi H, K là trực tâm của Í EAD,Í EBC .Khi đó, HK là trục đẳng phương của 2 đường tròn đường kính BD, AC .

Chứng minh. Gọi M , N là hình chiếu của B, C lên EC , EB. Khi đó, MNBC là tứ giácnội tiếp nên KN .KC = KM .KB.

Mặt khác, M , N lần lượt nằm trên đường tròn đường kính BD, AC mà KN .KC =KM .KB nên K nằm trên trục đẳng phương của 2 đường tròn trên. Chứng minh tươngtự, HK là trục đẳng phương của đường tròn đường kính BD và đường tròn đườngkính AC .

Trở lại bài toán,

Tập san Toán học STAR EDUCATION

Page 48: lovetoan.files.wordpress.com€¦ · 2 NGUYŽN TĂNG VŨ - NGUYŽN NG¯C DUY - VƯƠNG TRUNG DŨNG LÊ PHÚC LÚ - TRƒN BÁ Đ—T T−P SAN TOÁN H¯C STAR EDUCATION SŁ thø 03

TRƯƠNG TUẤN NGHĨA

Gọi M , N là giao điểm của X F, X E với CA, AB. Khi đó, theo bổ đề 1 dễ có T là trungđiểm của MN nên theo bổ đề ERIQ, ta chỉ cần chứng minh BN

BZ =C MCY . Gọi U , V là hình

chiếu của N , M lên BC . Theo bổ đề 2 thì HX là trục đẳng phương của đường trònđường kính MB, NC . Dễ thấy U , V lần lượt nằm trên đường tròn đường kính CN , BMnên và P nằm trên HX , BC nên ta có PU .PC = PV.PB hay PB

PU =PCPV , và theo định lý

Thales thì BNBZ =

C MCY . Vậy ta thu được LT chia đôi BC .

Bài toán 5. Cho Í ABC , P bất kỳ trên BC , J là trung điểm của AP. Gọi E, F là giaođiểm của (J , JA) với CA, AB. Gọi L là tâm đường tròn ngoại tiếp Í J EF . Chứng minhrằng khi P di chuyển trên BC thì L chuyển động trên đường thẳng cố định.

Lời giải. Trước hết ta chứng minh bổ đề sau: Cho Í ABC , lấy điểm M cố định trênBC , P bất kỳ trên BC . Gọi E, F là hình chiếu của P lên CA, AB, K , L là hình chiếu củaM lên CA, AB. Khi đó, tỉ số EK

F L không phụ thuộc vào vị trí của P trên BC .

Tập san Toán học STAR EDUCATION

Page 49: lovetoan.files.wordpress.com€¦ · 2 NGUYŽN TĂNG VŨ - NGUYŽN NG¯C DUY - VƯƠNG TRUNG DŨNG LÊ PHÚC LÚ - TRƒN BÁ Đ—T T−P SAN TOÁN H¯C STAR EDUCATION SŁ thø 03

48 TRƯƠNG TUẤN NGHĨA

Chứng minh. Gọi X , Y là hình chiếu của M , P lên PF, MK . Khi đó,

MX = LF = M P. cos∠X M P = M P.cos∠ABC;

Y P = KE = M P. cos∠Y PM = M P. cos∠ACB.

Do đó, EKF L =

cos∠ACBcos∠ABC .

Trở lại bài toán,

Lấy M , N cố định trên BC . X , Z là hình chiếu của M lên AB, AC; Y, T là hình chiếucủa N lên AB, AC . Khi đó, theo bổ đề 1 thì dễ có được Y F

Y X =T ET Z . (1) Do J là tâm đường

tròn ngoại tiếp Í AEF nên ∠FJ E = 2.∠BAC . Mà L là tâm đường tròn ngoại tiếp củaÍ J EF nên ∠F LE = 360◦ − 4.∠BAC . Theo (1) và bổ đề ERIQ thì các đỉnh của tamgiác cân có đáy F E, Y T, X Z và có góc ở đỉnh là 360◦ − 4.∠BAC thì thẳng hàng màM , N cố định nên L nằm trên đường thẳng cố định. (đpcm)

Bài toán 6. (Nguyễn Văn Linh) Cho Í ABC , đường cao AD, K ∈ AD. Gọi E, F lầnlượt là giao điểm của BK , CK với CA, AB. Giả sử DE, DF cắt lại đường tròn ngoại tiếpÍ ABD;Í AC D tại M , N . Gọi T là trung điểm của MN . Chứng minh rằng AT chia đôiđoạn thẳng EF.

Lời giải. Gọi BP, CQ là đường cao của Í ABC , đường thẳng qua A song song BC cắtDE, DF tại K , L. Theo kết quả quen thuộc DF, DE đối xứng nhau qua AD và DQ, DPđối xứng nhau qua AD. Nên A là trung điểm của K L.

Tập san Toán học STAR EDUCATION

Page 50: lovetoan.files.wordpress.com€¦ · 2 NGUYŽN TĂNG VŨ - NGUYŽN NG¯C DUY - VƯƠNG TRUNG DŨNG LÊ PHÚC LÚ - TRƒN BÁ Đ—T T−P SAN TOÁN H¯C STAR EDUCATION SŁ thø 03

TRƯƠNG TUẤN NGHĨA

Khi đó, theo bổ đề ERIQ, ta chỉ cần chứng minh N LN F =

MKM E . Ta có, A, M , P, D,Q nằm

trên đường tròn và A, N ,Q, D, C nằm trên đường tròn. (1)Do đó, ∠NAQ = ∠N DQ,∠MAP = ∠M DP. Do DF, DE đối xứng nhau qua AD vàDQ, DP đối xứng nhau qua AD,nên ∠QDF = ∠PDE. Từ (1), ta cũng có ∠AQN =∠ADN = ∠ADM = ∠APM . Do đó, Í ANQ ∼Í AM P. (2)Mặt khác, F L

AL =sin LFAsin LAF ; KA

KE =sin KAEsin KEA. Vì AK = AL;∠FAL = ∠ABC;∠EAK = ∠ACB, nên

F LAL

.KAKE=

sin LFAsin FAL

.sin KAEsin KEA

=F LKE

=sin LFAsin KEA

.sin KAEsin FAL

=sin ACBsin ABC

.sin LFAsin KEA

=ABAC

.sin LFAsin KEA

.

Ta lại có

sin LFAsin KEA

=sin N FAsin NAF

.sin MAPsin M EA

=ANFN

.M EMA=

ANAM

.M EFN=

AQAP

.M EFN=

ACAB

.M EFN

.

(do (2)). Vậy nênF LKE=

ABAC

.sin LFAsin KEA

=AB.ACAC .AB

.M EN F=

M EN F

.

Bài toán 7. (Chọn đội tuyển PTNK TPHCM) Cho Í ABC , trực tâm H. Lấy điểm Mbất kỳ trên cung BHC của (BHC). Trên BM , C M lấy các điểm E, F sao cho ∠ECA=∠FBA= 90◦. Chứng minh rằng khi M chuyển động thì trung điểm EF luôn nằm trênđường thẳng cố định.

Lời giải. Ở bài toán này, ta có hai hướng tiếp cận như sau:Cách 1. Gọi N là giao điểm của C E, BF. Lấy P đối xứng với N qua BC , BP, C P lần lượtcắt C E, BF tại X , Y. Dễ dàng chứng minh B, H, M , P, C nằm trên đường tròn.

Tập san Toán học STAR EDUCATION

Page 51: lovetoan.files.wordpress.com€¦ · 2 NGUYŽN TĂNG VŨ - NGUYŽN NG¯C DUY - VƯƠNG TRUNG DŨNG LÊ PHÚC LÚ - TRƒN BÁ Đ—T T−P SAN TOÁN H¯C STAR EDUCATION SŁ thø 03

50 TRƯƠNG TUẤN NGHĨA

Ta sẽ chứng minh X EY F không đổi khi M chuyển động trên cung BHC . Do ∠BMC =

∠BNC = 180◦ −∠BAC nên ∠C M E = ∠CN F hay 4 điểm M , N , E, F nằm trên đườngtròn nên ∠C FY = ∠BEX . (1) Mặt khác, do B, H, M , P, Cnằm trên đường tròn nên∠Y C F = ∠MC P = ∠X BE. (2) Từ (1) và (2) suy ra Í CY F ∼Í BX E(g.g). Do đó,X EY F =

BXCY không đổi. Vậy X E

Y F không đổi khi M chuyển động trên cung BHC nên theobổ đề ERIQ, trung điểm của EF luôn nằm trên đường thẳng cố định.

Cách 2. Trước hết ta phát biểu và chứng minh bổ đề sau: (IMO2009 Shortlist G4)Cho tứ giác ABC D nội tiếp đường tròn (O). AC cắt BD ở E, AD cắt BC tại F. Gọi M , Nlần lượt là trung điểm của AB, C D. Khi đó, EF tiếp xúc với đường tròn ngoại tiếp củaÍ EMN .

Chứng minh. Gọi I là trung điểm của EF. Xét tứ giác toàn phần AEBF.C D có I , M , Nlần lượt là trung điểm của các đường chéo EF, AB, C D nên I , M , N thẳng hàng.

Tập san Toán học STAR EDUCATION

Page 52: lovetoan.files.wordpress.com€¦ · 2 NGUYŽN TĂNG VŨ - NGUYŽN NG¯C DUY - VƯƠNG TRUNG DŨNG LÊ PHÚC LÚ - TRƒN BÁ Đ—T T−P SAN TOÁN H¯C STAR EDUCATION SŁ thø 03

TRƯƠNG TUẤN NGHĨA

Ta sẽ chứng minh I M .IN = I E2. Gọi L, P, T lần lượt là giao điểm của AB với C D,EF với AB, C D. Khi đó, (LP, AB) = (LT, C D) = −1 nên áp dụng hệ thức Maclaurinvà ABC D là tứ giác nội tiếp, ta thu được LM .LP = LA.LB = LC .LD = LT .LN nên4 điểm M , P, N , T nằm trên đường tròn. Do đó, I M .IN = I P.I T . Mặt khác, ta lại có(EF, PT ) = −1 nên theo I E2 = I T .I P. Vậy I M .IN = I E2. Do đó, EF là tiếp tuyến củađường tròn ngoại tiếp Í EMN . (đpcm)

Trở lại bài toán,Gọi N là giao điểm của C E, BF. Lấy I , P,Q lần lượt là trung điểm của BC , EF, MN .

Theo lời giải thứ nhất, ta có 4 điểm M , N , E, F nằm trên đường tròn nên theo bổ đề 4thì BC là tiếp tuyến của (QC P) hay IC2 = IQ.I P. Do đó, I IC2

I : P↔Q. (1)Mặt khác V 2

N : Q 7→ M mà M chuyển động trên cung BHC nên Q chuyển động trênđường tròn (ω) cố định. (2)Từ (1) và (2), ta thu được P chuyển động trên đường thẳng ảnh của (ω) qua I IC2

I :P↔Q.

Nhận xét. Qua các bài toán trên, ta có thể thấy được ứng dụng của bổ đề ERIQ trongcác bài toán hình học. Sau đây sẽ là một số các bài toán luyện tập.

3. Luyện tập

1. Cho Í ABC nội tiếp (O). Tiếp tuyến của (O) tại A cắt tiếp tuyến của (O) tại B, Clần lượt tại E, F . Gọi M , N là trung điểm của BF, C E. Đường thẳng qua O vàvuông góc với OA cắt BC tại S. Chứng minh rằng MN chia đôi SO.

2. Cho Í ABC , trực tâm H, trung tuyến AM . P bất kỳ trên HM . Đường tròn đườngkính AP cắt CA, AB tại E, F . Tiếp tuyến tại E, F của (AEF) cắt nhau tại T . Chứngminh rằng T B = T C .

Tập san Toán học STAR EDUCATION

Page 53: lovetoan.files.wordpress.com€¦ · 2 NGUYŽN TĂNG VŨ - NGUYŽN NG¯C DUY - VƯƠNG TRUNG DŨNG LÊ PHÚC LÚ - TRƒN BÁ Đ—T T−P SAN TOÁN H¯C STAR EDUCATION SŁ thø 03

52 TRƯƠNG TUẤN NGHĨA

3. Cho Í ABC , đường tròn (K) đi qua B, C cắt CA, AB tại E, F . Gọi H là giao điểmcủa BE, C F. Lấy P bất kỳ trên BC . Đường thẳng qua P và song song với AH cắtCA, AB tại X , Y.Lấy Q bất kỳ trên HP. Đường thẳng qua Q song song với BE, C Fcắt CA, AB tại X , Y, Z , T.a) Chứng minh rằng 4 điểm X , Y, Z , T nằm trên đường tròn (L).b) K L cắt trung trực PQ tại Z . Chứng minh rằng Í Z PQ ∼Í KBC .

4. Cho Í ABC , P bất kỳ trên BC . Đường thẳng qua P song song với CA, AB cắttrung trực BA, AC tại M , N . Chứng minh rằng khi P chuyển động trên BC , tâmđường tròn ngoại tiếp của Í MN P luôn nằm trên một đường thẳng cố định.

5. (Việt Nam TST 2008) Cho Í ABC nhọn không cân nội tiếp (O). Với k ∈ R+,trên các đoạn phân giác AD, BE, C F, lấy M , N , P sao cho AM

AD =BNBE =

C PC F = k. Vẽ

đường tròn (O1) đi qua A, M và tiếp xúc với OA; vẽ đường tròn (O2) đi qua B, Nvà tiếp xúc với OB; vẽ đường tròn (O3) đi qua C , P và tiếp xúc với OC . Tìm tấtcả các giá trị k sao cho (O1), (O2), (O3) có đúng hai điểm chung.

6. Cho tam giác ABC nhọn không cân có điểm D thay đổi trong tam giác sao cho∠ABD = ∠AC D, lấy E ∈ AB, F ∈ AC sao cho D là trực tâm tam giác AEF. Chứngminh rằng:a) Trung tuyến đỉnh D của tam giác DEF luôn đi qua điểm cố định.b) Trung trực EF luôn đi qua điểm cố định.c) Tâm đường tròn ngoại tiếp tam giác (DEF) luôn thuộc đường cố định.d) Trục đẳng phương của (BDE), (C DF) luôn đi qua một điểm cố định.

Tập san Toán học STAR EDUCATION

Page 54: lovetoan.files.wordpress.com€¦ · 2 NGUYŽN TĂNG VŨ - NGUYŽN NG¯C DUY - VƯƠNG TRUNG DŨNG LÊ PHÚC LÚ - TRƒN BÁ Đ—T T−P SAN TOÁN H¯C STAR EDUCATION SŁ thø 03

Tài liệu tham khảo

[1] Nguyễn Văn Linh, Về bài 3 đề VMO 2016.

[2] Nguyễn Văn Linh, 2015, Định lý ERIQ, https://nguyenvanlinh.wordpress.com/.

[3] Diễn đàn artofproblemsolving.com/community

[4] Trần Quang Hùng, Các bài giảng đội tuyển.

53

Page 55: lovetoan.files.wordpress.com€¦ · 2 NGUYŽN TĂNG VŨ - NGUYŽN NG¯C DUY - VƯƠNG TRUNG DŨNG LÊ PHÚC LÚ - TRƒN BÁ Đ—T T−P SAN TOÁN H¯C STAR EDUCATION SŁ thø 03

Đếm bằng hai cách trong tổ hợp

Nguyễn Tăng Vũ(Giáo viên PTNK TP Hồ Chí Minh)

1. Giới thiệu

Đếm bằng hai cách là một phương pháp hay gặp trong đời sống, ví dụ bài toán sau:Một công ty nhập vào 3 xe hàng A, B, C gồm hai loại hàng I và I I . Trong đó xe A có 3loại I và 2 loại I I , xe B có 4 loại I và 6 loại I I , xe C có 4 loại I và 6 loại I I . Tính sốlượng hàng mà công ty nhâp vào.Đây là bài toán khá đơn giản, để giải bài toán ta có thể lập bảng và khi đó ta có thểtính bằng 2 cách như sau: Tính tổng số hàng trên mỗi xe rồi cộng lại; hoặc ta có thểtính tổng số hàng loại I trên 3 xe,tổng số hàng loại 2 trên 3 xe, rồi sau đó cộng lại.

Trên đây là một ví dụ của tính bằng hai cách, ta có thể tính tổng theo dòng hoặc cóthể tính tổng theo cột. Tổng quát hơn ta có công thức đại số sau

i∈I , j∈J

ai j =∑

i∈I

j∈J

ai j

=∑

j∈J

i∈I

ai j

Trong một số tình huống đề bài yêu cầu đếm số phần tử của một tập hợp mà khôngquan tâm ta đếm bằng cách nào, khi đó đếm bằng hai cách cho ta cùng một đáp sốgiống nhau, khi đó ta sẽ thiết lập được một đẳng thức tổ hợp. Một ví dụ đơn giảnnhư đếm số tập con của tập có n phần tử, ta có thể đếm số tập có k phần tử vớik = 0, 1, ..., n, lấy tổng ta được

C0n + C1

n + ....+ Cnn .

Nhưng nếu ta đếm bằng cách khác như sau: xét một tập hợp A bất kì, khi đó phần tửi có thể thuộc A hoặc i không thuộc A, mỗi phần tử có 2 trường hợp, mà có n phần tửnên số tập A là 2n. Từ đó ta có đẳng thức

C0n + C1

n + ....+ Cnn = 2n.

Đếm bằng hai cách cho ta một phương pháp để chứng minh đẳng thức liên quan tớihệ số khai triển nhị phân hay các đẳng thức tổ hợp.

54

Page 56: lovetoan.files.wordpress.com€¦ · 2 NGUYŽN TĂNG VŨ - NGUYŽN NG¯C DUY - VƯƠNG TRUNG DŨNG LÊ PHÚC LÚ - TRƒN BÁ Đ—T T−P SAN TOÁN H¯C STAR EDUCATION SŁ thø 03

NGUYỄN TĂNG VŨ

Ngoài ra đếm bằng hai cách có thể áp dụng trong các bài toán bất đẳng thức, cực trịtổ hợp hay một số bài toán chứng minh sự tồn tại. Để sử dụng phương pháp đếm bằnghai cách, đòi hỏi học sinh phải biết và vận dụng tốt các phép đếm cơ bản. Bài viết nàyđược sử dụng để giảng dạy cho học sinh lớp 10 chuyên Toán, các em mới bước đầulàm quen với các bài toán tổ hợp nói chung và các bài toán đếm nói riêng nên ví dụđược nêu ra có độ khó không cao giúp các em làm quen với phương pháp này.

2. Chứng minh các đẳng thức tổ hợp

Bài toán 1. Cho các số nguyên dương n và k với 0< k ≤ n. Chứng minh:

1. C kn = C k

n−1 + C k−1n−1

2.∑

k≥0 C2kn = 2n−1

Lời giải

1. Dễ thấy vế trái của đẳng thức là số cách chọn k phần tử từ n phần tử. Để chọnk phần tử từ n phần tử ta có thể làm như sau: Xét phần tử a, nếu a được chọnthì ta cần chọn thêm k−1 phần tử từ n−1 phần tử còn lại ta có C k−1

n−1 cách. Nếua không được chọn, ta chọn k phần tử từ n− 1 phần tử còn lại, ta có C k

n−1. Dođó số cách chọn trong hai trường hợp là C k

n−1 + C k−1n−1 . Từ đó ta có đpcm.

2. Ta xét bài toán "đếm số cách chọn một số chẵn phần tử từ n phần tử":Cách 1: Ta có số cách chọn 2k phần tử từ n phần tử là C2k

n . Khi đó∑

k≥0 C2kn lần

tổng số cách chọn một số chẵn phần tử từ n phần tử.Cách 2: Xét một phần tử a, thì có hai khả năng a được chọn hoặc a không đượcchọn, ta có 2 trường hợp. Khi đó với n−1 phần tử đầu tiên, thì số trường hợp là2n−1. Tới phần tử thứ n, nếu ta đã chọn được một số chẵn phần tử thì ta khôngchọn, còn nếu ta đã chọn được một số lẻ phần tử thì phần tử này sẽ được chọn,do đó số cách chọn là 2n−1.

Bài toán 2. Cho các số nguyên dương n và k với 0≤ k ≤ n. Chứng minh rằng:

1. kC kn = nC k−1

n−1

2.∑n

k=0 kC kn = n2n−1

Tập san Toán học STAR EDUCATION

Page 57: lovetoan.files.wordpress.com€¦ · 2 NGUYŽN TĂNG VŨ - NGUYŽN NG¯C DUY - VƯƠNG TRUNG DŨNG LÊ PHÚC LÚ - TRƒN BÁ Đ—T T−P SAN TOÁN H¯C STAR EDUCATION SŁ thø 03

56 NGUYỄN TĂNG VŨ

Lời giải

1. Xét bài toán "Một đội văn nghệ có n thành viên, có bao nhiêu cách chọn k ngườithể hiện một tiết mục hát tốp ca trong đó có một bạn hát sô lô".Cách 1: Chọn đội văn nghệ gồm k người từ n ta có số cách là C k

n , từ k người nàyta chọn một người hát sô lô có k cách. Khi đó số cách chọn là kC k

n .(1)Cách 2: Chọn người hát sô lô trước, có n cách, sau đó chọn k− 1 người từ n− 1người còn lại có C k−1

n−1 cách. Vậy số cách chọn là nC k−1n−1 . (2)

Từ (1) và (2) ta có đẳng thức kC kn = nC k−1

n−1 .

2. Xét bài toán "Từ n thành viên của đội văn nghệ, có bao nhiêu cách lập một nhómhát trong đó có một nhóm trưởng?". Làm tương tự như câu trên ta sẽ có đẳngthức cần chứng minh.

Bài toán 3. Cho các số nguyên dương n và k với 0≤ k ≤ n. Chứng minh rằng:

1.∑n

m=k C km = C k+1

n+1

2.∑n−k

m=k C kmC k

n−m = C2k+1n+1 với 0≤ k ≤

n2

.

Lời giải

1. Xét tập X = 1,2, ..., n+ 1. Khi đó ta đếm số tập con có k+ 1 phần tử của X .Cách 1: Rõ ràng số tập con là C k+1

n+1 .Cách 2: Ta chọn tập con sao cho phần tử lớn nhất là m. Khi đó số tập con cóphần tử lớn nhất m là C k

m. Vì k ≤ m≤ n nên ta có số tập con là C kk+C k

k+1+...+C kn .

Từ đó suy ra đẳng thức cần chứng minh.

2. Xét bài toán "Đếm số tập con có 2k+ 1 phần tử của X ".Cách 1: Số tập con là C2k+1

n .Cách 2: Ta xét phần tử thứ k + 1, giả sử đó là m, khi đó ta chọn k phần tử nhỏhơn m và k phần tử lớn hơn m, số cách chọn là C k

mC kn−m, vì k ≤ m ≤ n− k nên

ta có số cách chọn là∑n−k

m=k C kmC k

n−m. Từ đó ta có đẳng thức cần chứng minh.

Tiếp theo là các bài tập áp dụng:

Bài 1 Cho 0≤ m≤ k ≤ n. Chứng minh các đẳng thức sau:

1. C kn C k

m = Cmn C k−m

n−m

2.∑

k≥0 k(C kn )

2 = nCn−12n−1

3.∑

k≥0 C kn Cm−k

n−k = 2mCmn

Bài 2. Chứng minh các đẳng thức sau:

1. C0n .C k

n + C1n .C k−1

n−1 + ...+ C kn .C0

n−k = 2k.C kn

2. k.C km.C0

p + (k− 1).C k−1m .C1

p + ...+ C1mC k−1

p =m

m+ p.k.C k

m+p

3. Cmn .C0

k + Cm−1n−1 .C1

k+1 + ...+ C0n−m.Cm

k+m = Cmn+k+1

Tập san Toán học STAR EDUCATION

Page 58: lovetoan.files.wordpress.com€¦ · 2 NGUYŽN TĂNG VŨ - NGUYŽN NG¯C DUY - VƯƠNG TRUNG DŨNG LÊ PHÚC LÚ - TRƒN BÁ Đ—T T−P SAN TOÁN H¯C STAR EDUCATION SŁ thø 03

NGUYỄN TĂNG VŨ

3. Các bài toán chứng minh

Bài toán 4. Trong một hội nghị, mỗi thành viên tham gia đúng 3 cuộc họp và mỗicuộc họp thì có đúng 6 thành viên tham gia. Chứng minh rằng số cuộc họp thì bằngnửa số thành viên tham gia hội nghị.

Lời giải Gọi số thành viên là n, số cuộc hộp là m. Khi đó mỗi cuộc họp có 6 thànhviên tham gia, nên tổng số lượt thành viên tham gia m cuộc họp là 6m (có lặp lại).Tương tự mỗi thành viên tham gia 3 cuộc họp mà có n thành viên nên số lượt thànhviên tham gia là 3n. Do đó 3n= 6m hay n= 2m.Trong bài toán trên ta có thể làm như sau: giả sử có m cuộc họp là 1,2, ..., m và nthành viên là 1,2, 3, ..., n. Xét bảng vuông m× n gồm m dòng và n cột trên đó ghi cácsố dòng thứ i cột j là ai j thỏa ai j = 1 nếu người j tham gia cuộc họp thứ i và ai j = 0trong trường hợp ngược lại. Ta được bảng sau:

Dựa vào trên, ta thấy mỗi dòng có 6 số 1 và mỗi cột có 3 số 1. Khi đó ta có 6m = 3nhay n= 2m.

Bảng trên được gọi là một ma trận nhị phân, dùng để biểu diễn các mối quan hệ haingôi như phần tử thuộc tập hợp, quen nhau, đồ thị... và là mô hình biểu diễn rất hữudụng trong các bài toán tổ hợp. Trong mỗi bảng nhị phân trên, nếu gọi ri là số số 1 ởdòng thứ i và c j là số số 1 ở cột thứ j, ta có

m∑

i=1

ri =n∑

j=1

c j.

Bài toán 5. (Hồng Kông 1994) Trong một trường học có m giáo viên và n học sinhthỏa điều kiện sau:

i) Mỗi giáo viên dạy đúng p học sinh.

ii) Với hai học sinh phân biệt thì có đúng q giáo viên dạy họ.

Chứng minh rằngmq=

n(n− 1)p(p− 1)

Lời giảiLập bảng gồm m dòng và n cột trong đó ai j = 1 nếu giáo viên i dạy học sinh j, vàbằng 0 nếu ngược lại. Khi đó từ (i) thì mỗi dòng có đúng p số 1. Ta đếm các cặp số(1;1) trên cùng một dòng. Nếu đếm theo dòng thì mỗi dòng có C2

p cặp, có m dòng

nên số cặp là mC2p . (1)

Nếu đếm theo cột, do điều kiện (ii) nên với hai cột bất kì thì có đúng q cặp. Do đó số

Tập san Toán học STAR EDUCATION

Page 59: lovetoan.files.wordpress.com€¦ · 2 NGUYŽN TĂNG VŨ - NGUYŽN NG¯C DUY - VƯƠNG TRUNG DŨNG LÊ PHÚC LÚ - TRƒN BÁ Đ—T T−P SAN TOÁN H¯C STAR EDUCATION SŁ thø 03

58 NGUYỄN TĂNG VŨ

cặp là qC2n (2). Từ (1) và (2) ta có mC2

p = qC2n hay

mq=

n(n− 1)p(p− 1)

.

Trên đây là một kĩ thuật đếm theo cặp (1;1) cùng một dòng hoặc cùng một cột. Ta cómệnh đề sau:

Định lý 1. Nếu trong một bảng nhị phân m× n, mỗi dòng có k số 1, hai cột bất kỳ cóđúng p cặp (1;1) cùng một dòng. Khi đó ta có pC2

n = kC2m.

Tiếp theo là các bài tập áp dụng:

1. Cho tập X = 1, 2, ..., 8 và các tập A1, A2, ...,A6 là các tập con của X sao cho mỗitập Ai có 4 phần tử và mỗi phần tử của S thuộc m tập Ai. Tìm m.

2. Trong một vòng thi toán chung kết tại trường PNTK, các thí sinh phải giải 9 bàitoán. Biết rằng mỗi thí sinh giải được đúng 6 bài, và với hai thí sinh bất kì thìgiải đúng chung 3 bài. Tìm số thí sinh dự thi.

3. Gọi p(n, k) là số hoán vị của 1, 2, ..., n có k điểm bất động. Chứng minh rằng:

n∑

k=1

kp(n, k) = n!.

4. Các bài toán về bất đẳng thức và cực trị tổ hợp

Bài toán 6. (Iran 2011) Cho n điểm trên mặt phẳng sao cho không có 3 điểm nàothẳng hàng. Chứng minh rằng số tam giác có diện tích bằng 1 có các đỉnh thuộc n

điểm trên không vượt quá23(n2 − n).

Lời giảiBài toán này ta đi tính số cặp (cạnh;tam giác). Với đoạn thẳng AB, khi đó nếu

điểm C thỏa SABC = 1 thì khoảng cách từ C đến AB bằng2

AB, vì không có 3 điểm nào

thẳng hàng nên chỉ có nhiều nhất 4 điểm thỏa. Vậy với 1 đoạn ta sẽ có nhiều nhất 4tam giác có diện tích 1 nhận đoạn thẳng đó làm đỉnh. Suy ra tổng số cặp nhiều nhấtlà 4C2

n .Mặt khác nếu gọi số tam giác là m thì tổng số cặp là 3m. Từ đó ta có:

3m≤ 4C2n hay m≤

23(n2 − n).

Bài toán 7. (IMO 1998, P2) Trong một cuộc thi có a thí sinh và b giám khảo, vớib là số lẻ lớn hơn 3. Mội giám khảo có thể đánh giá thí sinh rớt hay đậu.Giả sử vớihai giám khảo bất kì thì quyết định giống nhau nhiều nhất là k thí sinh. Chứng minh

rằngka≥

b− 12b

Tập san Toán học STAR EDUCATION

Page 60: lovetoan.files.wordpress.com€¦ · 2 NGUYŽN TĂNG VŨ - NGUYŽN NG¯C DUY - VƯƠNG TRUNG DŨNG LÊ PHÚC LÚ - TRƒN BÁ Đ—T T−P SAN TOÁN H¯C STAR EDUCATION SŁ thø 03

NGUYỄN TĂNG VŨ

Lời giải Cũng như ví dụ trên, ta thấy việc biểu diễn các mối quan hệ bằng bảng nhịphân rất thuận lợi trong việc trình bày lời giải. Trong bài này ta cũng có thể lập bảngb× a theo quy tắc sau: dòng i cột j bằng 1 nếu giám khảo i cho thí sinh j đậu. Ta sẽđếm số cặp (0;0) và (1; 1) cùng một cột bằng hai cách.Cách 1 ta đếm theo dòng: Vì với hai vị giáo bất kì có nhiều nhất k kết luận giống nhaunên với hai dòng bất kì có k cặp, do đó số cặp nhiều nhất là kC2

b .Cách 2 ta đếm theo cột: Trong mỗi cột số cặp là C2

m + C2n cặp, trong đó m là số các số

0 và n là số các số 1, ta có m+ n= b = 2t + 1, suy ra n= 2t + 1−m. Khi đó

C2m + C2

n =m(m− 1) + (21−m)(2t −m− 1)

2=(2t −m)2 +m2

2≥ t2 =

(b− 1)2

4.

Từ đó ta có

kC2b ≥

a(b− 1)2

4suy ra

ka≥

b− 12b.

Bài toán 8. (USA TST 2005) Cho n> 1. Với số nguyên dương m. Đặt Xm = {1,2, ..., mn}.Xét họ T gồm 2n tập hợp thỏa các điều kiện sau:

i) Mỗi phần tử của T là một tập con có m phần tử của Xm.

ii) Mỗi cặp thuộc T có nhiều nhất một phần tử chung.

iii) Mỗi phần tử thuộc Xm thuộc đúng hai tập của T.Tìm giá trị lớn nhất của m theo n.

Lời giải Xét bảng vuông sao cho gồm 2n dòng và mn cột sao cho ai j = 1 nếu số jthuộc ai và bằng 0 trong trường hợp ngược lại.Ta xét bài toán đếm số cặp (1; 1) cùng một cột. Do (i) nên ta có số cặp nhiều nhất làC2

2n. Do (ii) nên ta có số cặp là mn.Do đó mn ≥ C2

2n, suy ra m ≥ 2n − 1. Nếu m = 2n − 1, ta xét mô hình sau. Cho 2nđường thẳng không có 3 đường nào đồng quy và không có hai đường nào song song.Khi Xm là tập các giao điểm và T là họ gồm các điểm thuộc một đường thẳng. Rõràng đây là mô hình thỏa đề bài. Bảng sau cho ví dụ n= 2, m= 3.

Bài toán 9. Cho n điểm trong mặt phẳng. Chứng minh rằng số cặp điểm có khoảng

cách bằng 1 không quán4+p

2n3

2.

Tập san Toán học STAR EDUCATION

Page 61: lovetoan.files.wordpress.com€¦ · 2 NGUYŽN TĂNG VŨ - NGUYŽN NG¯C DUY - VƯƠNG TRUNG DŨNG LÊ PHÚC LÚ - TRƒN BÁ Đ—T T−P SAN TOÁN H¯C STAR EDUCATION SŁ thø 03

60 NGUYỄN TĂNG VŨ

Lời giải Gọi di là số đoạn thẳng đơn vị có đỉnh là Ai. Đặt khi đó số cặp điểm là

k =12(d1 + d2 + ...+ dn).

Ta đếm số cặp (A, B) mà khoảng cách từ A, B đến Ai bằng 1. Số cặp là C2di, suy ra tổng

số cặp là∑n

i=1 C2di

. Ta biết rằng hai điểm C , D thì có chung nhiều nhất một cặp (A, B)nên số cặp không vượt quá 2C2

n . Do đó:∑n

i=1 C2di≤ n(n− 1) hay

2k(2k− n)2n

≤ n(n− 1)⇔ 2k2 − nk− n2(n− 1)≤ 0.

Do đó k ≤n4+p

2n3

2.

5. Các bài toán tồn tại

Bài toán 10. Cho 133 số nguyên dương, có ít nhất 799 cặp số là nguyên tố cùngnhau. Chứng minh rằng tồn tại 4 số nguyên dương phân biệt a, b, c, d sao cho a vàb; b và c, c và d; d và a nguyên tố cùng nhau.

Lời giải Mỗi số được đại diện bởi một điểm, hai số nào nguyên tố cùng nhau thìhai điểm tương ứng được nối nhau bởi một đoạn. Ta cần chứng minh có 4 đoạnAB, BC , C D, DA. Ta cần chứng minh rằng có hai điểm B và D cùng nối với hai điểm Avà C .Gọi di là số cạnh có đỉnh là Ai. Khi đó ta có d1 + d2 + ...+ d133 = 2. Nếu hai đỉnh Y, Zcùng nối với đỉnh X thì ta sẽ xem (Y ; Z) là một cặp. Ta sẽ tính số cặp này. Rõ ràng,tổng số cặp là

133∑

i=1

C2di=

12

133∑

i=1

d2i −

133∑

i=1

di

Theo BĐT Cauchy-Schwarz thì∑133

i=1 d2i ≥

1133

∑133i=1 di

�2nên do đó

133∑

i=1

C2di≥

12

1133

133∑

i=1

di

�2

−133∑

i=1

di

> C2133.

Nhưng 133 điểm thì có C2133 cặp, nên sẽ có một cặp nào đó được tính hai lần, tức là

tồn tại cặp (A, C) cùng được nối với B và D; tức là ta có 4 đoạn AB, BC , C D, DA.

Bài toán 11. Cho tập X có n phần tử, gọi A1, A2, ..., Am là một họ các tập con của X ,sao cho |Ai|= 3 và |Ai ∩A j| ≤ 1 với i 6= j. Chứng minh rằng tồn tại một tập con A củaX có ít nhất [

p2n] phần tử và không chứa bất kì tập Ai nào.

Lời giải Ta xét tập tất cả các tập con của X mà không chứa bất kỳ tập Ai nào, khiđó dễ thấy tập này là khác rỗng (xét tập có 2 phần tử là thỏa) và hữu hạn, nên

Tập san Toán học STAR EDUCATION

Page 62: lovetoan.files.wordpress.com€¦ · 2 NGUYŽN TĂNG VŨ - NGUYŽN NG¯C DUY - VƯƠNG TRUNG DŨNG LÊ PHÚC LÚ - TRƒN BÁ Đ—T T−P SAN TOÁN H¯C STAR EDUCATION SŁ thø 03

NGUYỄN TĂNG VŨ

tồn tại một tập M có nhiều phần tử nhất. Đặt |M | = k. Khi đó, do M có số phầntử lớn nhất nên mọi tập có số phần tử lớn hơn M đều chứa một tập Ai. Xét tậpM ′ = X \ M = {a1, a2, ..., an−k}. Khi đó M ∪ {ai} có k + 1 phần tử, nên theo cách xácđịnh M thì sẽ tồn tại Ai ⊂ M ′, do Ai * M nên Ai = {ai, x , y} trong đó x , y ∈ M .

Hơn nữa hai tập giao nhau có không quá một phần tử nên với mỗi ai có nhiều nhấtmột cặp (x , y) ∈ Ai. Ta đếm số cặp (x , y) theo hai cách:

• Số cặp (x , y) ∈ X là C2k .

• Vì i = 1,2, ..., n− k nên có n− k cặp nên ta có: n− k ≤ C2k ⇔ k2 + k ≥ 2n

Mà k ≤p

k2 + k ≤ k+ 1, suy ra k ≥ [p

2n]. Ta có điều cần chứng minh.

Cuối cùng là một số bài toán rèn luyện:

1. Cho 7 tập A1, A2, ...,A7 là các tập con của X = {1, 2,3, 4,5, 6,7}, sao cho mội cặpphần tử thuộc X thuộc đúng một tập con, và |Ai| ≥ 3 với mọi i. Chứng minhrằng |Ai ∩ Aj|= 1 với mọi i, j.

2. Cho 16 bạn học sinh làm một bài kiểm tra trắc nghiệm, trong đó mỗi câu hỏi có4 lựa chọn. Sau bài kiểm tra, ta thấy rằng với hai học sinh bất kì có nhiều nhấtmột câu trả lời giống nhau. Hỏi bài kiểm tra có nhiều nhất bao nhiêu câu hỏi?

3. Một hội nghị có n thành viên tham gia, hội nghị đã tổ chứng n + 1 cuộc họp,trong đó mỗi cuộc họp có đúng 3 người và không có cuộc họp nào có thành viêngiống nhau. Chứng minh rằng có hai cuộc họp mà có chung đúng một thànhviên.

4. (China 1996) Trong một hội nghị có 8 người tham gia, hội nghị tổ chức m cuộchọp, mỗi cuộc họp có đúng 4 người tham gia. Hơn nữa hai người bất kì thì cùngtham gia một số cuộc họp như nhau. Tìm giá trị nhỏ nhất của m.

5. Cho A1, A2, ..., Ak là các tập con của S = {1, 2, ..., 10} sao cho:i) |Ai|= 5, i = 1, 2, ..., k.ii) |Ai ∩ A j| ≤ 2,1≤ i < j ≤ k. Tìm giá trị lớn nhất của k.

6. (IMO 2001) Có 21 bạn nam và 21 bạn nữ tham dự một kì thi học sinh giỏi toán.Biết rằng:

i) Mỗi bạn giải được nhiều nhất sáu bài.

ii) Mỗi cặp một nam và một nữ thì có ít nhất một bài toán được giải bởi haingười đó

Chứng minh rằng có môt bài toán mà giải được bởi ít nhất 3 nam và 3 nữ.

7. (USAMO 2001) Có 8 hộp, mỗi hộp chứa 6 viên bi. Mỗi viên bi được tô màu saocho:i) Mội hộp chứa các viên bi khác màu.ii) Không có hai màu nào cùng xuất hiện nhiều hơn trong một hộp. Tìm số màuít nhất cần dùng.

Tập san Toán học STAR EDUCATION

Page 63: lovetoan.files.wordpress.com€¦ · 2 NGUYŽN TĂNG VŨ - NGUYŽN NG¯C DUY - VƯƠNG TRUNG DŨNG LÊ PHÚC LÚ - TRƒN BÁ Đ—T T−P SAN TOÁN H¯C STAR EDUCATION SŁ thø 03

62 NGUYỄN TĂNG VŨ

8. (IMO 1989) Cho n và k là các số nguyên dương và S là tập n điểm trong mặtphẳng sao cho:i) Không có 3 điểm nào thẳng hàng,ii) Với điểm P bất kì thuộc S thì có ít nhất k điểm của S cách đều P.

Chứng minh rằng k <12+p

2n

9. (IMO 2005) Trong một cuộc thi toán trong đó đề thi có 6 bài. Mỗi một cặp bài

toán được giải bởi nhiều hơn25

số thí sinh. Không có ai giải được 6 bài. Chứng

minh rằng có ít nhất 2 thí sinh giải được đúng 5 bài.

10. Trong một hội nghị có 35 người tham gia. Biết rằng có 110 cặp đôi một quennhau. Chứng minh rằng có thể chọn ra 4 thành viên xếp ngồi vào một bàn trònsao cho hai người ngồi gần nhau thì quen nhau.

Tập san Toán học STAR EDUCATION

Page 64: lovetoan.files.wordpress.com€¦ · 2 NGUYŽN TĂNG VŨ - NGUYŽN NG¯C DUY - VƯƠNG TRUNG DŨNG LÊ PHÚC LÚ - TRƒN BÁ Đ—T T−P SAN TOÁN H¯C STAR EDUCATION SŁ thø 03

Đề thi tham khảo hướng tới VMO2019-2020

Kỳ thi chọn HSG quốc gia (viết tắt là VMO) năm nay diễn ra vào các ngày 27, 28tháng 12/2019. Trong bài viết này, Ban biên tập chúng tôi sẽ đưa ra giới thiệu một đềthi thử cùng đáp án và phân tích chi tiết. Một số nội dung ở đây đã có giới thiệu tạigroup "Hướng tới VMO-TST" trên Facebook.

1. Đề thi

1.1. Đề thi số 1

1. Cho số thực c > 1 và hàm số f : (0;+∞)→ R liên tục, thỏa mãn f (x) = f (x c)với mọi x > 0.a) Chứng minh rằng f là hàm hằng.b) Khẳng định trên còn đúng không nếu f không phải là hàm số liên tục?

2. Cho hai đa thức hệ số nguyên, monic là P(x),Q(x), trong đó deg P = 3,degQ =2. Giả sử rằng P(x) có ba nghiệm vô tỷ phân biệt có tổng bằng 0 là a, b, c đồngthời Q(a) = b.a) Chứng minh rằng P(Q(x)) chia hết cho P(x).b) Chứng minh rằng Q(a) +Q(b) +Q(c) = 0.

3. Cho tam giác ABC nhọn, không cân nội tiếp trong đường tròn (O) với trực tâmH. Điểm R thay đổi trên cung lớn BC của (O) sao cho AR không song song vớiBC . Lấy các điểm S, T trên đường thẳng BC sao cho (ARS), (ART ) cùng tiếp xúcvới BC . Đường thẳng qua H, vuông góc với AS, AT lần lượt cắt (HBC) ở X , Y.a) Chứng minh rằng đường thẳng X Y luôn đi qua điểm cố định.b) Chứng minh rằng tâm của đường tròn (RST ) di chuyển trên đường thẳng cốđịnh.

4. Tìm tất cả các hàm số f : R → R thỏa mãn điều kiện f (x − 3 f (y)) = f (x +f (y) + y3) + f (4 f (y) + y3) + 1 với mọi x , y ∈ R.

5. Cho tập hợp S gồm n số square-free lớn hơn 1 có tích bằng m là một số nguyêndương có đúng 13 ước nguyên tố phân biệt. Biết rằng bất kỳ 5 số nào trong Scũng không có ước nguyên tố chung và tích 2 số bất kỳ trong S thì không là sốsquare-free.a) Chứng minh rằng n≤ 13.b) Chứng minh rằng khi n = 13 thì m là số chính phương và mỗi số trong S cóđúng 16 ước nguyên dương.

63

Page 65: lovetoan.files.wordpress.com€¦ · 2 NGUYŽN TĂNG VŨ - NGUYŽN NG¯C DUY - VƯƠNG TRUNG DŨNG LÊ PHÚC LÚ - TRƒN BÁ Đ—T T−P SAN TOÁN H¯C STAR EDUCATION SŁ thø 03

64 BAN BIÊN TẬP

1.2. Đề thi số 2

1. Cho dãy số thực (xn) thỏa mãn

x0 = 1, xnn = xn

n−1 + n với n≥ 1.

Chứng minh rằng với mọi n > 1 thì xn − xn−1 <1

2n−1 , từ đó suy ra dãy có giớihạn hữu hạn không vượt quá 3.

2. Cho tam giác ABC nhọn nội tiếp đường tròn (O). Trên các cạnh AB, AC , lần lượtlấy các điểm M , N . Đường tròn đường kính BN , C M cắt nhau tại P,Q. Giả sửrằng P ∈ (O).a) Chứng minh rằng Q thuộc đường tròn Euler của tam giác ABC .b) Chứng minh rằng M , N , O thẳng hàng.

3. Tìm tất cả cặp số nguyên dương m, n sao cho 5m + 3m và 5n + 3n cùng chia hếtcho mn.

4. Với các số nguyên dương m, n, xét bảng ô vuông m× n được chia thành các ôvuông con. Giả sử có thể tô màu trắng – đen bảng này sao cho với mỗi ô vuôngđơn vị thì số ô vuông có cùng màu và có điểm chung với nó (không tính nó) làsố lẻ.a) Chứng minh rằng mn là số chẵn.b) Giả sử mn chẵn, chứng minh rằng có thể tô màu thỏa mãn điều kiện đề bài.

5. Hỏi có bao nhiêu cách sắp xếp các số 1,2, 3, . . . , 10 thành dãy a1, a2, . . . , a10 saocho ak < a[ k

2] với mọi 2≤ k ≤ 10?

6. Cho tam giác ABC nhọn không cân và đường tròn (O) qua B, C cắt các cạnhAB, AC ở D, E. Giả sử BE cắt C D ở I . Gọi M , N là trung điểm BE, C D và MN cắtAB, AC ở P,Q.a) Chứng minh rằng AI là đường đối trung của tam giác APQ.b) Chứng minh rằng (APQ) tiếp xúc với đường tròn đường kính IO.

7. a) Với n ∈ Z+ và x > 0, chứng minh rằng x2n+x2n−2+···+x2+1x2n−1+x2n−3+···+x ≥

n+1n .

b) Tìm n nguyên dương nhỏ nhất để tồn tại đa thức P(x) có bậc 2n thỏa mãni. Đa thức này có ít nhất một nghiệm thực.ii. Tất cả các hệ số đều là các số thực thuộc đoạn [19; 20].

8. Tìm tất cả các hàm số f : R→ R thỏa mãn

f (|x |+ y + f (y + f (y))) = 3y + | f (x)|

với mọi x , y ∈ R.a) Chứng minh rằng f (x) = 0⇒ x = 0 và f là hàm số lẻ.b) Chứng minh rằng f cộng tính trên R+, từ đó tìm tất cả các hàm số thỏa mãnđề bài.

Tập san Toán học STAR EDUCATION

Page 66: lovetoan.files.wordpress.com€¦ · 2 NGUYŽN TĂNG VŨ - NGUYŽN NG¯C DUY - VƯƠNG TRUNG DŨNG LÊ PHÚC LÚ - TRƒN BÁ Đ—T T−P SAN TOÁN H¯C STAR EDUCATION SŁ thø 03

BAN BIÊN TẬP

1.3. Đề thi số 3

1. Tìm tất cả các hàm f : R→ R thỏa mãn

f�

x2 − f 2(y)�

= x f (x) + y2, ∀x , y ∈ R.

2. Cho dãy số (un) được xác định như sau

u1 = 5,

un+1 =3n+ 24n+ 2

(un + 1), ∀n= 1, 2,3, . . .

Chứng minh dãy số đã cho có giới hạn hữu hạn và tìm giới hạn đó.

3. Cho tam giác nhọn không cân ABC có các đường cao AD, BE, C F đồng quy tạitrực tâm H. Gọi K , M lần lượt là trung điểm AH và BC . Giả sử BK cắt AM tại L.a) Chứng minh rằng ∠LED = 90◦.b) Đường thẳng qua tâm ngoại tiếp O của ABC cắt AC ở I . Chứng minh rằngIK ‖ LE.

4. Tìm tất cả bộ ba các số nguyên tố phân biệt (p, q, r) sao cho

3p4 − 5q4 − 4r2 = 26.

5. Trong CLB của Nam, các thành viên có thể quen hoặc không quen biết nhau.Nam nhận thấy rằng: trong mọi khả năng thì chỉ có thể xếp được tối đa 21thành viên của CLB lên 1 ghế dài sao cho 2 bạn ngồi cạnh thì quen nhau. Hỏinếu Nam không ngồi trên ghế đó thì quen được nhiều nhất bao nhiêu bạn ngồitrên ghế?

6. Trên bàn cờ 8× 8, bạn A sẽ đặt vào đó hai quân mã đen, còn bạn B sẽ đặt vàohai quân mã đỏ sao cho không có hai quân mã cùng màu nào có thể ăn nhau(mỗi quân mã một ô). Gọi a là số bàn cờ khác nhau có thể xảy ra (coi vị trí cácô trên bàn cờ là phân biệt). Tìm số dư của a khi chia cho 7.

7. Cho các số a, b, c ∈ R đôi một phân biệt sao cho:• x2 + ax + 1= 0, x2 + bx + c = 0 có nghiệm thực chung.• x2+ x+a = 0, x2+ cx+ b = 0 có nghiệm thực chung. Tính tổng T = a+ b+ c.

8. Cho tam giác ABC nhọn không cân nội tiếp (O), có M là trung điểm BC và điểmD 6= M thay đổi trên đoạn BC . Đường tròn (ABD) cắt AC ở E, đường tròn (AC D)cắt AB ở F. Gọi I là tâm ngoại tiếp tam giác AEF và xét đường tròn (ω) qua B, Ctiếp xúc ngoài với (I) ở T. Giả sử rằng BE ∩ C F = K , AK ∩ EF = R, (I)∩ (O) ={A, P} và AP ∩ BC = S.a. Chứng minh rằng RD⊥BC và ST⊥I T.b. Chứng minh rằng R, T, M thẳng hàng.

2. Lời giải đề thi số 1

Tập san Toán học STAR EDUCATION

Page 67: lovetoan.files.wordpress.com€¦ · 2 NGUYŽN TĂNG VŨ - NGUYŽN NG¯C DUY - VƯƠNG TRUNG DŨNG LÊ PHÚC LÚ - TRƒN BÁ Đ—T T−P SAN TOÁN H¯C STAR EDUCATION SŁ thø 03

66 BAN BIÊN TẬP

Bài 13. Cho dãy số thực (xn) thỏa mãn

x0 = 1, xnn = xn

n−1 + n với n≥ 1.

Chứng minh rằng với mọi n> 1 thì xn− xn−1 <1

2n−1 , từ đó suy ra dãy có giới hạn hữuhạn không vượt quá 3.

Lời giải. Ta có x1 = 2 và dễ thấy dãy tăng ngặt. Với n > 1, ta có n = xnn − xn

n−1 =(xn − xn−1)(xn−1

n + · · ·+ xn−1n−1) > n(xn − xn−1)xn−1

n−1 nên xn − xn−1 <1

xn−1n−1≤ 1

2n−1 . Từ đósuy ra

xn = (xn − xn−1) + (xn−1 − xn−2) + · · ·+ (x2 − x1) + x1

<1

2n−1+ · · ·+

12+ 2< 3.

Dãy tăng và bị chặn trên bởi 3 nên sẽ có giới hạn hữu hạn không vượt quá 3.

Bài 14. Cho tam giác ABC nhọn nội tiếp đường tròn (O). Trên các cạnh AB, AC , lầnlượt lấy các điểm M , N . Đường tròn đường kính BN , C M cắt nhau tại P,Q. Giả sửrằng P ∈ (O).a) Chứng minh rằng Q thuộc đường tròn Euler của tam giác ABC .b) Chứng minh rằng M , N , O thẳng hàng.

Lời giải. a) Gọi BB′, CC ′ là đường cao của tam giác ABC và H là trực tâm. Dễ thấy PQlà trục đẳng phương của (BN), (C M) và B′ ∈ (BN), C ′ ∈ (C M) nên H có cùng phươngtích đến hai đường tròn này. Suy ra H ∈ PQ.

Đặt k = HB · HB′ và xét phép nghịch đảo I kH . Khi đó: P ↔ Q và (O)↔ (Euler). Vì

P ∈ (O) nên Q ∈ (Euler).

Tập san Toán học STAR EDUCATION

Page 68: lovetoan.files.wordpress.com€¦ · 2 NGUYŽN TĂNG VŨ - NGUYŽN NG¯C DUY - VƯƠNG TRUNG DŨNG LÊ PHÚC LÚ - TRƒN BÁ Đ—T T−P SAN TOÁN H¯C STAR EDUCATION SŁ thø 03

BAN BIÊN TẬP

b) Kẻ đường kính BB1, CC1 của (O) thì vì có góc vuông nên dễ thấy B1, P, N và C1, P, Mthẳng hàng. Áp dụng định lý Pascal, ta có MN đi qua O.

Bài 15. Tìm tất cả cặp số nguyên dương m, n sao cho 5m + 3m và 5n + 3n cùng chiahết cho mn.

Lời giải. Trước hết, ta thấy nếu m, n cùng chẵn thì 5m + 3m chia hết cho 4. Trong khiđó 5m + 3m ≡ 1+ (−1)m ≡ 2(mod4),vô lý nên 1 trong 2 số phải lẻ, giả sử là m.Nếu m > 1, ta gọi p là ước nguyên tố lẻ nhỏ nhất của m. Dễ thấy (p, 3) = (p, 5) =1. Khi đó 52m ≡ 32m(modp), 5p−1 ≡ 3p−1 ≡ 1(modp) theo Fermat nhỏ. Suy ra h =gcd(p − 1,2m) cũng thỏa mãn 5h ≡ 3h(modp). Để ý rằng vì p là ước lẻ nhỏ nhất củam nên gcd(p− 1, m) = 1. Suy ra h= 2, kéo theo 52 ≡ 32(mod3)→ p|16, vô lý.Do đó, số p không tồn tại và kéo theo m = 1. Thay vào, ta có 8 chia hết cho n. Thửtrực tiếp, ta thấy n= 1,2 thỏa mãn.Vậy có các cặp số là (m, n) = (1,1), (1, 2), (2,1).

Bài 16. Với các số nguyên dương m, n, xét bảng ô vuông m× n được chia thành cácô vuông con. Giả sử có thể tô màu trắng – đen bảng này sao cho với mỗi ô vuông đơnvị thì số ô vuông có cùng màu và có điểm chung với nó (không tính nó) là số lẻ.a) Chứng minh rằng mn là số chẵn.b) Giả sử mn chẵn, chứng minh rằng có thể tô màu bảng thỏa mãn điều kiện đề bài.

Lời giải. Điều kiện cần: ta gọi ô kề cạnh/kề đỉnh với một ô vuông cho trước là lánggiềng của nó. Giả sử bảng m× n thỏa mãn điều kiện đề bài.Ta xét graph G = (V, E) với V là tất cả các ô được tô đen và hai đỉnh được nối vớinhau nếu chúng là láng giềng của nhau. Theo giả thiết thì mỗi đỉnh của graph nàyđều có bậc lẻ, chứng tỏ số đỉnh phải chẵn (theo bổ đề bắt tay). Do đó, số ô được tôđen là chẵn.Một cách tương tự thì số ô được tô trắng cũng là chẵn nên tổng số ô của bảng là chẵn,kéo theo mn chẵn.Điều kiện đủ: Với mn chẵn, giả sử 2|m thì ta tô màu toàn bộ hai hàng đầu tiên là đen,hai hàng tiếp theo là trắng và cứ thế. Khi đó, mỗi ô có đúng 3 láng giềng cùng màu.Vì thế nên điều kiện cần tìm là mn là số chẵn.

Bài 17. Hỏi có bao nhiêu cách sắp xếp các số 1, 2,3, . . . , 10 thành dãy a1, a2, . . . , a10

sao cho ak < a[ k2] với mọi 2≤ k ≤ 10?

Lời giải. Xét sơ đồ hình cây như trên, trong đó mỗi số ở nút trên sẽ tương ứng lớnhơn số ở nút dưới.Rõ ràng a1 phải lớn nhất trong tất cả các số nên a1 = 10. Hai nhánh hai bên độc lậpvới nhau nên số cách chọn 3 số cho nhánh phải là C3

9 , tiếp theo a3 lớn nhất trong 3số đó nên có 1 cách chọn, a6, a7 tương ứng có 2 cách chọn. Suy ra có 1× 2 cách đểxếp 3 số đó. Tiếp theo, ta sẽ có a2 lớn nhất trong 6 số ở nhánh trái. Tương tự ta cóC3

5 × 1× 2× 1× 1 cách sắp xếp các số ở bên dưới.

Tập san Toán học STAR EDUCATION

Page 69: lovetoan.files.wordpress.com€¦ · 2 NGUYŽN TĂNG VŨ - NGUYŽN NG¯C DUY - VƯƠNG TRUNG DŨNG LÊ PHÚC LÚ - TRƒN BÁ Đ—T T−P SAN TOÁN H¯C STAR EDUCATION SŁ thø 03

68 BAN BIÊN TẬP

Từ đó, theo nguyên lý nhân, số cây như trên, cũng chính là số dãy cần tìm là C39 ×

C35 × 22.

Bài 18. Cho tam giác ABC nhọn không cân và đường tròn (O) qua B, C cắt các cạnhAB, AC ở D, E. Giả sử BE cắt C D ở I . Gọi M , N là trung điểm BE, C D và MN cắtAB, AC ở P,Q.a) Chứng minh rằng AI là đường đối trung của tam giác APQ.b) Chứng minh rằng (APQ) tiếp xúc với đường tròn đường kính IO.

Lời giải. a) Giả sử DE ∩BC = F và MN ∩AF = T. Kéo dài BE, C D cắt AF ở X , Y thì từcác hàng điểm điều hòa, theo hệ thức Maclaurin, ta có I M · IX = IB · I E = IC · I D =IN · IY .

Do đó MNY X nội tiếp. Lại theo hệ thức Newton thì TA2 = T X · T Y = T M · T Nnên TA tiếp xúc với (AMN). Chú ý rằng AM , AN đẳng giác trong góc A nên theo bổđề quen thuộc về đẳng giác, ta cũng có TA tiếp xúc với (APQ). Cuối cùng, vì chùmA(T I , BC) = −1, mà AT là tiếp tuyến nên AI là đối trung của APQ.b) Theo định lý Brocard cho tứ giác toàn phần BC ED.AF thì I là trực tâm tam giácOAF. Giả sử AI ∩ (APQ) = S thì dễ thấy TS cũng là tiếp tuyến của (APQ) do tứ giácAPSQ điều hòa. Do đó TS = TA = T F. Suy ra ∠ASF = 90◦ hay AS⊥FS. Mà AI⊥FOnên S ∈ FO.

Tập san Toán học STAR EDUCATION

Page 70: lovetoan.files.wordpress.com€¦ · 2 NGUYŽN TĂNG VŨ - NGUYŽN NG¯C DUY - VƯƠNG TRUNG DŨNG LÊ PHÚC LÚ - TRƒN BÁ Đ—T T−P SAN TOÁN H¯C STAR EDUCATION SŁ thø 03

BAN BIÊN TẬP

Dễ thấy rằng S là chân đường cao trong tam giác AFO nên TS tiếp xúc với (IO) ở S.Từ đó suy ra (APQ), (IO) tiếp xúc nhau tại S.

Bài 19. a) Với n ∈ Z+ và x > 0, chứng minh rằng x2n+x2n−2+···+x2+1x2n−1+x2n−3+···+x ≥

n+1n .

b) Tìm số nguyên dương n nhỏ nhất sao cho tồn tại đa thức P(x) có bậc 2n thỏa mãni. Đa thức này có ít nhất một nghiệm thực.ii. Tất cả các hệ số đều là các số thực thuộc đoạn [19;20].

Lời giải. a) Nếu x = 1, ta có đẳng thức đúng. Nếu x ∈ (0; 1) thì thay y = 1x > 1, bất

đẳng thức cũng không đổi.Vì thế nên ta có thể giả sử x > 1. Biến đổi được x2n + · · · + x2 + 1 = x2n+2−1

x2−1 và

x2n−1+· · ·+x = x2n+1−xx2−1 . Ta đưa về x2n+2−1

x2n+1−x ≥n+1

n hay nx2n+2−(n+1)x2n+1+(n+1)x−n≥0. Coi vế trái là hàm số f (x) với x > 1. Ta có

f ′(x) = n(2n+ 2)x2n+1 − (n+ 1)(2n+ 1)x2n + (n+ 1)

= (n+ 1)�

2nx2n+1 − (2n+ 1)x2n+1 + 1�

≥ 0

Bất đẳng thức cuối đúng theo AM − GM cho 2n + 1 số. Vì thế nên f đồng biến vàf (x)> f (1) = 0. Ta có đpcm.b) Vì các hệ số của đa thức P(x) đều dương nên nghiệm của nó phải âm. Đặt là −x0

vàP(x) = a2n x2n + a2n−1 x2n−1 + · · ·+ a2 x2 + a1 x + a0.

Suy ra a2n x2n0 + · · ·+ a2 x2

0 + a0 = a2n−1 x2n−10 + · · ·+ a1 x0. Dễ thấy V T ≥ 19(x2n

0 + · · ·+1), V P ≤ 20(x2n−1

0 + · · ·+ x0) nên

2019≥

x2n0 + · · ·+ 1

x2n−10 + · · ·+ x0

≥n+ 1

nhay n≥ 19.

Ta cũng có đa thức P(x) = 19x38+20x37+19x36+20x35+ · · ·+20x +19 có nghiệmx = −1 thỏa mãn đề bài. Vậy nmin = 19.

Bài 20. Tìm tất cả các hàm số f : R → R thỏa mãn f (|x |+ y + f (y + f (y))) =3y + | f (x)| với mọi x , y ∈ R.a) Chứng minh rằng f (x) = 0⇒ x = 0 và f là hàm số lẻ.b) Chứng minh rằng f cộng tính trên R+, từ đó tìm tất cả các hàm số thỏa mãn.

Lời giải. a) Dễ thấy f toàn ánh. Giả sử f (a) = 0 và thay x = 0, y = a, ta có

0= 3a+ | f (0)| .

Suy ra a tồn tại duy nhất và a = −13 | f (0)| ≤ 0. Lại thay x = y = a, ta có f (0) = 3a ≤

0. Lại thay x = −a, y = a thì chú ý rằng |−a|+ a = 0, ta có f (0) = 3a + | f (−a)| nênf (−a) = 0, điều này kéo theo a = −a hay a = 0 (do tính duy nhất ở trên).

Tập san Toán học STAR EDUCATION

Page 71: lovetoan.files.wordpress.com€¦ · 2 NGUYŽN TĂNG VŨ - NGUYŽN NG¯C DUY - VƯƠNG TRUNG DŨNG LÊ PHÚC LÚ - TRƒN BÁ Đ—T T−P SAN TOÁN H¯C STAR EDUCATION SŁ thø 03

70 BAN BIÊN TẬP

Thay y = 0 thì f (|x |) = | f (x)| nên f (x) ≥ 0,∀x ≥ 0. Xét x > 0 và y = − f (x)3 , ta có

f�

x − f (x)3 + f

− f (x)3 + f

− f (x)3

���

= 0 nên

−f (x)

3+ f

−f (x)

3+ f

−f (x)

3

��

= −x

với mọi x > 0. Trong đề bài, thay x = 0 thì f (y + f (y + f (y))) = 3y . Thay y →− f (x)

3 thì f�

− f (x)3 + f

− f (x)3 + f

− f (x)3

���

= − f (x). So sánh hai đẳng thức trên, ta

có f (−x) = − f (x),∀x > 0 nên f là hàm số lẻ.b) Từ tính chất hàm số lẻ, ta có f

f (x)3 + f

f (x)3 + f

f (x)3

���

= f (x) với mọi x > 0.

Trong đề bài, xét x ≥ 0 và y → f (y)3 , ta có

f�

x +f (y)

3+ f

f (y)3+ f

f (y)3

���

= f (y) + f (x)

hay f (x + y) = f (x) + f (y) với mọi x , y > 0. Vì f cộng tính trên R+ nên ta cóf (x) = ax ,∀x > 0. Lại do tính chất hàm lẻ, ta suy ra f (x) = ax ,∀x ∈ R. Thay vàođề bài, ta có a = 1. Vậy tất cả các hàm số cần tìm là f (x) = x .

3. Đề thi số 2

Bài 1. Tìm tất cả các hàm f : R→ R thỏa mãn

f�

x2 − f 2(y)�

= x f (x) + y2, ∀x , y ∈ R.

Lời giải. Thay x = y = 0, ta có f (− f 2(0)) = 0 nên tồn tại u = − f 2(0) để f (u) = 0.

Thay x = 0, y = u, ta có f (0) = u2 nên u= − f 2(0) = −u4 hay

u= 0

u= −1.

Nếu u = −1 thì f (−1) = 0, f (0) = 1. Thay x = −1, y = 0, ta có f (0) = − f (−1) nênu = 0, vô lý. Suy ra f (0) = 0. Thay y = 0, ta có f (x2) = x f (x), lại thay x → −x , dễdàng có f là hàm lẻ. Mặt khác, thay x = 0 thì f (− f 2(y)) = y2 nên f toàn ánh trênR+, kết hợp với hàm lẻ, ta thấy f toàn ánh trên R. Ta viết giả thiết lại thành

f (x2 − f 2(y)) = f (x2) + f (− f 2(y)).

Thay x2, f 2(y) → x , y ≥ 0 ta thấy rằng f (x − y) = f (x) + f (−y) với mọi x , y ≥0. Kết hợp với yếu tố hàm lẻ, dễ dàng có được f cộng tính trên R. Đến đây xétf�

(x + 1)2�

= (x +1) f (x +1) = (x +1)( f (x)+1) và f�

(x + 1)2�

= f (x2+2x +1) =f (x2) + f (2x) + f (1) = x f (x) + 2 f (x) + f (1). So sánh hai đẳng thức trên, dễ dàngcó được f (x) = ax với a = f (1). Thay vào thử lại, ta được a = −1 nên hàm số cầntìm là f (x) = −x .

Bài 2. Cho dãy số (un) được xác định như sau

u1 = 5,

un+1 =3n+ 24n+ 2

(un + 1), ∀n= 1, 2,3, . . .

Tập san Toán học STAR EDUCATION

Page 72: lovetoan.files.wordpress.com€¦ · 2 NGUYŽN TĂNG VŨ - NGUYŽN NG¯C DUY - VƯƠNG TRUNG DŨNG LÊ PHÚC LÚ - TRƒN BÁ Đ—T T−P SAN TOÁN H¯C STAR EDUCATION SŁ thø 03

BAN BIÊN TẬP

Chứng minh dãy số đã cho có giới hạn hữu hạn và tìm giới hạn đó.

Lời giải. Ta có hai cách xử lý như sau:Cách 1. Tính toán trực tiếp, ta dự đoán dãy giảm. Xét hiệu

un+1 − un =(3n+ 2)(un + 1)− (4n+ 2)un

4n+ 2=(3n+ 2)− nun

4n+ 2

nên ta sẽ đưa về chứng minh rằng un ≥ 3+ 2n với mọi n ≥ 1. Quy nạp, với n = 1 thì

u1 = 5, đúng. Giả sử rằng đã có un ≥ 3+ 2n thì

un+1 ≥3n+ 24n+ 2

4+2n

=3n+ 2

n= 3+

2n> 3+

2n+ 1

.

Từ đó suy ra dãy giảm, bị chặn dưới bởi 3 nên hội tụ. Đặt lim un = L ≥ 3 thì L =34(L + 1)⇒ L = 3.

Cách 2. Dự đoán lim un = 3 nên ta viết un+1 − 3= 3n+24n+2(un − 3) + 2

4n+2 nên

|un+1 − 2| ≤3n+ 24n+ 2

|un − 3|+2

4n+ 2≤

56|un − 3|+

24n+ 2

.

Áp dụng bổ đề: Cho hai dãy dương (an), (bn) với lim bn = 0 và an+1 ≤ qan+ bn với mọin. Khi đó ta cũng có lim an = 0. Từ bổ đề suy ra lim |un − 3|= 0 hay lim un = 3.

Bài 3. Cho tam giác nhọn không cân ABC có các đường cao AD, BE, C F đồng quy tạitrực tâm H. Gọi K , M lần lượt là trung điểm AH và BC . Giả sử BK cắt AM tại L.a) Chứng minh rằng ∠LED = 90◦.b) Đường thẳng qua tâm ngoại tiếp O của ABC cắt AC ở I . Chứng minh rằng IK ‖ LE.

Lời giải. Để có ∠LED = 90◦, ta cần có ∠LEA= ∠DEB = ∠F EB. Gọi N là trung điểmEF. Ta thấy rằng hai tia AN , AM đẳng giác trong góc A nên hai tia AN , AL đẳng giáctrong tam giác ABE.

Tập san Toán học STAR EDUCATION

Page 73: lovetoan.files.wordpress.com€¦ · 2 NGUYŽN TĂNG VŨ - NGUYŽN NG¯C DUY - VƯƠNG TRUNG DŨNG LÊ PHÚC LÚ - TRƒN BÁ Đ—T T−P SAN TOÁN H¯C STAR EDUCATION SŁ thø 03

72 BAN BIÊN TẬP

Lại có ABH, EBF đồng dạng nghịch nên BK , BN là hai trung tuyến tương ứng sẽ đẳnggiác với nhau trong góc B. Từ đó ta được L, N là hai điểm liên hợp đẳng giác trongtam giác ABE.Điều này kéo theo ∠LEA= ∠N EB.b) Theo câu a thì ta chỉ cần chứng minh ∠BKI = 90◦ là được. Kẻ I J⊥BC với J ∈ BC .Dễ thấy rằng I J = OM = HK = KA nên có các tứ giác HKIJ , AKJ I là hình bình hành.Suy ra KJ ‖ AC nên BH⊥KJ , chứng tỏ H là trực tâm của tam giác BKJ . Do đó,HJ⊥BK , mà HJ ‖ KI nên ta được BK⊥KI . Ta có đpcm.

Bài 4. Tìm tất cả bộ ba các số nguyên tố phân biệt (p, q, r) sao cho

3p4 − 5q4 − 4r2 = 26.

Lời giải. Xét modulo 3, nếu q, r > 3 thì 3p4 − 5q4 − 4r2 ≡ 0 (mod3), mâu thuẫn. Dođó q = 3 hoặc r = 3. Ta xét hai trường hợp:(1) Nếu q = 3, ta có 3p4 − 4r2 = 431, xét mod5 thì V T ≡ 3 + r2 ≡ V P ≡ 1 nênr2 ≡ −2 (mod5), vô lý.(2) Nếu r = 3 thì 3p4 − 5q4 = 62. Nếu p > 5 thì V T ≡ 3≡ V P ≡ 2 (mod5), vô lý.Do đó, ta có p = 5, và tìm được r = 19.

Bài 5. Trong CLB của Nam, các thành viên có thể quen hoặc không quen biết nhau.Nam nhận thấy rằng: trong mọi khả năng thì chỉ có thể xếp được tối đa 21 thành viêncủa CLB lên 1 ghế dài sao cho 2 bạn ngồi cạnh thì quen nhau. Hỏi nếu Nam khôngngồi trên ghế đó thì quen được nhiều nhất bao nhiêu bạn ngồi trên ghế?

Lời giải. Nam không thể quen hai bạn ngồi hai đầu ghế vì nếu không thì có thể xếpNam ngồi lên dãy ghế, mâu thuẫn với tính lớn nhất. Đánh số các thành viên còn lạingồi trên ghế là 1,2, . . . , 19.

Khi đó, Nam quen được tối đa 10 bạn có số thứ tự lẻ, vì nếu quen nhiều hơn, Nam sẽquen 2 bạn có số thứ tự liên tiếp. Khi đó, Nam có thể ngồi chen giữa vào hai bạn ấyvà cũng mâu thuẫn. Vậy số người Nam có thể quen nhiều nhất là 10.

Bài 6. Trên bàn cờ 8×8, bạn A sẽ đặt vào đó hai quân mã đen, còn bạn B sẽ đặt vàohai quân mã đỏ sao cho không có hai quân mã cùng màu nào có thể ăn nhau (mỗiquân mã một ô). Gọi a là số bàn cờ khác nhau có thể xảy ra (coi vị trí các ô trên bàncờ là phân biệt). Tìm số dư của a khi chia cho 7.

Tập san Toán học STAR EDUCATION

Page 74: lovetoan.files.wordpress.com€¦ · 2 NGUYŽN TĂNG VŨ - NGUYŽN NG¯C DUY - VƯƠNG TRUNG DŨNG LÊ PHÚC LÚ - TRƒN BÁ Đ—T T−P SAN TOÁN H¯C STAR EDUCATION SŁ thø 03

BAN BIÊN TẬP

Lời giải. Xét vị trí quân mã đen – đỏ như các dấu X và dấu O. Gọi S là tập hợp cácvị trí có thể có của 2 dấu X và 2 dấu O. Hai ô ở vị trí (a, b) và (c, d) nào đó mà|(a− c)(b− d)| = 2 được gọi là “liên kết” nhau. Gọi SX ⊂ S là các vị trí mà 2 dấu Xliên kết nhau. Định nghĩa tương tự với SO, SXO. Với mỗi ô trong bảng 8×8, ta viết vàođó số lượng các ô liên kết với nó như bên dưới:

Tổng số lượng các số trên bảng cũng chính bằng hai lần số cặp ô liên kết nhau (theobổ đề bắt tay) và bằng

4× 2+ 8× 3+ 20× 4+ 16× 6+ 16× 82

= 168.

Từ đó, ta dễ dàng tính được |S| = C264 · C

262 ≡ 0 (mod7), |SX | = |SO| = 168 · C2

62 ≡0 (mod7). Mấu chốt vấn đề là tính |SXO| . Tuy nhiên, khi chọn một cặp ô, ta vẫn phảiloại trường hợp hai cặp đó có chung một ô. Ta lại tiếp tục đếm số bộ ba (A, {B, C}) mà(A, B) và (A, C) liên kết nhau. Tổng số bộ là

�S′XO

�= 4× C22 + 8× C2

3 + 20× C24 + 16× C2

6 + 16× C28 ≡ 1 (mod 7).

Do đó |SXO| = C2168 − |S

′XO| ≡ 6 (mod7). Theo nguyên lý bù trừ, ta tính được a =

|S| − (|SX |+ |SO| − |SXO|) chia 7 dư 6.

Bài 7. Cho các số a, b, c ∈ R đôi một phân biệt sao cho:• x2 + ax + 1= 0, x2 + bx + c = 0 có nghiệm thực chung.• x2 + x + a = 0, x2 + cx + b = 0 có nghiệm thực chung. Tính tổng T = a+ b+ c.

Lời giải. Gọi x1, x2 lần lượt là nghiệm chung của hai phương trình bậc hai trên. Ta có

x21 + ax1 + 1= x2

1 + bx1 + c = 0

x22 + x2 + a = 0, x2

2 + cx2 + b = 0.

Suy ra�

(a− b)x1 + 1− c = 0⇒ (a− b)x1 = c − 1

(1− c)x2 + a− b = 0⇒ (c − 1)x2 = a− b

Do a 6= b nên c 6= 1, nhân hai đẳng thức trên lại, ta suy ra x1 x2 = 1. Để ý rằng phươngtrình x2+ax+1= 0 có nghiệm là x1 nên còn nghiệm khác là x ′1 thỏa mãn x1 x ′1 = 1,suy ra x2 = x ′1 hay x2 cũng là nghiệm của x2 + ax + 1 = 0. Gọi x0 là nghiệm chungcủa phương trình x2+ ax + 1= x2+ x + a = 0 thì x2

0 + ax0+ 1= x20 + x0+ a = 0 nên

(a − 1)x0 + 1− a = 0⇔ (a − 1)(x0 − 1) = 0. Dễ thấy a = 1 không thỏa nên x0 = 1,từ đó có a = −2, b+ c = −1 nên T = −3.

Tập san Toán học STAR EDUCATION

Page 75: lovetoan.files.wordpress.com€¦ · 2 NGUYŽN TĂNG VŨ - NGUYŽN NG¯C DUY - VƯƠNG TRUNG DŨNG LÊ PHÚC LÚ - TRƒN BÁ Đ—T T−P SAN TOÁN H¯C STAR EDUCATION SŁ thø 03

74 BAN BIÊN TẬP

Bài 8. Cho tam giác ABC nhọn không cân nội tiếp (O), có M là trung điểm BC vàđiểm D 6= M thay đổi trên đoạn BC . Đường tròn (ABD) cắt AC ở E, đường tròn (AC D)cắt AB ở F. Gọi I là tâm ngoại tiếp tam giác AEF và xét đường tròn (ω) qua B, C tiếpxúc ngoài với (I) ở T. Giả sử rằng BE ∩ C F = K , AK ∩ EF = R, (I)∩ (O) = {A, P} vàAP ∩ BC = S.a. Chứng minh rằng RD⊥BC và ST⊥I T.b. Chứng minh rằng R, T, M thẳng hàng.

Lời giải. a) Từ các tứ giác nội tiếp, ta có ∠AFK +∠AEK = ∠AFC +∠AEC = ∠ADB +∠ADC = 180◦, suy ra AEKF nội tiếp. Áp dụng định lý Brocard cho tứ giác toàn phầnAEKF.BC với chú ý rằng D là điểm Miquel và R là giao điểm hai đường chéo, ta cóRD⊥BC .

Tiếp theo, ta thấy rằng SP ·SA= SB·SC nên S là tâm đẳng phương của (I), (ABC), (ω).Do đó, ST tiếp xúc với (I) và ST⊥AT.

b) Gọi L là tiếp điểm khác T của tiếp tuyến kẻ từ S đến (I).Xét đường tròn (I) thì: BClà đường đối cực của R, đi qua S nên đối cực của S là LT sẽ đi qua R.Suy ra RT⊥SI . Để kết thúc bài toán, ta chỉ cần chứng minh rằng RM⊥SI hay R là trựctâm của tam giác SI M . Để ý rằng R là trực tâm tam giác BIC nên có DR·DI = DB ·DC .Do đó, ta đưa về chứng minh DB · DC = DM · DS hay (SD, BC) = −1. Theo phươngtích thì BD · BC = BF · BA, C D · CB = C E · CA nên BD

C D =BFC E ·

ABAC .

Để ý rằng hai tam giác PBF, PC E đồng dạng nên BFC E =

PBPC . Cũng có SPB, SCA đồng

dạng nên PBAC =

SBSA; tương tự thì SPC , SBA đồng dạng nên PC

AB =SCSA . Chia xuống, ta có

PBPC·

ABAC=

SBSC

.

Kết hợp lại ta được BDC D =

SBSC nên (SD, BC) = −1. Đến đây ta có đpcm.

Tập san Toán học STAR EDUCATION

Page 76: lovetoan.files.wordpress.com€¦ · 2 NGUYŽN TĂNG VŨ - NGUYŽN NG¯C DUY - VƯƠNG TRUNG DŨNG LÊ PHÚC LÚ - TRƒN BÁ Đ—T T−P SAN TOÁN H¯C STAR EDUCATION SŁ thø 03

BAN BIÊN TẬP

4. Lời giải đề thi số 3

Bài 1. Cho số thực c > 1 và hàm số f : (0;+∞) → R liên tục, thỏa mãn f (x) =f (x c) với mọi x > 0. a) Chứng minh rằng f là hàm hằng. b) Khẳng định trên cònđúng không nếu f không phải là hàm số liên tục?

Lời giải. a) Theo giả thiết thì với mọi x > 0, ta có f (x) = f (x1/c) nên thực hiện phépthế liên tiếp thì f (x) = f (x1/cn

). Chú ý rằng limn→+∞

1cn = 0 nên theo tính chất hàm liên

tục, ta có f (x) = limn→+∞

f (x1/cn) = f

limn→+∞

x1/cn�

= f (x0) = f (1). Vì thế nên f là

hàm hằng.b) Khẳng định sẽ không còn đúng nữa. Chẳng hạn ta chọn hàm số f (1) = 1, f (x) =2,∀x 6= 1. Rõ ràng ∀x 6= 1 thì ta đều có x c 6= 1 nên ta luôn có f (x) = f (x c),∀x > 0.Ngoài ra, vì không liên tục nên không thể thực hiện phép thế liên tiếp và lấy giới hạnnhư trên, giá trị của f (1) độc lập với các giá trị còn lại, thế nên hàm số trên thỏa mãnđề bài và rõ ràng nó không phải là hàm hằng.

Nhận xét. Câu hỏi tương tự là: Chứng minh rằng hàm số liên tục f : R+→ R thỏa mãnđiều kiện sau là hàm hằng f (x) = f

x2 + 14

,∀x ∈ R+. Đặc điểm chung của các bàinày là có thể thực hiện phép thế liên tiếp để từ một giá trị x bất kỳ, ta tạo được dãy hộitụ về giá trị cụ thể.

Bài 2. Cho hai đa thức hệ số nguyên, monic là P(x),Q(x), trong đó deg P =3, degQ = 2. Giả sử rằng P(x) có ba nghiệm vô tỷ phân biệt có tổng bằng 0 làa, b, c đồng thời Q(a) = b.a) Chứng minh rằng P(Q(x)) chia hết cho P(x).b) Chứng minh rằng Q(a) +Q(b) +Q(c) = 0.

Lời giải. a) Ta sẽ chứng minh rằng P(x) là đa thức nguyên bậc nhỏ nhất nhận x = alà nghiệm.

Chứng minh. Giả sử ngược lại có đa thức nguyên khác hằng f (x) bậc nhỏ hơn 3 màf (a) = 0, rõ ràng f (x) không thể là bậc nhất vì a là số vô tỷ. Suy ra f bậc hai. Xétphép chia đa thức

P(x) = f (x) · g(x) + r(x)

thì dễ thấy r(a) = 0, và deg r < deg f = 2 nên deg r = 0. Suy ra r(x)≡ 0, tức là P(x)chia hết cho f (x), nên P(x) = f (x)g(x), với g(x) ∈Q[x], chứng tỏ g có nghiệm hữutỷ, và nghiệm đó lại là của P(x), vô lý.Tiếp theo, nếu có đa thức nguyên f1(x) mà f1(a) = 0 thì xét phép chia f1(x) =P(x) · g1(x) + r1(x),ta có r1(a) = 0, mà deg r1 < deg P nên phải có r1(x) ≡ 0, tức làf1(x) chia hết cho P(x).

Trở lại bài toán, rõ ràng P(Q(a)) = P(b) = 0 nên x = a là nghiệm của P(Q(x)), từ cácnhận xét trên, ta phải có P(Q(x)) chia hết cho P(x).b) Theo trên, ta có P(Q(b)) = P(Q(c)) = 0. Suy ra Q(b),Q(c) ∈ {a, b, c}.

Tập san Toán học STAR EDUCATION

Page 77: lovetoan.files.wordpress.com€¦ · 2 NGUYŽN TĂNG VŨ - NGUYŽN NG¯C DUY - VƯƠNG TRUNG DŨNG LÊ PHÚC LÚ - TRƒN BÁ Đ—T T−P SAN TOÁN H¯C STAR EDUCATION SŁ thø 03

76 BAN BIÊN TẬP

Ta xét các trường hợp:Nếu Q(b) = b thì đa thức Q(x)− x có bậc hai nhận nghiệm x = b, vô lý vì P(x) là đathức bậc nhỏ nhất thỏa mãn điều này.Nếu Q(b) = a thì đặt Q(x) = x2 + px + q, ta có hệ

a2 + ap+ q = b

b2 + bp+ q = a

Trừ từng vế, ta được (a− b)(a+ b+ p) = b− a nên a+ b+ p = −1→ c = p+ 1, vô lývì c vô tỷ còn p+1 nguyên. Do đó Q(b) = c. Chứng minh tương tự ta có Q(c) = a nênQ(a) +Q(b) +Q(c) = a+ b+ c = 0.

Nhận xét. Bài toán là một ứng dụng thú vị của “đa thức tối tiểu”, là đa thức nguyênbậc nhỏ nhất nhận một số vô tỷ là nghiệm. Bài toán gốc được lấy từ đề China TST vớinội dung câu hỏi như sau (giả thiết tương tự): Chứng minh rằng p2 − 2p− 4q− 7 là sốchính phương. Ta xử lý tiếp như sau: từ hệ phương trình

a = b2 + pb+ q

b = c2 + pc + q

c = a2 + pa+ q

(*) Trừ từng vế hai phương trình đầu của (∗) , ta có a − b = b2 − c2 + p(b − c) =(b−c)(b+c+p) = (b−c)(p−a). Tương tự thì b−c = (c−a)(p−b) và c−a = (a−b)(p−c)nên nhân tất cả các hệ thức lại thì (p− a)(p− b)(p− c) = 1. Cộng các đẳng thức trong(∗) lại, ta được a2 + b2 + c2 + 3q = 0⇔ ab + bc + ca = 3

2q. Nhân phương trình thứ1, 2,3 của (∗) cho b, c, a rồi cộng lại, ta có

ab+ bc + ca = a3 + b3 + c3 + p(a2 + b2 + c2)⇔3q2= 3abc − 3pq⇔ abc =

q2+ pq.

Thay tất cả vào đẳng thức (p− a)(p− b)(p− c) = 1, ta được

p3 − p2(a+ b+ c) + p(ab+ bc + ca)− abc = 1⇔ p3 +3pq

2−

q2− pq = 1

⇔�

p = 1

q = −2(p2 + p+ 1)

(1) Nếu q = −2(p2+ p+ 1) thì T = p2− 2p+ 8(p2+ p+ 1)− 7= (3p+ 1)2 là số chínhphương.

(2) Nếu p = 1, khi đó abc = 32q và

a = b2 + b+ q

b = c2 + c + q

c = a2 + a+ q

nên

3q2= ab+ bc + ca =

(a2 + a+ q)(b2 + b+ q)

= a2 b2 + b2c2 + c2a2 − 3abc + ab+ bc + ca+ 2q(a2 + b2 + c2) + 3q2

=9q2

4−

9q2+

3q2+ 2q(−3q) + 3q2 = −

3q2

4− 3q

Do đó, ta có q = 0 hoặc q = −6. Rõ ràng q = 0 không thỏa vì khi đó abc = 0 nên phảicó một số bằng 0. Còn nếu q = −6, p = 1 thì T = 16, cũng là số chính phương.

Tập san Toán học STAR EDUCATION

Page 78: lovetoan.files.wordpress.com€¦ · 2 NGUYŽN TĂNG VŨ - NGUYŽN NG¯C DUY - VƯƠNG TRUNG DŨNG LÊ PHÚC LÚ - TRƒN BÁ Đ—T T−P SAN TOÁN H¯C STAR EDUCATION SŁ thø 03

BAN BIÊN TẬP

Bài 3. Cho tam giác ABC nhọn, không cân nội tiếp trong đường tròn (O) với trựctâm H. Điểm R thay đổi trên cung lớn BC của (O) sao cho AR không song song vớiBC . Lấy các điểm S, T trên đường thẳng BC sao cho (ARS), (ART ) cùng tiếp xúc vớiBC . Đường thẳng qua H, vuông góc với AS, AT lần lượt cắt (HBC) ở X , Y.a) Chứng minh rằng đường thẳng X Y luôn đi qua điểm cố định.b) Chứng minh rằng tâm của đường tròn (RST ) di chuyển trên đường thẳng cố định.

Lời giải. a) Gọi K là giao điểm của AR, BC . Theo phương tích thì KB ·KC = KA ·KR=KS2 = KT 2 nên theo hệ thức Newton thì (BC , RS) = −1. Vì chùm A(ST, BC) = −1 nêntrực giao đỉnh H với chú ý BH⊥AC , CH⊥AB, ta có H(MN , BC) = A(ST, CB) = −1.Suy ra tứ giác BMCN điều hòa và MN sẽ đi qua giao điểm hai tiếp tuyến của (HBC)ở B, C . Rõ ràng đó chính là điểm cố định.

b) Bằng biến đổi góc, ta có

∠SRT = ∠SRK +∠TRK = ∠ASK +∠AT K = 180◦ −∠SAT.

Suy ra hai đường tròn (RST ), (AST ) đối xứng nhau qua đường thẳng BC . Khi đó, đểchứng minh tâm của (RST ) thuộc đường thẳng cố định, ta đưa về chứng minh chotâm của (AST ). Gọi D là trung điểm BC và E là giao điểm của AD với (AST ) thì theohệ thức Newton thì DA · DE = DT · DS = DB2 = DC2 nên E là điểm cố định. Do đó,đường tròn (AST ) đi qua điểm cố định A, E nên tâm của nó sẽ di chuyển trên trungtrực của AE cũng cố định. Ta có đpcm.

Nhận xét. Các điểm R, E ở trên lần lượt chính là điểm Humpty của các tam giácAST, ABC . Khai thác tính chất của các điểm này, ta sẽ còn nhiều kết quả thú vị.

Tập san Toán học STAR EDUCATION

Page 79: lovetoan.files.wordpress.com€¦ · 2 NGUYŽN TĂNG VŨ - NGUYŽN NG¯C DUY - VƯƠNG TRUNG DŨNG LÊ PHÚC LÚ - TRƒN BÁ Đ—T T−P SAN TOÁN H¯C STAR EDUCATION SŁ thø 03

78 BAN BIÊN TẬP

Bài 4. Tìm tất cả các hàm số f : R→ R thỏa mãn điều kiện

f (x − 3 f (y)) = f (x + f (y) + y3) + f (4 f (y) + y3) + 1

với mọi x , y ∈ R.

Lời giải. Thay x → 3 f (y) vào phương trình đề cho, ta có f (0) = 2 f (4 f (y)+ y3)+1,hay f (4 f (y) + y3) = f (0)−1

2 = a − 1,∀y ∈ R trong đó a = f (0)+12 . Thay vào phương

trình đề bài, ta cóf (x − 3 f (y)) = f (x + f (y) + y3) + a

với mọi x , y ∈ R. Thay y → 0, x → x + f (y) + y3 − f (0), ta có f (x + f (y) + y3 −f (0)− 3 f (y)) = f (x + f (y) + y3) + a,∀x , y ∈ R hay tương đương

f (x − 2 f (y) + y3 − f (0)) = f (x + f (y) + y3) + a,∀x , y ∈ R.

So sánh hai phương trình trên ta suy ra f (x−3 f (y)) = f (x−2 f (y)+y3− f (0)),∀x , y ∈R. Thay x → x + 3 f (y), ta suy ra f (x) = f (x + f (y) + y3 − f (0)),∀x , y ∈ R. Nếuf (y)+ y3− f (0) = 0,∀y thì thử lại thấy f (y) = −y3+ c không là nghiệm của phươngtrình. Ngược lại, nếu tồn tại y0 thỏa f (y0)+ y3

0 − f (0) = c 6= 0 thay vào phương trìnhtrên, ta suy ra f (x) = f (x + c),∀x ∈ R. Khi đó thay y → y + c vào đẳng thức trên, tacó

f (x) = f (x + f (y) + (y + c)3 − f (0)) = f (x + f (y) + y3 − f (0),∀x , y ∈ R.

Thay x → x − f (y)− y3+ f (0) ta lại có f (x) = f (x + (y + c)3− y3),∀x , y ∈ R.. Nếuc > 0 thì (y + c)3− y3 đa thức bậc 2 có hệ số bậc cao nhất dương nên nhận giá trị trongtập [m;+∞) với m là một số thực nào đó. Do đó f (x) = f (x + r),∀x ∈ [m;+∞).Lấy x , y ∈ R bất kỳ mà x > y . Khi đó tồn tại q, r ∈ [m;+∞) thỏa mãn q− r = x − y .Ta có f (y) = f (y + q) = f (y + q − r) = f (y + x − y) = f (x). Điều này chứng tỏf là hàm hằng. Tương tự nếu c < 0 ta cũng suy ra f hằng. Thay vào đề bài, ta cóf (x) = −1,∀x thỏa mãn.Vậy tất cả các hàm số cần tìm là f (x) = −1,∀x ∈ R.

Nhận xét. Bài toán trên đây được phát triển từ bài toán

f (x − f (y)) = f (x + y2n) + f ( f (y) + y2n) + 1,

với mọi x , y ∈ R. Ở trong bài toán này, sau một vài bước xử lí, ta thu được một trườnghợp f là hàm tuần hoàn với chu kì c. Từ đó thay vào phương trình đề cho, ta sẽ cóf (x) = f (x + (y + c)2n− y2n), với mọi x , y ∈ R. Đến đây, do c 6= 0 nên (y + c)2n− y2n

là đa thức bậc 2n− 1 theo y , tức sẽ phủ hết các giá trị của tập R. Từ đó ta có f (x) =f (x + r), với mọi x , r ∈ R.Điều này chứng tỏ f là hàm hằng. Tuy nhiên, ở đây, đa thức (y + c)3 − y3 là đa thứcbậc hai nên không thể phủ hết giá trị của tập R mà thay vào đó là [m,+∞) với m ∈ Rnào đó (nếu c > 0) và (−∞, m] (nếu c < 0). Điều này đòi hỏi cách xử lí khéo léo hơn.Một trong những cách đó là: lấy x > y ∈ R bất kì, ta chứng minh f (x) = f (y). Thậtvậy, vì f (x) = f (x + r) với mọi x ∈ R, r ∈ [m,+∞) nên chọn q = m+ x − y.

Tập san Toán học STAR EDUCATION

Page 80: lovetoan.files.wordpress.com€¦ · 2 NGUYŽN TĂNG VŨ - NGUYŽN NG¯C DUY - VƯƠNG TRUNG DŨNG LÊ PHÚC LÚ - TRƒN BÁ Đ—T T−P SAN TOÁN H¯C STAR EDUCATION SŁ thø 03

BAN BIÊN TẬP

Bài 5. Cho tập hợp S gồm n số square-free lớn hơn 1 có tích bằng m là một số nguyêndương có đúng 13 ước nguyên tố phân biệt. Biết rằng bất kỳ 5 số nào trong S cũngkhông có ước nguyên tố chung và tích 2 số bất kỳ trong S thì không là số square-free.a) Chứng minh rằng n ≤ 13. b) Chứng minh rằng khi n = 13 thì m là số chínhphương và mỗi số trong S có đúng 16 ước nguyên dương.

Lời giải. a) Gọi P là tập hợp các ước nguyên tố của m. Theo giả thiết, ta nhận xét:- Mỗi số nguyên tố thuộc P sẽ là ước của không quá 4 số thuộc S.- Vì tích của hai số square-free thuộc S không phải là số square-free nên chúng phảicó ước nguyên tố chung.Từ đó, đếm số bộ ({a, b}, p) mà a, b ∈ S, p ∈ P và p là ước chung của a, b.• Cách 1. Đếm theo {a, b}, ta có ≥ C2

n · 1.• Cách 2. Đếm theo p, ta có ≤ 13 · C2

4 = 78.Do đó C2

n ≤ 78→ n(n− 1)≤ 156 nên n≤ 13.b) Khi n = 13, dễ thấy rằng khi đẳng thức xảy ra mỗi số nguyên tố là ước của đúng4 số nên m là một lũy thừa bậc 4 của tích các số nguyên tố, nói cách khác, nó là sốchính phương. Ngoài ra, hai số bất kỳ trong S đều phải có ước nguyên tố chung.Cuối cùng nếu có số a ∈ S nào đó chỉ có ≤ 3 ước nguyên tố thì mỗi ước đó sẽ thuộcvề đúng 3 số nữa. Khi đó, còn lại 13− 1− 3× 3 = 3 số sẽ không có ước chung với a,mâu thuẫn. Suy ra mỗi số trong S đều phải có ít nhất 4 ước nguyên tố.Đến đây, đếm số bộ (a, p) với a ∈ S, p ∈ P và p|a, ta thấy rằng nếu đếm theo a thì sẽcó ≥ 13 ·4= 52, còn đếm theo p thì sẽ có đúng 13 ·4= 52 nên đẳng thức phải xảy ra,tức là mỗi số trong S đều có đúng 4 ước nguyên tố nên số ước đúng bằng 24 = 16.

Nhận xét. Bài toán trên chính là một trường hợp đặc biệt của bài toán tổng quát thúvị sau: Một CLB có n > 1 học sinh được phân hoạch thành m > 1 nhóm nhỏ thỏa mãn:mỗi nhóm có số thành viên bằng nhau; hai nhóm tùy ý có đúng 1 thành viên chung; haithành viên tùy ý tham gia chung đúng 1 nhóm.Chứng minh rằng 2(m+ n)− 3 là số chính phương.

Trước hết, đếm số bộ (học sinh, học sinh, nhóm) mà hai học sinh cùng tham gia vàonhóm, ta có

C2n = mC2

k ⇔ n(n− 1) = k(k− 1)m.

Xét một học sinh A tùy ý và gọi sA là số nhóm mà A tham gia, giả sử trong đó có nhóm1. Ta sẽ đếm số bộ (học sinh, nhóm) mà học sinh khác A, nhóm khác 1 và học sinh nàycùng với A thuộc vào nhóm đó. Đếm theo học sinh, có n− k cách; còn đếm theo nhóm, có(sA− 1)(k − 1) cách. Từ đó suy ra sA− 1 = n−k

k−1 ⇔ sA =n−1k−1 . Do giá trị này cố định nên

các học sinh đều tham gia vào đúng n−1k−1 nhóm. * Đếm số bộ (nhóm, nhóm, học sinh) mà

hai nhóm có học sinh tham gia chung, ta có C2m = nC2

n−1k−1

hay (k− 1)(m− 1) = k(n− k).

Giải hệ, ta có m= n= k2 − k+ 1 nên

2(m+ n)− 3= 4(k2 − k+ 1)− 3= (2k− 1)2

là số chính phương. Khi m> k2−k+1, cấu trúc đẹp trên bị “phá vỡ”, ta đưa về mô hìnhđịnh lý sunflower: Xét một tập hợp n phần tử và m tập con của nó, mỗi tập có k phần tửsao cho hai tập bất kỳ đều có đúng một phần tử chung. Khi đó nếu như m > k2 − k+ 1thì sẽ có 1 phần tử thuộc về tất cả các tập con. Kết quả này chứng minh không khó bằngnguyên lý Dirichlet.

Tập san Toán học STAR EDUCATION